Anda di halaman 1dari 115

petitioners herein.

ELIS V. REPUBLIC Art. 15 adheres to the theory that jurisdiction over the status of a natural person
7 SCRA 962 is determined by the latters' nationality.
Petitioner Marvin G. Ellis, a native of San Fransisco, California and is married to Pursuant to this theory, the Court has jurisdiction over the status of Baby Rose, she
Gloria G. Ellis in Banger, Maine, United States. being a citizen of the Philippines, but not over
Both are citizens of the United States. Baby Rose was born on September 26, 1959. the status of the petitioners, who are foreigners.
Four or five days later, the mother of Rose Under the Philippine’s political law, which is patterned after the Anglo-American
left her with the Heart of Mary Villa — an institution for unwed mothers and their legal system, the Court have, likewise,
babies — stating that she (the mother) could adopted the latter's view to the effect that personal status, in general, is
not take of Rose without bringing disgrace upon her (the mother's family.). determined by and/or subject to the jurisdiction
Being without issue, on November 22, 1959, Mr. and Mrs. Ellis filed a petition with the of the domiciliary law (Restatement of the Law of Conflict of Laws, p. 86; The
Court of First Instance of Pampanga for Conflict of Laws by Beale, Vol. I, p. 305, Vol. II,
the adoption of the aforementioned baby. At the time of the hearing of the petition on pp. 713-714).
January 14, 1960, petitioner Marvin G. This, perhaps, is the reason why our Civil Code does not permit adoption by non-
Ellis and his wife had been in the Philippines for three (3) years, he being assigned resident aliens, and we have consistently
thereto as staff sergeant in the United States refused to recognize the validity of foreign decrees of divorce — regardless of the
Air Force Base, in Angeles, Pampanga where both lived at that time. They had been in grounds upon which the same are based
the Philippines before, or, to exact, in — involving citizens of the Philippines who are not bona fide residents of the
1953. forum, even when our laws authorized absolute
Issue: divorce in the Philippines (Ramirez v. Gmur, 42 Phil. 855; Gonayeb v. Hashim, 30 Phil.
Whether or not being permanent residents in the Philippines, petitioners are qualified to 22; Cousine Hix v. Fleumer, 55 Phil.
adopt Baby Rose.
Held: 851; Barretto Gonzales v. Gonzales, 58 Phil. 67; Recto v. Harden, L-6897, Nov. 29,
Inasmuch as petitioners herein are not domiciled in the Philippines — and, hence, non- 1955)".
resident aliens – the Court cannot assume
and exercise jurisdiction over the status, under either the nationality theory or the
domiciliary theory.
Article 335 of the Civil Code of the Philippines, provides that:
"The following cannot adopt:
xxxxxxxxx
(4) Non-resident aliens;".
xxxxxxxxx
This legal provisions is too clear to require interpretation.
Since adoption is a proceedings in rem, no court may entertain unless it has
jurisdiction, not only over the subject matter
of the case and over the parties, but also over the res, which is the personal status
of Baby Rose as well as that of
1
"The following cannot adopt:
G.R. No. L-16922 April 30, 1963
xxx xxx x x xchanrobles virtual law library
IN RE: ADOPTION OF CHILD BAPTIZED UNDER THE NAME OF ROSE,
MARVIN G. ELLIS and GLORIA C. ELLIS, petitioners-appellees, vs. REPUBLIC OF (4) Non-resident aliens;".
THE PHILIPPINES, oppositor-appellant.
xxx xxx xxx
Leonardo F. Lansangan for petitioners-appellees.
Office of the Solicitor General for oppositor-appellant. This legal provisions is too clear to require interpretation. No matter how much we
sympathize with the plight of Baby Rose and with the good intentions of petitioners
CONCEPCION, J.:chanrobles virtual law library herein, the law leaves us no choice but to apply its explicit terms, which unqualified
deny to petitioners the power to adopt anybody in the
Appeal taken by the Government from a decision of the Court of First Instance of Philippines.chanroblesvirtualawlibrarychanrobles virtual law library
Pampanga granting the petition of Marvin G. Ellis and Gloria C. Ellis for the adoption
of a Filipino baby girl named Rose.chanroblesvirtualawlibrarychanrobles virtual law In this connection, it should be noted that this is a proceedings in rem, which no court
library may entertain unless it has jurisdiction, not only over the subject matter of the case and
over the parties, but also over the res, which is the personal status of Baby Rose as well
Petitioner Marvin G. Ellis, a native of San Fransisco, California, is 28 years of age. On as that of petitioners herein. Our Civil Code (Art. 15) adheres to the theory that
September 3, 1949, he married Gloria G. Ellis in Banger, Maine, United States. Both jurisdiction over the status of a natural person is determined by the latters' nationality.
are citizens of the United States. Baby Rose was born on September 26, 1959, at the Pursuant to this theory, we have jurisdiction over the status of Baby Rose, she being a
Caloocan Maternity Hospital. Four or five days later, the mother of Rose left her with citizen of the Philippines, but not over the status of the petitioners, who are foreigners.
the Heart of Mary Villa - an institution for unwed mothers and their babies - stating that Under our political law, which is patterned after the Anglo-American legal system, we
she (the mother) could not take of Rose without bringing disgrace upon her (the have, likewise, adopted the latter's view to the effect that personal status, in general, is
mother's family.).chanroblesvirtualawlibrarychanrobles virtual law library determined by and/or subject to the jurisdiction of the domiciliary law (Restatement of
the Law of Conflict of Laws, p. 86; The Conflict of Laws by Beale, Vol. I, p. 305, Vol.
Being without issue, on November 22, 1959, Mr. and Mrs. Ellis filed a petition with the II, pp. 713-714). This, perhaps, is the reason why our Civil Code does not permit
Court of First Instance of Pampanga for the adoption of the aforementioned baby. At adoption by non-resident aliens, and we have consistently refused to recognize the
the time of the hearing of the petition on January 14,1960, petitioner Marvin G. Ellis validity of foreign decrees of divorce - regardless of the grounds upon which the same
and his wife had been in the Philippines for three (3) years, he being assigned thereto as are based - involving citizens of the Philippines who are not bona fide residents of the
staff sergeant in the United States Air Force Base, in Angeles, Pampanga where both forum, even when our laws authorized absolute divorce in the Philippines (Ramirez v.
lived at that time. They had been in the Philippines before, or, to exact, in Gmur, 42 Phil. 855; Gonayeb v. Hashim, 30 Phil. 22; Cousine Hix v. Fleumer, 55 Phil.
1953.chanroblesvirtualawlibrarychanrobles virtual law library 851; Barretto Gonzales v. Gonzales, 58 Phil. 67; Recto v. Harden, L-6897, Nov. 29,
1955)".chanroblesvirtualawlibrarychanrobles virtual law library
The only issue in this appeal is whether, not being permanent residents in the
Philippines, petitioners are qualified to adopt Baby Rose. Article 335 of the Civil Code Inasmuch as petitioners herein are not domiciled in the Philippines - and, hence, non-
of the Philippines, provides that: resident aliens - we cannot assume and exercise jurisdiction over the status, under
either the nationality theory or the domiciliary theory. In any event, whether the above -
2
quoted provision of said Art. 335 is predicated upon lack of jurisdiction over the res or
merely affects the cause of action, we have no authority to grant the relief prayed for by
petitioners herein, and it has been so held in Caraballo v. Republic, L-15080 (April 25,
1962) and Katansik v. Republic L-15472 (June 30,
1962).chanroblesvirtualawlibrarychanrobles virtual law library

WHEREFORE, the decision appealed from is hereby reversed, and another one shall be
entered denying the petition in this case.

3
Tecson v. Comelec absence of any other evidence, could
424 SCRA 277 have well been his place of residence before death, such that Lorenzo Pou would
Facts: have benefited from the "en masse
On 31 December 2003, Ronald Allan Kelly Poe, also known as Fernando Poe, Jr. have well been his place of residence before death, such that Lorenzo Pou would
(FPJ), filed his certificate of candidacy have benefited from the "en masse
for the position of President of the Republic of the Philippines under the Koalisyon Filipinization" that the Philippine Bill had effected in 1902. That citizenship (of
ng Nagkakaisang Pilipino (KNP) Party, in Lorenzo Pou), if acquired, would thereby
the 2004 national elections. In his certificate of candidacy, FPJ, representing himself extend to his son, Allan F. Poe, father of respondent FPJ. The 1935 Constitution,
to be a natural-born citizen of the during which regime respondent FPJ has
Philippines, stated his name to be "Fernando Jr.," or "Ronald Allan" Poe, his date seen first light, confers citizenship to all persons whose fathers are Filipino citizens
of birth to be 20 August 1939 and his regardless of whether such children are legitimate or illegitimate.
place of birth to be Manila. Victorino X. Fornier, initiated, on 9 January 2004, a
petition before the Commission on Elections
(COMELEC) to disqualify FPJ and to deny due course or to cancel his certificate of
candidacy upon the thesis that FPJ made a
material misrepresentation in his certificate of candidacy by claiming to be a
natural-born Filipino citizen when in truth,
according to Fornier, his parents were foreigners; his mother, Bessie Kelley Poe,
was an American, and his father, Allan
Poe, was a Spanish national, being the son of Lorenzo Pou, a Spanish subject. And
even if Allan F. Poe was a Filipino
citizen, he could not have transmitted his Filipino citizenship to FPJ, the latter being an
illegitimate child of an alien mother.
Fornier based the allegation of the illegitimate birth of FPJ on two assertions: (1) Allan
F. Poe contracted a prior marriage to a
certain Paulita Gomez before his marriage to Bessie Kelley and, (2) even if no such
prior marriage had existed, Allan F. Poe,
married Bessie Kelly only a year after the birth of FPJ.
Issue:
Whether FPJ was a natural born citizen, so as to be allowed to run for the offcie of the
President of the Philippines.
Held:
Any conclusion on the Filipino citizenship of Lorenzo Pou could only be drawn from
the presumption that having died in 1954
at 84 years old, Lorenzo would have been born sometime in the year 1870, when
the Philippines was under Spanish rule,
and that San Carlos, Pangasinan, his place of residence upon his death in 1954, in the
4
[G.R. No. 161434. March 3, 2004] MARIA JEANETTE C. TECSON and FELIX B. of President of the Republic of the Philippines under the Koalisyon ng Nagkakaisang
DESIDERIO, JR., petitioners, vs. The COMMISSION ON ELECTIONS, RONALD Pilipino (KNP) Party, in the forthcoming national elections. In his certificate of
ALLAN KELLY POE (a.k.a. FERNANDO POE, JR.) and VICTORINO X. candidacy, FPJ, representing himself to be a natural-born citizen of the Philippines,
FORNIER, respondents. stated his name to be "Fernando Jr.," or "Ronald Allan" Poe, his date of birth to be 20
August 1939 and his place of birth to be Manila.
[G.R. No. 161634. March 3, 2004] ZOILO ANTONIO VELEZ, petitioner, vs.
Victorino X. Fornier, petitioner in G.R. No. 161824, entitled "Victorino X. Fornier,
RONALD ALLAN KELLEY POE, a.k.a. FERNANDO POE, JR., respondent.
Petitioner, versus Hon. Commission on Elections and Ronald Allan Kelley Poe, also
known as Fernando Poe, Jr., Respondents," initiated, on 09 January 2004, a petition
[G. R. No. 161824. March 3, 2004] VICTORINO X. FORNIER, petitioner, vs. HON.
docketed SPA No. 04-003 before the Commission on Elections ("COMELEC") to
COMMISSION ON ELECTIONS and RONALD ALLAN KELLEY POE, ALSO
disqualify FPJ and to deny due course or to cancel his certificate of candidacy upon the
KNOWN AS FERNANDO POE JR., respondents.
thesis that FPJ made a material misrepresentation in his certificate of candidacy by
claiming to be a natural-born Filipino citizen when in truth, according to Fornier, his
DECISION
parents were foreigners; his mother, Bessie Kelley Poe, was an American, and his
VITUG, J.: father, Allan Poe, was a Spanish national, being the son of Lorenzo Pou, a Spanish
subject. Granting, petitioner asseverated, that Allan F. Poe was a Filipino citizen, he
Citizenship is a treasured right conferred on those whom the state believes are could not have transmitted his Filipino citizenship to FPJ, the latter being an
deserving of the privilege. It is a precious heritage, as well as an inestimable illegitimate child of an alien mother. Petitioner based the allegation of the illegitimate
acquisition,[1] that cannot be taken lightly by anyone - either by those who enjoy it or by birth of respondent on two assertions - first, Allan F. Poe contracted a prior marriage to
those who dispute it. a certain Paulita Gomez before his marriage to Bessie Kelley and, second, even if no
such prior marriage had existed, Allan F. Poe, married Bessie Kelly only a year after
Before the Court are three consolidated cases, all of which raise a single question the birth of respondent.
of profound importance to the nation. The issue of citizenship is brought up to
challenge the qualifications of a presidential candidate to hold the highest office of the In the hearing before the Third Division of the COMELEC on 19 January 2004,
land. Our people are waiting for the judgment of the Court with bated breath. Is petitioner, in support of his claim, presented several documentary exhibits - 1) a copy
Fernando Poe, Jr., the hero of silver screen, and now one of the main contenders for the of the certificate of birth of FPJ, 2) a certified photocopy of an affidavit executed in
presidency, a natural-born Filipino or is he not? Spanish by Paulita Poe y Gomez attesting to her having filed a case for bigamy and
concubinage against the father of respondent, Allan F. Poe, after discovering his
The moment of introspection takes us face to face with Spanish and American bigamous relationship with Bessie Kelley, 3) an English translation of the affidavit
colonial roots and reminds us of the rich heritage of civil law and common law aforesaid, 4) a certified photocopy of the certificate of birth of Allan F. Poe, 5) a
traditions, the fusion resulting in a hybrid of laws and jurisprudence that could be no certification issued by the Director of the Records Management and Archives Office,
less than distinctly Filipino. attesting to the fact that there was no record in the National Archives that a Lorenzo
Poe or Lorenzo Pou resided or entered the Philippines before 1907, and 6) a
certification from the Officer-In-Charge of the Archives Division of the National
Antecedent Case Settings Archives to the effect that no available information could be found in the files of the
National Archives regarding the birth of Allan F. Poe.
On 31 December 2003, respondent Ronald Allan Kelly Poe, also known as On his part, respondent, presented twenty-two documentary pieces of evidence, the
Fernando Poe, Jr. (hereinafter "FPJ"), filed his certificate of candidacy for the position more significant ones being - a) a certification issued by Estrella M. Domingo of the
5
Archives Division of the National Archives that there appeared to be no available deny due course to or cancel FPJs certificate of candidacy for alleged misrepresentation
information regarding the birth of Allan F. Poe in the registry of births for San Carlos, of a material fact (i.e., that FPJ was a natural-born citizen) before the COMELEC,
Pangasinan, b) a certification issued by the Officer-In-Charge of the Archives Division petitioner Fornier invoked Section 78 of the Omnibus Election Code
of the National Archives that no available information about the marriage of Allan F.
Poe and Paulita Gomez could be found, c) a certificate of birth of Ronald Allan Poe, d) Section 78. Petition to deny due course to or cancel a certificate of candidacy. --- A
Original Certificate of Title No. P-2247 of the Registry of Deeds for the Province of verified petition seeking to deny due course or to cancel a certificate of candidacy may
Pangasinan, in the name of Lorenzo Pou, e) copies of Tax Declaration No. 20844, No. be filed by any person exclusively on the ground that any material representation
20643, No. 23477 and No. 23478 in the name of Lorenzo Pou, f) a copy of the contained therein as required under Section 74 hereof is false
certificate of death of Lorenzo Pou, g) a copy of the purported marriage contract
between Fernando Pou and Bessie Kelley, and h) a certification issued by the City Civil in consonance with the general powers of COMELEC expressed in Section 52 of the
Registrar of San Carlos City, Pangasinan, stating that the records of birth in the said Omnibus Election Code -
office during the period of from 1900 until May 1946 were totally destroyed during
World War II. Section 52. Powers and functions of the Commission on Elections. In addition to the
powers and functions conferred upon it by the Constitution, the Commission shall have
On 23 January 2004, the COMELEC dismissed SPA No. 04-003 for lack of
exclusive charge of the enforcement and administration of all laws relative to the
merit. Three days later, or on 26 January 2004, Fornier filed his motion for
conduct of elections for the purpose of ensuring free, orderly and honest elections -
reconsideration. The motion was denied on 06 February 2004 by the COMELEC en
banc. On 10 February 2004, petitioner assailed the decision of the COMELEC before
and in relation to Article 69 of the Omnibus Election Code which would authorize "any
this Court conformably with Rule 64, in relation to Rule 65, of the Revised Rules of
interested party" to file a verified petition to deny or cancel the certificate of candidacy
Civil Procedure. The petition, docketed G. R. No. 161824, likewise prayed for a
of any nuisance candidate.
temporary restraining order, a writ of preliminary injunction or any other resolution that
would stay the finality and/or execution of the COMELEC resolutions. Decisions of the COMELEC on disqualification cases may be reviewed by the
Supreme Court per Rule 64[2] in an action for certiorari under Rule 65[3] of the Revised
The other petitions, later consolidated with G. R. No. 161824, would include G. R.
Rules of Civil Procedure. Section 7, Article IX, of the 1987 Constitution also reads
No. 161434, entitled "Maria Jeanette C. Tecson, and Felix B. Desiderio, Jr., vs. The
Commission on Elections, Ronald Allan Kelley Poe (a.k.a. Fernando Poe, Jr.), and
"Each Commission shall decide by a majority vote of all its Members any case or
Victorino X. Fornier," and the other, docketed G. R. No. 161634, entitled "Zoilo
matter brought before it within sixty days from the date of its submission for decision
Antonio G. Velez, vs. Ronald Allan Kelley Poe, a.k.a. Fernando Poe, Jr.," both
or resolution. A case or matter is deemed submitted for decision or resolution upon the
challenging the jurisdiction of the COMELEC and asserting that, under Article VII,
filing of the last pleading, brief, or memorandum, required by the rules of the
Section 4, paragraph 7, of the 1987 Constitution, only the Supreme Court had original
Commission or by the Commission itself. Unless otherwise provided by this
and exclusive jurisdiction to resolve the basic issue on the case.
Constitution or by law, any decision, order, or ruling of each Commission may be
brought to the Supreme Court on certiorari by the aggrieved party within thirty days
from receipt of a copy thereof."
Jurisdiction of the Court
Additionally, Section 1, Article VIII, of the same Constitution provides that
In G. R. No. 161824 judicial power is vested in one Supreme Court and in such lower courts as may be
established by law which power includes the duty of the courts of justice to settle actual
In seeking the disqualification of the candidacy of FPJ and to have the COMELEC controversies involving rights which are legally demandable and enforceable, and to
6
determine whether or not there has been a grave abuse of discretion amounting to lack election scenario. Election contests consist of either an election protest or a quo
or excess of jurisdiction on the part of any branch or instrumentality of the warranto which, although two distinct remedies, would have one objective in view, i.e.,
Government. to dislodge the winning candidate from office. A perusal of the phraseology in Rule 12,
Rule 13, and Rule 14 of the "Rules of the Presidential Electoral Tribunal," promulgated
It is sufficiently clear that the petition brought up in G. R. No. 161824 was aptly
by the Supreme Court en banc on 18 April 1992, would support this premise -
elevated to, and could well be taken cognizance of by, this Court. A contrary view
could be a gross denial to our people of their fundamental right to be fully informed,
Rule 12. Jurisdiction. - The Tribunal shall be the sole judge of all contests relating to
and to make a proper choice, on who could or should be elected to occupy the highest
the election, returns, and qualifications of the President or Vice-President of
government post in the land.
the Philippines.
In G. R. No. 161434 and G. R. No. 161634
Rule 13. How Initiated. - An election contest is initiated by the filing of an election
Petitioners Tecson, et al., in G. R. No. 161434, and Velez, in G. R. No. 161634,
protest or a petition for quo warranto against the President or Vice-President. An
invoke the provisions of Article VII, Section 4, paragraph 7, of the 1987 Constitution in
election protest shall not include a petition for quo warranto. A petition for quo
assailing the jurisdiction of the COMELEC when it took cognizance of SPA No. 04-
warranto shall not include an election protest.
003 and in urging the Supreme Court to instead take on the petitions they directly
instituted before it. The Constitutional provision cited reads:
Rule 14. Election Protest. - Only the registered candidate for President or for Vice-
President of the Philippines who received the second or third highest number of votes
"The Supreme Court, sitting en banc, shall be the sole judge of all contests relating to
may contest the election of the President or the Vice-President, as the case may be, by
the election, returns, and qualifications of the President or Vice-President, and may
filing a verified petition with the Clerk of the Presidential Electoral Tribunal within
promulgate its rules for the purpose."
thirty (30) days after the proclamation of the winner.
The provision is an innovation of the 1987 Constitution. The omission in the 1935 and
The rules categorically speak of the jurisdiction of the tribunal over contests
the 1973 Constitution to designate any tribunal to be the sole judge of presidential and
relating to the election, returns and qualifications of the "President" or "Vice-
vice-presidential contests, has constrained this Court to declare, in Lopez vs. Roxas,
[4] President", of the Philippines, and not of "candidates" for President or Vice-
as not (being) justiciable controversies or disputes involving contests on the
President. A quo warranto proceeding is generally defined as being an action against a
elections, returns and qualifications of the President or Vice-President. The
person who usurps, intrudes into, or unlawfully holds or exercises a public office. [5] In
constitutional lapse prompted Congress, on 21 June 1957, to enact Republic Act No.
such context, the election contest can only contemplate a post-election scenario. In
1793, "An Act Constituting an Independent Presidential Electoral Tribunal to Try, Hear
Rule 14, only a registered candidate who would have received either the second or third
and Decide Protests Contesting the Election of the President-Elect and the Vice-
highest number of votes could file an election protest. This rule again presupposes a
President-Elect of the Philippines and Providing for the Manner of Hearing the
post-election scenario.
Same." Republic Act 1793 designated the Chief Justice and the Associate Justices of
the Supreme Court to be the members of the tribunal. Although the subsequent It is fair to conclude that the jurisdiction of the Supreme Court, defined by Section
adoption of the parliamentary form of government under the 1973 Constitution might 4, paragraph 7, of the 1987 Constitution, would not include cases directly brought
have implicitly affected Republic Act No. 1793, the statutory set-up, nonetheless, before it, questioning the qualifications of a candidate for the presidency or vice-
would now be deemed revived under the present Section 4, paragraph 7, of the 1987 presidency before the elections are held.
Constitution.
Accordingly, G. R. No. 161434, entitled "Maria Jeanette C. Tecson, et al., vs.
Ordinary usage would characterize a "contest" in reference to a post- Commission on Elections et al.," and G. R. No. 161634, entitled "Zoilo Antonio Velez
7
vs. Ronald Allan Kelley Poe a.k.a. Fernando Poe, Jr." would have to be dismissed for called 'indios', denoting a low regard for the inhabitants of the archipelago. Spanish
want of jurisdiction. laws on citizenship became highly codified during the 19th century but their sheer
number made it difficult to point to one comprehensive law. Not all of these citizenship
The Citizenship Issue
laws of Spain however, were made to apply to the Philippine Islands except for those
Now, to the basic issue; it should be helpful to first give a brief historical explicitly extended by Royal Decrees.[14]
background on the concept of citizenship.
Spanish laws on citizenship were traced back to the Novisima
Perhaps, the earliest understanding of citizenship was that given by Aristotle, who, Recopilacion, promulgated in Spain on 16 July 1805 but as to whether the law was
sometime in 384 to 322 B.C., described the "citizen" to refer to a man who shared in extended to the Philippines remained to be the subject of differing views among
the administration of justice and in the holding of an office.[6] Aristotle saw its experts;[15] however, three royal decrees were undisputably made applicable to
significance if only to determine the constituency of the "State," which he described as Spaniards in the Philippines - the Order de la Regencia of 14 August 1841,[16] the Royal
being composed of such persons who would be adequate in number to achieve a self- Decree of 23 August 1868 specifically defining the political status of children born in
sufficient existence.[7] The concept grew to include one who would both govern and be the Philippine Islands,[17] and finally, the Ley Extranjera de Ultramar of 04 July 1870,
governed, for which qualifications like autonomy, judgment and loyalty could be which was expressly made applicable to the Philippines by the Royal Decree of 13 July
expected. Citizenship was seen to deal with rights and entitlements, on the one hand, 1870.[18]
and with concomitant obligations, on the other.[8] In its ideal setting, a citizen was
The Spanish Constitution of 1876 was never extended to the Philippine Islands
active in public life and fundamentally willing to submit his private interests to the
because of the express mandate of its Article 89, according to which the provisions of
general interest of society.
the Ultramar among which this country was included, would be governed by special
The concept of citizenship had undergone changes over the centuries. In the 18th laws.[19]
century, the concept was limited, by and large, to civil citizenship, which established
It was only the Civil Code of Spain, made effective in this jurisdiction on 18
the rights necessary for individual freedom, such as rights to property, personal liberty
December 1889, which came out with the first categorical enumeration of who were
and justice.[9] Its meaning expanded during the 19th century to include political
Spanish citizens. -
citizenship, which encompassed the right to participate in the exercise of political
power.[10] The 20th century saw the next stage of the development of social citizenship,
(a) Persons born in Spanish territory,
which laid emphasis on the right of the citizen to economic well-being and social
security.[11] The idea of citizenship has gained expression in the modern welfare state as
(b) Children of a Spanish father or mother, even if they were born outside of
it so developed in Western Europe. An ongoing and final stage of development, in
Spain,
keeping with the rapidly shrinking global village, might well be the internationalization
of citizenship.[12]
(c) Foreigners who have obtained naturalization papers,

(d) Those who, without such papers, may have become domiciled inhabitants
The Local Setting - from Spanish
of any town of the Monarchy.[20]
Times to the Present
The year 1898 was another turning point in Philippine history. Already in the state
There was no such term as "Philippine citizens" during the Spanish regime but of decline as a superpower, Spain was forced to so cede her sole colony in the East to
"subjects of Spain" or "Spanish subjects."[13] In church records, the natives were an upcoming world power, the United States. An accepted principle of international

8
law dictated that a change in sovereignty, while resulting in an abrogation of all Spanish subjects on the 11th day of April, 1891, and then resided in said Islands, and
political laws then in force, would have no effect on civil laws, which would remain their children born subsequent thereto, shall be deemed and held to be citizens of the
virtually intact. Philippine Islands and as such entitled to the protection of the United States, except
such as shall have elected to preserve their allegiance to the Crown of Spain in
The Treaty of Paris was entered into on 10 December 1898 between Spain and the
accordance with the provisions of the treaty of peace between the United States and
United States.[21] Under Article IX of the treaty, the civil rights and political status of
Spain, signed at Paris, December tenth eighteen hundred and ninety eight."[23]
the native inhabitants of the territories ceded to the United States would be determined
by its Congress -
Under the organic act, a citizen of the Philippines was one who was an inhabitant of the
Philippines, and a Spanish subject on the 11th day of April 1899. The term inhabitant
"Spanish subjects, natives of the Peninsula, residing in the territory over which Spain
was taken to include 1) a native-born inhabitant, 2) an inhabitant who was a native of
by the present treaty relinquishes or cedes her sovereignty may remain in such territory
Peninsular Spain, and 3) an inhabitant who obtained Spanish papers on or before 11
or may remove therefrom, retaining in either event all their rights of property, including
April 1899.[24]
the right to sell or dispose of such property or of its proceeds; and they shall also have
the right to carry on their industry, commerce, and professions, being subject in respect Controversy arose on to the status of children born in the Philippines from 11 April
thereof to such laws as are applicable to foreigners. In case they remain in the territory 1899 to 01 July 1902, during which period no citizenship law was extant in the
they may preserve their allegiance to the Crown of Spain by making, before a court of Philippines. Weight was given to the view, articulated in jurisprudential writing at the
record, within a year from the date of the exchange of ratifications of this treaty, a time, that the common law principle of jus soli, otherwise also known as the principle
declaration of their decision to preserve such allegiance; in default of which declaration of territoriality, operative in the United States and England, governed those born in the
they shall be held to have renounced it and to have adopted the nationality of the Philippine Archipelago within that period.[25] More about this later.
territory in which they reside.
In 23 March 1912, the Congress of the United States made the following
amendment to the Philippine Bill of 1902 -
Thus
"Provided, That the Philippine Legislature is hereby authorized to provide by law for
"The civil rights and political status of the native inhabitants of the territories hereby
the acquisition of Philippine citizenship by those natives of the Philippine Islands who
ceded to the United States shall be determined by the Congress."[22]
do not come within the foregoing provisions, the natives of other insular possession of
the United States, and such other persons residing in the Philippine Islands who would
Upon the ratification of the treaty, and pending legislation by the United States
become citizens of the United States, under the laws of the United States, if residing
Congress on the subject, the native inhabitants of the Philippines ceased to be Spanish
therein."[26]
subjects. Although they did not become American citizens, they, however, also ceased
to be "aliens" under American laws and were thus issued passports describing them to
With the adoption of the Philippine Bill of 1902, the concept of "Philippine
be citizens of the Philippines entitled to the protection of the United States.
citizens" had for the first time crystallized. The word "Filipino" was used by William
The term "citizens of the Philippine Islands" appeared for the first time in the H. Taft, the first Civil Governor General in the Philippines when he initially made
Philippine Bill of 1902, also commonly referred to as the Philippine Organic Act of mention of it in his slogan, "The Philippines for the Filipinos." In 1916, the Philippine
1902, the first comprehensive legislation of the Congress of the United States on the Autonomy Act, also known as the Jones Law restated virtually the provisions of the
Philippines - Philippine Bill of 1902, as so amended by the Act of Congress in 1912 -

".... that all inhabitants of the Philippine Islands continuing to reside therein, who were That all inhabitants of the Philippine Islands who were Spanish subjects on the
9
eleventh day of April, eighteen hundred and ninety-nine, and then resided in said Subsection (4), Article III, of the 1935 Constitution, taken together with existing
Islands, and their children born subsequently thereto, shall be deemed and held to be civil law provisions at the time, which provided that women would automatically lose
citizens of the Philippine Islands, except such as shall have elected to preserve their their Filipino citizenship and acquire that of their foreign husbands, resulted in
allegiance to the Crown of Spain in accordance with the provisions of the treaty of discriminatory situations that effectively incapacitated the women from transmitting
peace between the United States and Spain, signed at Paris December tenth, eighteen their Filipino citizenship to their legitimate children and required illegitimate children
hundred and ninety-eight and except such others as have since become citizens of some of Filipino mothers to still elect Filipino citizenship upon reaching the age of
other country; Provided, That the Philippine Legislature, herein provided for, is hereby majority. Seeking to correct this anomaly, as well as fully cognizant of the newly found
authorized to provide for the acquisition of Philippine citizenship by those natives of status of Filipino women as equals to men, the framers of the 1973 Constitution crafted
the Philippine Islands who do not come within the foregoing provisions, the natives of the provisions of the new Constitution on citizenship to reflect such concerns -
the insular possessions of the United States, and such other persons residing in the
Philippine Islands who are citizens of the United States, or who could become citizens Section 1, Article III, 1973 Constitution - The following are citizens of the Philippines:
of the United States under the laws of the United States, if residing therein."
(1) Those who are citizens of the Philippines at the time of the adoption of this
Under the Jones Law, a native-born inhabitant of the Philippines was deemed to be Constitution.
a citizen of the Philippines as of 11 April 1899 if he was 1) a subject of Spain on 11
April 1899, 2) residing in the Philippines on said date, and, 3) since that date, not a (2) Those whose fathers or mothers are citizens of the Philippines.
citizen of some other country.
(3) Those who elect Philippine citizenship pursuant to the provisions of the
While there was, at one brief time, divergent views on whether or not jus soli was
Constitution of nineteen hundred and thirty-five.
a mode of acquiring citizenship, the 1935 Constitution brought to an end to any such
link with common law, by adopting, once and for all, jus sanguinis or blood
(4) Those who are naturalized in accordance with law.
relationship as being the basis of Filipino citizenship -
For good measure, Section 2 of the same article also further provided that
Section 1, Article III, 1935 Constitution. The following are citizens of the Philippines -
"A female citizen of the Philippines who marries an alien retains her Philippine
(1) Those who are citizens of the Philippine Islands at the time of the adoption of this
citizenship, unless by her act or omission she is deemed, under the law to have
Constitution
renounced her citizenship."
(2) Those born in the Philippines Islands of foreign parents who, before the adoption of
The 1987 Constitution generally adopted the provisions of the 1973 Constitution,
this Constitution, had been elected to public office in the Philippine Islands.
except for subsection (3) thereof that aimed to correct the irregular situation generated
by the questionable proviso in the 1935 Constitution.
(3) Those whose fathers are citizens of the Philippines.
Section I, Article IV, 1987 Constitution now provides:
(4) Those whose mothers are citizens of the Philippines and upon reaching the age of
majority, elect Philippine citizenship. The following are citizens of the Philippines:

(5) Those who are naturalized in accordance with law. (1) Those who are citizens of the Philippines at the time of the adoption of this

10
Constitution. his death on 11 September 1954. The certificate of birth of the father of FPJ, Allan F.
Poe, showed that he was born on 17 May 1915 to an Espaol father, Lorenzo Pou, and a
(2) Those whose fathers or mothers are citizens of the Philippines. mestiza Espaol mother, Marta Reyes. Introduced by petitioner was an uncertified copy
of a supposed certificate of the alleged marriage of Allan F. Poe and Paulita Gomez on
(3) Those born before January 17, 1973 of Filipino mothers, who elect Philippine 05 July 1936. The marriage certificate of Allan F. Poe and Bessie Kelley reflected the
citizenship upon reaching the age of majority; and date of their marriage to be on 16 September 1940. In the same certificate, Allan F. Poe
was stated to be twenty-five years old, unmarried, and a Filipino citizen, and Bessie
(4) Those who are naturalized in accordance with law. Kelley to be twenty-two years old, unmarried, and an American citizen. The birth
certificate of FPJ, would disclose that he was born on 20 August 1939 to Allan F. Poe, a
Filipino, twenty-four years old, married to Bessie Kelly, an American citizen, twenty-
The Case Of FPJ one years old and married.
Considering the reservations made by the parties on the veracity of some of the
Section 2, Article VII, of the 1987 Constitution expresses: entries on the birth certificate of respondent and the marriage certificate of his parents,
the only conclusions that could be drawn with some degree of certainty from the
"No person may be elected President unless he is a natural-born citizen of the documents would be that -
Philippines, a registered voter, able to read and write, at least forty years of age on the
day of the election, and a resident of the Philippines for at least ten years immediately 1. The parents of FPJ were Allan F. Poe and Bessie Kelley;
preceding such election."
2. FPJ was born to them on 20 August 1939;
The term "natural-born citizens," is defined to include "those who are citizens of
the Philippines from birth without having to perform any act to acquire or perfect their 3. Allan F. Poe and Bessie Kelley were married to each other on 16
Philippine citizenship."[27] September, 1940;
The date, month and year of birth of FPJ appeared to be 20 August 1939 during the 4. The father of Allan F. Poe was Lorenzo Poe; and
regime of the 1935 Constitution. Through its history, four modes of acquiring
citizenship - naturalization, jus soli, res judicata and jus sanguinis[28] had been in 5. At the time of his death on 11 September 1954, Lorenzo Poe was 84 years
vogue. Only two, i.e., jus soli and jus sanguinis, could qualify a person to being a old.
natural-born citizen of the Philippines. Jus soli, per Roa vs. Collector of
Customs[29] (1912), did not last long. With the adoption of the 1935 Constitution and Would the above facts be sufficient or insufficient to establish the fact that FPJ is a
the reversal of Roa in Tan Chong vs. Secretary of Labor[30] (1947), jus sanguinis or natural-born Filipino citizen? The marriage certificate of Allan F. Poe and Bessie
blood relationship would now become the primary basis of citizenship by birth. Kelley, the birth certificate of FPJ, and the death certificate of Lorenzo Pou are
Documentary evidence adduced by petitioner would tend to indicate that the documents of public record in the custody of a public officer. The documents have been
earliest established direct ascendant of FPJ was his paternal grandfather Lorenzo Pou, submitted in evidence by both contending parties during the proceedings before the
married to Marta Reyes, the father of Allan F. Poe. While the record of birth of Lorenzo COMELEC.
Pou had not been presented in evidence, his death certificate, however, identified him The birth certificate of FPJ was marked Exhibit "A" for petitioner and Exhibit "3"
to be a Filipino, a resident of San Carlos, Pangasinan, and 84 years old at the time of for respondent. The marriage certificate of Allan F. Poe to Bessie Kelley was submitted
11
as Exhibit "21" for respondent. The death certificate of Lorenzo Pou was submitted by during the crucial period of from 1898 to 1902 considering that there was no existing
respondent as his Exhibit "5." While the last two documents were submitted in record about such fact in the Records Management and Archives Office. Petitioner,
evidence for respondent, the admissibility thereof, particularly in reference to the facts however, likewise failed to show that Lorenzo Pou was at any other place during the
which they purported to show, i.e., the marriage certificate in relation to the date of same period. In his death certificate, the residence of Lorenzo Pou was stated to be San
marriage of Allan F. Poe to Bessie Kelley and the death certificate relative to the death Carlos, Pangasinan. In the absence of any evidence to the contrary, it should be sound
of Lorenzo Pou on 11 September 1954 in San Carlos, Pangasinan, were all admitted by to conclude, or at least to presume, that the place of residence of a person at the time of
petitioner, who had utilized those material statements in his argument. All three his death was also his residence before death. It would be extremely doubtful if the
documents were certified true copies of the originals. Records Management and Archives Office would have had complete records of all
residents of the Philippines from 1898 to 1902.
Section 3, Rule 130, Rules of Court states that -

Original document must be produced; exceptions. - When the subject of inquiry is the Proof of Paternity and Filiation
contents of a document, no evidence shall be admissible other than the original Under Civil Law.
document itself, except in the following cases:

xxxxxxxxx Petitioner submits, in any case, that in establishing filiation (relationship or civil
status of the child to the father [or mother]) or paternity (relationship or civil status of
(d) When the original is a public record in the custody of a public office or is recorded the father to the child) of an illegitimate child, FPJ evidently being an illegitimate son
in a public office. according to petitioner, the mandatory rules under civil law must be used.
Under the Civil Code of Spain, which was in force in the Philippines from 08
Being public documents, the death certificate of Lorenzo Pou, the marriage certificate December 1889 up until the day prior to 30 August 1950 when the Civil Code of the
of Allan F. Poe and Bessie Kelly, and the birth certificate of FPJ, constitute prima Philippines took effect, acknowledgment was required to establish filiation or
facie proof of their contents. Section 44, Rule 130, of the Rules of Court provides: paternity. Acknowledgment was either judicial (compulsory) or voluntary. Judicial or
compulsory acknowledgment was possible only if done during the lifetime of the
Entries in official records. Entries in official records made in the performance of his putative parent; voluntary acknowledgment could only be had in a record of birth, a
duty by a public officer of the Philippines, or by a person in the performance of a duty will, or a public document.[32] Complementary to the new code was Act No. 3753 or the
specially enjoined by law, are prima facieevidence of the facts therein stated. Civil Registry Law expressing in Section 5 thereof, that -

The trustworthiness of public documents and the value given to the entries made In case of an illegitimate child, the birth certificate shall be signed and sworn to
therein could be grounded on 1) the sense of official duty in the preparation of the jointly by the parents of the infant or only by the mother if the father refuses. In the
statement made, 2) the penalty which is usually affixed to a breach of that duty, 3) the latter case, it shall not be permissible to state or reveal in the document the name of the
routine and disinterested origin of most such statements, and 4) the publicity of record father who refuses to acknowledge the child, or to give therein any information by
which makes more likely the prior exposure of such errors as might have occurred.[31] which such father could be identified.
The death certificate of Lorenzo Pou would indicate that he died on 11 September
1954, at the age of 84 years, in San Carlos, Pangasinan. It could thus be assumed that In order that the birth certificate could then be utilized to prove voluntary
Lorenzo Pou was born sometime in the year 1870 when the Philippines was still a acknowledgment of filiation or paternity, the certificate was required to be signed or
colony of Spain. Petitioner would argue that Lorenzo Pou was not in the Philippines sworn to by the father. The failure of such requirement rendered the same useless as
12
being an authoritative document of recognition.[33] In Mendoza vs. Mella,[34] the Court to claim legitimacy which would last during the lifetime of the child, and might pass
ruled - exceptionally to the heirs of the child, an action to claim acknowledgment, however,
could only be brought during the lifetime of the presumed parent.
"Since Rodolfo was born in 1935, after the registry law was enacted, the question here
Amicus Curiae Ruben F. Balane defined, during the oral argument, "authentic
really is whether or not his birth certificate (Exhibit 1), which is merely a certified copy
writing," so as to be an authentic writing for purposes of voluntary recognition, simply
of the registry record, may be relied upon as sufficient proof of his having been
as being a genuine or indubitable writing of the father. The term would include a public
voluntarily recognized. No such reliance, in our judgment, may be placed upon
instrument (one duly acknowledged before a notary public or other competent official)
it. While it contains the names of both parents, there is no showing that they signed the
or a private writing admitted by the father to be his.
original, let alone swore to its contents as required in Section 5 of Act No. 3753. For all
that might have happened, it was not even they or either of them who furnished the data The Family Code has further liberalized the rules; Article 172, Article 173, and
to be entered in the civil register.Petitioners say that in any event the birth certificate is Article 175 provide:
in the nature of a public document wherein voluntary recognition of a natural child may
also be made, according to the same Article 131. True enough, but in such a case, there Art. 172. The filiation of legitimate children is established by any of the following:
must be a clear statement in the document that the parent recognizes the child as his or
her own." (1) The record of birth appearing in the civil register or a final judgment; or

In the birth certificate of respondent FPJ, presented by both parties, nowhere in the (2) An admission of legitimate filiation in a public document or a private handwritten
document was the signature of Allan F. Poe found. There being no will apparently instrument and signed by the parent concerned.
executed, or at least shown to have been executed, by decedent Allan F. Poe, the only
other proof of voluntary recognition remained to be "some other public In the absence of the foregoing evidence, the legitimate filiation shall be proved by:
document." In Pareja vs. Pareja,[35] this Court defined what could constitute such a
document as proof of voluntary acknowledgment: (1) The open and continuous possession of the status of a legitimate child; or

"Under the Spanish Civil Code there are two classes of public documents, (2) Any other means allowed by the Rules of Court and special laws.
those executed by private individuals which must be authenticated by notaries, and
those issued by competent public officials by reason of their office. The public Art. 173. The action to claim legitimacy may be brought by the child during his or her
document pointed out in Article 131 as one of the means by which recognition may be lifetime and shall be transmitted to the heirs should the child die during minority or in a
made belongs to the first class." state of insanity. In these cases, the heirs shall have a period of five years within which
to institute the action.
Let us leave it at that for the moment.
The 1950 Civil Code categorized the acknowledgment or recognition of The action already commenced by the child shall survive notwithstanding the death of
illegitimate children into voluntary, legal or compulsory. Voluntary recognition was either or both of the parties.
required to be expressedly made in a record of birth, a will, a statement before a court
of record or in any authentic writing. Legal acknowledgment took place in favor of full x x x x x x x x x.
blood brothers and sisters of an illegitimate child who was recognized or judicially
declared as natural. Compulsory acknowledgment could be demanded generally in Art. 175. Illegitimate children may establish their illegitimate filiation in the same way
cases when the child had in his favor any evidence to prove filiation. Unlike an action and on the same, evidence as legitimate children.
13
The action must be brought within the same period specified in Article 173, except family rights and duties, or to the status, condition and legal capacity of persons,
when the action is based on the second paragraph of Article 172, in which case the govern Spaniards although they reside in a foreign country; that, in consequence, 'all
action may be brought during the lifetime of the alleged parent. questions of a civil nature, such as those dealing with the validity or nullity of the
matrimonial bond, the domicile of the husband and wife, their support, as between
The provisions of the Family Code are retroactively applied; Article 256 of the them, the separation of their properties, the rules governing property, marital authority,
code reads: division of conjugal property, the classification of their property, legal causes for
divorce, the extent of the latter, the authority to decree it, and, in general, the civil
"Art. 256. This Code shall have retroactive effect insofar as it does not prejudice or effects of marriage and divorce upon the persons and properties of the spouses, are
impair vested or acquired rights in accordance with the Civil Code or other laws. questions that are governed exclusively by the national law of the husband and wife."

Thus, in Vda. de Sy-Quia vs. Court of Appeals,[36] the Court has ruled: The relevance of "citizenship" or "nationality" to Civil Law is best exemplified in
Article 15 of the Civil Code, stating that -
"We hold that whether Jose was a voluntarily recognized natural child should be
decided under Article 278 of the Civil Code of the Philippines. Article 2260 of that "Laws relating to family rights and duties, or to the status, condition and legal capacity
Code provides that 'the voluntary recognition of a natural child shall take place of persons are binding upon citizens of the Philippines, even though living abroad" -
according to this Code, even if the child was born before the effectivity of this body of
laws' or before August 30, 1950. Hence, Article 278 may be given retroactive effect." that explains the need to incorporate in the code a reiteration of the Constitutional
provisions on citizenship. Similarly, citizenship is significant in civil relationships
It should be apparent that the growing trend to liberalize the acknowledgment or found in different parts of the Civil Code, [39] such as on successional rights and family
recognition of illegitimate children is an attempt to break away from the traditional idea relations.[40] In adoption, for instance, an adopted child would be considered the child of
of keeping well apart legitimate and non-legitimate relationships within the family in his adoptive parents and accorded the same rights as their legitimate child but such
favor of the greater interest and welfare of the child. The provisions are intended to legal fiction extended only to define his rights under civil law [41] and not his political
merely govern the private and personal affairs of the family. There is little, if any, to status.
indicate that the legitimate or illegitimate civil status of the individual would also affect
Civil law provisions point to an obvious bias against illegitimacy. This
his political rights or, in general, his relationship to the State. While, indeed, provisions
discriminatory attitude may be traced to the Spanish family and property laws, which,
on "citizenship" could be found in the Civil Code, such provisions must be taken in the
while defining proprietary and successional rights of members of the family, provided
context of private relations, the domain of civil law; particularly -
distinctions in the rights of legitimate and illegitimate children. In the monarchial set-
up of old Spain, the distribution and inheritance of titles and wealth were strictly
"Civil Law is that branch of law which has for its double purpose the organization of
according to bloodlines and the concern to keep these bloodlines uncontaminated by
the family and the regulation of property. It has thus [been] defined as the mass of
foreign blood was paramount.
precepts which determine and regulate the relations of assistance, authority and
obedience among members of a family, and those which exist among members of a These distinctions between legitimacy and illegitimacy were codified in the
society for the protection of private interests."[37] Spanish Civil Code, and the invidious discrimination survived when the Spanish Civil
Code became the primary source of our own Civil Code. Such distinction, however,
In Yaez de Barnuevo vs. Fuster,[38] the Court has held: remains and should remain only in the sphere of civil law and not unduly impede or
impinge on the domain of political law.
"In accordance with Article 9 of the Civil Code of Spain, x x x the laws relating to
The proof of filiation or paternity for purposes of determining his citizenship status
14
should thus be deemed independent from and not inextricably tied up with that 3. Fernando and Bessie Poe had a son by the name of Ronald Allan Poe, more
prescribed for civil law purposes. The Civil Code or Family Code provisions on proof popularly known in the Philippines as `Fernando Poe, Jr., or `FPJ.
of filiation or paternity, although good law, do not have preclusive effects on matters
alien to personal and family relations. The ordinary rules on evidence could well and 4. Ronald Allan Poe `FPJ was born on August 20, 1939 at St. Luke's Hospital,
should govern. For instance, the matter about pedigree is not necessarily precluded Magdalena Street, Manila.
from being applicable by the Civil Code or Family Code provisions.
xxxxxxxxx
Section 39, Rule 130, of the Rules of Court provides -
7. Fernando Poe Sr., and my sister Bessie, met and became engaged while
Act or Declaration about pedigree. The act or declaration of a person deceased, or
they were students at the University of the Philippines in 1936. I was
unable to testify, in respect to the pedigree of another person related to him by birth or
also introduced to Fernando Poe, Sr., by my sister that same year.
marriage, may be received in evidence where it occurred before the controversy, and
the relationship between the two persons is shown by evidence other than such act or
8. Fernando Poe, Sr., and my sister Bessie had their first child in 1938.
declaration. The word `pedigree includes relationship, family genealogy, birth,
marriage, death, the dates when and the places where these facts occurred, and the
9. Fernando Poe, Sr., my sister Bessie and their first three children, Elizabeth,
names of the relatives. It embraces also facts of family history intimately connected
Ronald, Allan and Fernando II, and myself lived together with our
with pedigree.
mother at our family's house on Dakota St. (now Jorge Bocobo St.),
Malate until the liberation of Manila in 1945, except for some months
For the above rule to apply, it would be necessary that (a) the declarant is already
between 1943-1944.
dead or unable to testify, (b) the pedigree of a person must be at issue, (c) the declarant
must be a relative of the person whose pedigree is in question, (d) declaration must be
10. Fernando Poe, Sr., and my sister, Bessie, were blessed with four (4) more
made before the controversy has occurred, and (e) the relationship between the
children after Ronald Allan Poe.
declarant and the person whose pedigree is in question must be shown by evidence
other than such act or declaration.
xxxxxxxxx
Thus, the duly notarized declaration made by Ruby Kelley Mangahas, sister of
Bessie Kelley Poe submitted as Exhibit 20 before the COMELEC, might be accepted to 18. I am executing this Declaration to attest to the fact that my nephew,
prove the acts of Allan F. Poe, recognizing his own paternal relationship with FPJ, Ronald Allan Poe is a natural born Filipino, and that he is the legitimate
i.e, living together with Bessie Kelley and his children (including respondent FPJ) in child of Fernando Poe, Sr.
one house, and as one family -
Done in City of Stockton, California, U.S.A., this 12th day of January 2004.
"I, Ruby Kelley Mangahas, of legal age and sound mind, presently residing in
Stockton, California, U.S.A., after being sworn in accordance with law do hereby Ruby Kelley Mangahas
declare that:
Declarant
1. I am the sister of the late Bessie Kelley Poe.

2. Bessie Kelley Poe was the wife of Fernando Poe, Sr. DNA Testing
15
In case proof of filiation or paternity would be unlikely to satisfactorily establish or of his mother, Bessie Kelley, an American citizen, basing his stand on the ruling of this
would be difficult to obtain, DNA testing, which examines genetic codes obtained from Court in Morano vs. Vivo,[43] citing Chiongbian vs. de Leon[44] and Serra vs. Republic.
[45]
body cells of the illegitimate child and any physical residue of the long dead parent
could be resorted to. A positive match would clear up filiation or paternity. In Tijing vs.
On the above score, the disquisition made by amicus curiae Joaquin G. Bernas, SJ,
Court of Appeals,[42] this Court has acknowledged the strong weight of DNA testing -
is most convincing; he states -
"Parentage will still be resolved using conventional methods unless we adopt the
"We must analyze these cases and ask what the lis mota was in each of them. If the
modern and scientific ways available. Fortunately, we have now the facility and
pronouncement of the Court on jus sanguinis was on the lis mota, the pronouncement
expertise in using DNA test for identification and parentage testing. The University of
would be a decision constituting doctrine under the rule of stare decisis. But if the
the Philippines Natural Science Research Institute (UP-NSRI) DNA Analysis
pronouncement was irrelevant to the lis mota, the pronouncement would not be a
Laboratory has now the capability to conduct DNA typing using short tandem repeat
decision but a mere obiter dictum which did not establish doctrine. I therefore invite the
(STR) analysis. The analysis is based on the fact that the DNA of a child/person has
Court to look closely into these cases.
two (2) copies, one copy from the mother and the other from the father. The DNA from
the mother, the alleged father and the child are analyzed to establish parentage. Of
First, Morano vs. Vivo. The case was not about an illegitimate child of a Filipino
course, being a novel scientific technique, the use of DNA test as evidence is still open
father. It was about a stepson of a Filipino, a stepson who was the child of a Chinese
to challenge. Eventually, as the appropriate case comes, courts should not hesitate to
mother and a Chinese father. The issue was whether the stepson followed the
rule on the admissibility of DNA evidence. For it was said, that courts should apply the
naturalization of the stepfather. Nothing about jus sanguinis there. The stepson did not
results of science when competently obtained in aid of situations presented, since to
have the blood of the naturalized stepfather.
reject said result is to deny progress."
Second, Chiongbian vs. de Leon. This case was not about the illegitimate son of a
Filipino father. It was about a legitimate son of a father who had become Filipino by
Petitioners Argument For
election to public office before the 1935 Constitution pursuant to Article IV, Section
Jurisprudential Conclusiveness
1(2) of the 1935 Constitution. No one was illegitimate here.

Petitioner would have it that even if Allan F. Poe were a Filipino citizen, he could Third, Serra vs. Republic. The case was not about the illegitimate son of a Filipino
not have transmitted his citizenship to respondent FPJ, the latter being an illegitimate father. Serra was an illegitimate child of a Chinese father and a Filipino mother. The
child. According to petitioner, prior to his marriage to Bessie Kelley, Allan F. Poe, on issue was whether one who was already a Filipino because of his mother who still
July 5, 1936, contracted marriage with a certain Paulita Gomez, making his subsequent needed to be naturalized. There is nothing there about invidious jus sanguinis.
marriage to Bessie Kelley bigamous and respondent FPJ an illegitimate child. The
veracity of the supposed certificate of marriage between Allan F. Poe and Paulita Finally, Paa vs. Chan.[46] This is a more complicated case. The case was about the
Gomez could be most doubtful at best. But the documentary evidence introduced by no citizenship of Quintin Chan who was the son of Leoncio Chan. Quintin Chan claimed
less than respondent himself, consisting of a birth certificate of respondent and a that his father, Leoncio, was the illegitimate son of a Chinese father and a Filipino
marriage certificate of his parents showed that FPJ was born on 20 August 1939 to a mother. Quintin therefore argued that he got his citizenship from Leoncio, his
Filipino father and an American mother who were married to each other a year later, or father. But the Supreme Court said that there was no valid proof that Leoncio was in
on 16 September 1940. Birth to unmarried parents would make FPJ an illegitimate fact the son of a Filipina mother. The Court therefore concluded that Leoncio was not
child. Petitioner contended that as an illegitimate child, FPJ so followed the citizenship Filipino. If Leoncio was not Filipino, neither was his son Quintin. Quintin therefore
was not only not a natural-born Filipino but was not even a Filipino.
16
The Court should have stopped there. But instead it followed with an obiter for the illegitimate child of an alien father in line with the assumption that the mother
dictum. The Court said obiter that even if Leoncio, Quintin's father, were Filipino, had custody, would exercise parental authority and had the duty to support her
Quintin would not be Filipino because Quintin was illegitimate. This statement about illegitimate child. It was to help the child, not to prejudice or discriminate against him.
Quintin, based on a contrary to fact assumption, was absolutely unnecessary for the
The fact of the matter perhaps the most significant consideration is that the 1935
case. x x x It was obiter dictum, pure and simple, simply repeating the obiter dictum
Constitution, the fundamental law prevailing on the day, month and year of birth of
in Morano vs. Vivo.
respondent FPJ, can never be more explicit than it is. Providing neither conditions nor
distinctions, the Constitution states that among the citizens of the Philippines are those
xxxxxxxxx
whose fathers are citizens of the Philippines. There utterly is no cogent justification to
prescribe conditions or distinctions where there clearly are none provided.
"Aside from the fact that such a pronouncement would have no textual foundation in
the Constitution, it would also violate the equal protection clause of the Constitution
not once but twice. First, it would make an illegitimate distinction between a legitimate
child and an illegitimate child, and second, it would make an illegitimate distinction In Sum
between the illegitimate child of a Filipino father and the illegitimate child of a Filipino
mother. (1) The Court, in the exercise of its power of judicial review, possesses jurisdiction
over the petition in G. R. No. 161824, filed under Rule 64, in relation to Rule 65, of the
The doctrine on constitutionally allowable distinctions was established long ago by Revised Rules of Civil Procedure. G.R. No. 161824 assails the resolution of the
People vs. Cayat.[47] I would grant that the distinction between legitimate children and COMELEC for alleged grave abuse of discretion in dismissing, for lack of merit, the
illegitimate children rests on real differences. x x x But real differences alone do not petition in SPA No. 04-003 which has prayed for the disqualification of respondent FPJ
justify invidious distinction. Real differences may justify distinction for one purpose from running for the position of President in the 10th May 2004 national elections on
but not for another purpose. the contention that FPJ has committed material representation in his certificate of
candidacy by representing himself to be a natural-born citizen of the Philippines.
x x x What is the relevance of legitimacy or illegitimacy to elective public
service? What possible state interest can there be for disqualifying an illegitimate child (2) The Court must dismiss, for lack of jurisdiction and prematurity, the petitions
from becoming a public officer. It was not the fault of the child that his parents had in G. R. No. 161434 and No. 161634 both having been directly elevated to this Court in
illicit liaison. Why deprive the child of the fullness of political rights for no fault of his the latters capacity as the only tribunal to resolve a presidential and vice-presidential
own? To disqualify an illegitimate child from holding an important public office is to election contest under the Constitution. Evidently, the primary jurisdiction of the Court
punish him for the indiscretion of his parents. There is neither justice nor rationality in can directly be invoked only after, not before, the elections are held.
that. And if there is neither justice nor rationality in the distinction, then the distinction (3) In ascertaining, in G.R. No. 161824, whether grave abuse of discretion has
transgresses the equal protection clause and must be reprobated. been committed by the COMELEC, it is necessary to take on the matter of whether or
not respondent FPJ is a natural-born citizen, which, in turn, depended on whether or not
The other amici curiae, Mr. Justice Vicente Mendoza (a former member of this the father of respondent, Allan F. Poe, would have himself been a Filipino citizen and,
Court), Professor Ruben Balane and Dean Martin Magallona, at bottom, have in the affirmative, whether or not the alleged illegitimacy of respondent prevents him
expressed similar views. The thesis of petitioner, unfortunately hinging solely on from taking after the Filipino citizenship of his putative father. Any conclusion on the
pure obiter dicta, should indeed fail. Filipino citizenship of Lorenzo Pou could only be drawn from the presumption that
Where jurisprudence regarded an illegitimate child as taking after the citizenship having died in 1954 at 84 years old, Lorenzo would have been born sometime in the
of its mother, it did so for the benefit the child. It was to ensure a Filipino nationality year 1870, when the Philippines was under Spanish rule, and that San Carlos,
17
Pangasinan, his place of residence upon his death in 1954, in the absence of any other
evidence, could have well been his place of residence before death, such that Lorenzo
Pou would have benefited from the en masse Filipinization that the Philippine Bill had
effected in 1902. That citizenship (of Lorenzo Pou), if acquired, would thereby extend
to his son, Allan F. Poe, father of respondent FPJ. The 1935 Constitution, during which
regime respondent FPJ has seen first light, confers citizenship to all persons whose
fathers are Filipino citizens regardless of whether such children are legitimate or
illegitimate.
(4) But while the totality of the evidence may not establish conclusively that
respondent FPJ is a natural-born citizen of the Philippines, the evidence on hand still
would preponderate in his favor enough to hold that he cannot be held guilty of having
made a material misrepresentation in his certificate of candidacy in violation of Section
78, in relation to Section 74, of the Omnibus Election Code. Petitioner has utterly failed
to substantiate his case before the Court, notwithstanding the ample opportunity given
to the parties to present their position and evidence, and to prove whether or not there
has been material misrepresentation, which, as so ruled in Romualdez-Marcos vs.
COMELEC,[48] must not only be material, but also deliberate and willful.
WHEREFORE, the Court RESOLVES to DISMISS
1. G. R. No. 161434, entitled "Maria Jeanette C. Tecson and Felix B. Desiderio, Jr.,
Petitioners, versus Commission on Elections, Ronald Allan Kelley Poe
(a.k.a. "Fernando Poe, Jr.,) and Victorino X. Fornier, Respondents," and G. R. No.
161634, entitled "Zoilo Antonio Velez, Petitioner, versus Ronald Allan Kelley
Poe, a.k.a. Fernando Poe, Jr., Respondent," for want of jurisdiction.
2. G. R. No. 161824, entitled Victorino X. Fornier, Petitioner, versus Hon.
Commission on Elections and Ronald Allan Kelley Poe, also known as Fernando Poe,
Jr., for failure to show grave abuse of discretion on the part of respondent Commission
on Elections in dismissing the petition in SPA No. 04-003.
No Costs.
SO ORDERED.

18
AASJS vs. Datumanong renounces his foreign citizenship. Plainly,
G.R. No. 160869, May 11, 2007 from Section 3, Rep. Act No. 9225 stayed clear out of the problem of dual
Sec. 3, RA 9225 stayed clear of the problem of dual allegiance and shifted the burden allegiance and shifted the burden of confronting
of confronting the issue of whether or not the issue of whether or not there is dual allegiance to the concerned foreign
there is dual allegiance to the concerned foreign country. country. What happens to the other citizenship
FACTS: was not made a concern of Rep. Act No. 9225.
Petitioner filed the instant petition against respondent, then Secretary of Justice Simeon For its part, the OSG counters that pursuant to Section 5, Article IV of the 1987
Datumanong, the official tasked to Constitution, dual allegiance shall be dealt with
implement laws governing citizenship in order to prevent the Justice Secretary from by law. Thus, until a law on dual allegiance is enacted by Congress, the Supreme Court
implementing R. A. 9225. Petitioner argues is without any jurisdiction to entertain
that RA 9225 is unconstitutional as it violates Sec. 5, Article VI of the Constitution issues regarding dual allegiance.
which states that dual allegiance of citizens is Moreover, Section 5, Article IV of the Constitution is a declaration of a policy and
inimical to national interest and shall be dealt with by law. The Office of the Solicitor it is not a self-executing provision. The
General (OSG) claims that Section 2 legislature still has to enact the law on dual allegiance. In Sections 2 and 3 of Rep.
merely declares as a state policy that "Philippine citizens who become citizens of Act No. 9225, the framers were not
another country shall be deemed not to have concerned with dual citizenship per se, but with the status of naturalized citizens
lost their Philippine citizenship." The OSG further claims that the oath in Section 3 who maintain their allegiance to their
does not allow dual allegiance since the oath countries of origin even after their naturalization. Congress was given a mandate to
taken by the former Filipino citizen is an effective renunciation and repudiation of his draft a law that would set specific
foreign citizenship. The fact that the parameters of what really constitutes dual allegiance. Until this is done, it would be
applicant taking the oath recognizes and accepts the supreme authority of the premature for the judicial department,
Philippines is an unmistakable and
categorical affirmation of his undivided loyalty to the Republic. including this Court, to rule on issues pertaining to dual allegiance.
Issue:
Whether R.A. 9225 is unconstitutional and whether the court
Held:
R.A. 9225 is constitutional and that the Court has no jurisdiction yet to pass upon the
issue of dual allegiance. The court held
that that the intent of the legislature in drafting Rep. Act No. 9225 is to do away
with the provision in Commonwealth Act
No. 635 which takes away Philippine citizenship from natural-born Filipinos who
become naturalized citizens of other
countries. What Rep. Act No. 9225 does is allow dual citizenship to natural-born
Filipino citizens who have lost Philippine
citizenship by reason of their naturalization as citizens of a foreign country. On its
face, it does not recognize dual allegiance.
By swearing to the supreme authority of the Republic, the person implicitly
19
SEC. 3. Retention of Philippine Citizenship.-Any provision of law to the contrary
EN BANC notwithstanding, natural-born citizens of the Philippines who have lost their Philippine
citizenship by reason of their naturalization as citizens of a foreign country are hereby
[G.R. NO. 160869 : May 11, 2007] deemed to have reacquired Philippine citizenship upon taking the following oath of
allegiance to the Republic:
AASJS (ADVOCATES AND ADHERENTS OF SOCIAL JUSTICE FOR SCHOOL
TEACHERS AND ALLIED WORKERS) MEMBER - HECTOR GUMANGAN "I ___________________________, solemnly swear (or affirm) that I will support and
CALILUNG, Petitioner, v. THE HONORABLE SIMEON DATUMANONG, in his defend the Constitution of the Republic of the Philippines and obey the laws and legal
official capacity as the Secretary of Justice, Respondent. orders promulgated by the duly constituted authorities of the Philippines; and I hereby
declare that I recognize and accept the supreme authority of the Philippines and will
DECISION maintain true faith and allegiance thereto; and that I impose this obligation upon myself
voluntarily without mental reservation or purpose of evasion."
QUISUMBING, J.:
Natural-born citizens of the Philippines who, after the effectivity of this Act, become
This is an original action for prohibition under Rule 65 of the 1997 Revised Rules of citizens of a foreign country shall retain their Philippine citizenship upon taking the
Civil Procedure. aforesaid oath.

Petitioner filed the instant petition against respondent, then Secretary of Justice Simeon SEC. 4. Derivative Citizenship. - The unmarried child, whether legitimate, illegitimate
Datumanong, the official tasked to implement laws governing citizenship.1 Petitioner or adopted, below eighteen (18) years of age, of those who reacquire Philippine
prays that a writ of prohibition be issued to stop respondent from implementing citizenship upon effectivity of this Act shall be deemed citizens of the Philippines.
Republic Act No. 9225, entitled "An Act Making the Citizenship of Philippine Citizens
Who Acquire Foreign Citizenship Permanent, Amending for the Purpose SEC. 5. Civil and Political Rights and Liabilities. - Those who retain or reacquire
Commonwealth Act No. 63, As Amended, and for Other Purposes." Petitioner avers Philippine citizenship under this Act shall enjoy full civil and political rights and be
that Rep. Act No. 9225 is unconstitutional as it violates Section 5, Article IV of the subject to all attendant liabilities and responsibilities under existing laws of the
1987 Constitution that states, "Dual allegiance of citizens is inimical to the national Philippines and the following conditions:
interest and shall be dealt with by law."
(1) Those intending to exercise their right of suffrage must meet the requirements under
Rep. Act No. 9225, signed into law by President Gloria M. Arroyo on August 29, 2003, Section 1, Article V of the Constitution, Republic Act No. 9189, otherwise known as
reads: "The Overseas Absentee Voting Act of 2003" and other existing laws;

SECTION 1. Short Title.-This Act shall be known as the "Citizenship Retention and (2) Those seeking elective public office in the Philippines shall meet the qualifications
Reacquisition Act of 2003." for holding such public office as required by the Constitution and existing laws and, at
the time of the filing of the certificate of candidacy, make a personal and sworn
SEC. 2. Declaration of Policy.-It is hereby declared the policy of the State that all renunciation of any and all foreign citizenship before any public officer authorized to
Philippine citizens who become citizens of another country shall be deemed not to have administer an oath;
lost their Philippine citizenship under the conditions of this Act.
(3) Those appointed to any public office shall subscribe and swear to an oath of
20
allegiance to the Republic of the Philippines and its duly constituted authorities prior to law allows natural-born citizens of the Philippines to regain their Philippine citizenship
their assumption of office: Provided, That they renounce their oath of allegiance to the by simply taking an oath of allegiance without forfeiting their foreign allegiance.2 The
country where they took that oath; Constitution, however, is categorical that dual allegiance is inimical to the national
interest.
(4) Those intending to practice their profession in the Philippines shall apply with the
proper authority for a license or permit to engage in such practice; andcralawlibrary The Office of the Solicitor General (OSG) claims that Section 2 merely declares as a
state policy that "Philippine citizens who become citizens of another country shall be
(5) That right to vote or be elected or appointed to any public office in the Philippines deemed not to have lost their Philippine citizenship." The OSG further claims that the
cannot be exercised by, or extended to, those who: oath in Section 3 does not allow dual allegiance since the oath taken by the former
Filipino citizen is an effective renunciation and repudiation of his foreign citizenship.
(a) are candidates for or are occupying any public office in the country of which they The fact that the applicant taking the oath recognizes and accepts the supreme authority
are naturalized citizens; and/or of the Philippines is an unmistakable and categorical affirmation of his undivided
loyalty to the Republic.3
(b) are in the active service as commissioned or noncommissioned officers in the armed
forces of the country which they are naturalized citizens. In resolving the aforecited issues in this case, resort to the deliberations of Congress is
necessary to determine the intent of the legislative branch in drafting the assailed law.
SEC. 6. Separability Clause. - If any section or provision of this Act is held During the deliberations, the issue of whether Rep. Act No. 9225 would allow dual
unconstitutional or invalid, any other section or provision not affected thereby shall allegiance had in fact been the subject of debate. The record of the legislative
remain valid and effective. deliberations reveals the following:

SEC. 7. Repealing Clause. - All laws, decrees, orders, rules and regulations inconsistent x x x
with the provisions of this Act are hereby repealed or modified accordingly.
Pursuing his point, Rep. Dilangalen noted that under the measure, two situations exist -
SEC. 8. Effectivity Clause. - This Act shall take effect after fifteen (15) days following - the retention of foreign citizenship, and the reacquisition of Philippine citizenship. In
its publication in the Official Gazette or two (2) newspapers of general circulation. this case, he observed that there are two citizenships and therefore, two allegiances. He
pointed out that under the Constitution, dual allegiance is inimical to public interest. He
In this petition for prohibition, the following issues have been raised: (1) Is Rep. Act thereafter asked whether with the creation of dual allegiance by reason of retention of
No. 9225 unconstitutional? (2) Does this Court have jurisdiction to pass upon the issue foreign citizenship and the reacquisition of Philippine citizenship, there will now be a
of dual allegiance?cra lawlibrary violation of the Constitution'

We shall discuss these issues jointly. Rep. Locsin underscored that the measure does not seek to address the constitutional
injunction on dual allegiance as inimical to public interest. He said that the proposed
Petitioner contends that Rep. Act No. 9225 cheapens Philippine citizenship. He avers law aims to facilitate the reacquisition of Philippine citizenship by speedy means.
that Sections 2 and 3 of Rep. Act No. 9225, together, allow dual allegiance and not dual However, he said that in one sense, it addresses the problem of dual citizenship by
citizenship. Petitioner maintains that Section 2 allows all Filipinos, either natural-born requiring the taking of an oath. He explained that the problem of dual citizenship is
or naturalized, who become foreign citizens, to retain their Philippine citizenship transferred from the Philippines to the foreign country because the latest oath that will
without losing their foreign citizenship. Section 3 permits dual allegiance because said be taken by the former Filipino is one of allegiance to the Philippines and not to the
21
United States, as the case may be. He added that this is a matter which the Philippine left it at this stage, he explained. In the present measure, he clarified, a person is
government will have no concern and competence over. required to take an oath and the last he utters is one of allegiance to the country. He
then said that the problem of dual allegiance is no longer the problem of the Philippines
Rep. Dilangalen asked why this will no longer be the country's concern, when dual but of the other foreign country.4 (Emphasis supplied.)
allegiance is involved.
From the above excerpts of the legislative record, it is clear that the intent of the
Rep. Locsin clarified that this was precisely his objection to the original version of the legislature in drafting Rep. Act No. 9225 is to do away with the provision in
bill, which did not require an oath of allegiance. Since the measure now requires this Commonwealth Act No. 635 which takes away Philippine citizenship from natural-born
oath, the problem of dual allegiance is transferred from the Philippines to the foreign Filipinos who become naturalized citizens of other countries. What Rep. Act No. 9225
country concerned, he explained. does is allow dual citizenship to natural-born Filipino citizens who have lost Philippine
citizenship by reason of their naturalization as citizens of a foreign country. On its face,
x x x it does not recognize dual allegiance. By swearing to the supreme authority of the
Republic, the person implicitly renounces his foreign citizenship. Plainly, from Section
Rep. Dilangalen asked whether in the particular case, the person did not denounce his 3, Rep. Act No. 9225 stayed clear out of the problem of dual allegiance and shifted the
foreign citizenship and therefore still owes allegiance to the foreign government, and at burden of confronting the issue of whether or not there is dual allegiance to the
the same time, owes his allegiance to the Philippine government, such that there is now concerned foreign country. What happens to the other citizenship was not made a
a case of dual citizenship and dual allegiance. concern of Rep. Act No. 9225.

Rep. Locsin clarified that by swearing to the supreme authority of the Republic, the Petitioner likewise advances the proposition that although Congress has not yet passed
person implicitly renounces his foreign citizenship. However, he said that this is not a any law on the matter of dual allegiance, such absence of a law should not be
matter that he wishes to address in Congress because he is not a member of a foreign justification why this Court could not rule on the issue. He further contends that while
parliament but a Member of the House. it is true that there is no enabling law yet on dual allegiance, the Supreme Court,
through Mercado v. Manzano,6 already had drawn up the guidelines on how to
x x x distinguish dual allegiance from dual citizenship.7

Rep. Locsin replied that it is imperative that those who have dual allegiance contrary to For its part, the OSG counters that pursuant to Section 5, Article IV of the 1987
national interest should be dealt with by law. However, he said that the dual allegiance Constitution, dual allegiance shall be dealt with by law. Thus, until a law on dual
problem is not addressed in the bill. He then cited the Declaration of Policy in the bill allegiance is enacted by Congress, the Supreme Court is without any jurisdiction to
which states that "It is hereby declared the policy of the State that all citizens who entertain issues regarding dual allegiance.8
become citizens of another country shall be deemed not to have lost their Philippine
citizenship under the conditions of this Act." He stressed that what the bill does is To begin with, Section 5, Article IV of the Constitution is a declaration of a policy and
recognize Philippine citizenship but says nothing about the other citizenship. it is not a self-executing provision. The legislature still has to enact the law on dual
allegiance. In Sections 2 and 3 of Rep. Act No. 9225, the framers were not concerned
Rep. Locsin further pointed out that the problem of dual allegiance is created wherein a with dual citizenship per se, but with the status of naturalized citizens who maintain
natural-born citizen of the Philippines takes an oath of allegiance to another country their allegiance to their countries of origin even after their naturalization.9 Congress
and in that oath says that he abjures and absolutely renounces all allegiance to his was given a mandate to draft a law that would set specific parameters of what really
country of origin and swears allegiance to that foreign country. The original Bill had constitutes dual allegiance.10 Until this is done, it would be premature for the judicial
22
department, including this Court, to rule on issues pertaining to dual allegiance.

Neither can we subscribe to the proposition of petitioner that a law is not needed since
the case of Mercado had already set the guidelines for determining dual allegiance.
Petitioner misreads Mercado. That case did not set the parameters of what constitutes
dual allegiance but merely made a distinction between dual allegiance and dual
citizenship.

Moreover, in Estrada v. Sandiganbayan,11 we said that the courts must assume that the
legislature is ever conscious of the borders and edges of its plenary powers, and passed
laws with full knowledge of the facts and for the purpose of promoting what is right
and advancing the welfare of the majority. Hence, in determining whether the acts of
the legislature are in tune with the fundamental law, we must proceed with judicial
restraint and act with caution and forbearance.12 The doctrine of separation of powers
demands no less. We cannot arrogate the duty of setting the parameters of what
constitutes dual allegiance when the Constitution itself has clearly delegated the duty of
determining what acts constitute dual allegiance for study and legislation by Congress.

WHEREFORE, the petition is hereby DISMISSED for lack of merit.

23
As an incident only of the adjudication of the right of the parties to a controversy,
Republic v. Maddela the court may pass upon, and make a
pronouncement relative to, their status. Otherwise, such a pronouncement is beyond
27 SCRA 702 judicial power. Thus, for instance, no
This is a petition to have the petitioners Miguela Tan Suat, and Chan Po Lan, all action or proceeding may be instituted for a declaration to the effect that plaintiff or
Chinese Nationals, to be declared a Filipino petitioner is married, or single, or a
citizens. legitimate child, although a finding thereon may be made as a necessary premise to
That sometime in the year 1937 Miguela was legally married to Sy Ing Seng, a Filipino justify a given relief available only to one
citizen; likewise, in the year 1961, Chan enjoying said status. At times, the law permits the acquisition of a given status, such as
Po Lan was legally married to Cu Bon Piao, a Filipino citizen; and that the petitioners naturalization by judicial decree. But
have all the qualifications and none of the there is no similar legislation authorizing the institution of a judicial proceeding to
disqualifications to become Filipino citizens. declare that a given person is a Filipino
The court inquired from Fiscal Veluz, who represents the Solicitor General, if he has
any opposition to the petition to which the Citizen. (Tan v. Republic, L-14159, April 18, 1960).
Fiscal answered that he has no opposition. The Court had it announced to the public if
there is any opposition to the petitions of
both to be declared a Filipino citizen and nobody in the crowded courtroom registered
his opposition.
As such, the petitioners Miguela Tan Suat and Chan Po Lan were declared a Filipino
citizens by marriage and the Commissioner
of Immigration is hereby ordered to cancel the necessary alien certificate of registration
and immigrant certificate of residence of
the petitioner and to issue the corresponding identification card.
The Solicitor General filed the instant petitions instead, including the Commissioner of
Immigration as co-petitioner in view of
the fact that the dispositive parts of the decisions of the lower court are addressed to
him for compliance.
Issue:
Whether or not a person claiming to be a citizen may get a judicial declaration of
citizenship.
Held:
Under Philippine laws, there can be no action or proceeding for the judicial
declaration of the citizenship of an individual.
Courts of justice exist for the settlement of justiciable controversies, which imply a
given right, legally demandable and
enforceable, an act or omission violative of said right, and a remedy, granted or
sanctioned by law, for said breach of right.
24
married to Sy Ing Seng, a Filipino citizen; and that the petitioner has all the
G.R. No. L-21664 March 28, 1969 qualifications and none of the disqualifications to become a Filipino citizen.
After the submission of the evidence for the petitioner, the court inquired from
REPUBLIC OF THE PHILIPPINES and THE COMMISSIONER OF Fiscal Veluz if he has any opposition to the petition to which the Fiscal
IMMIGRATION, petitioners, answered that he has no opposition, neither has he any evidence to warrant
vs. opposition. The Court had it announced to the public if there is any opposition
HON. MANOLO L. MADDELA, as Judge of the Court of First Instance of Quezon, to the petition of Miguela Tan Suat to be declared a Filipino citizen and nobody
Branch II, and MIGUELA TAN SUAT, respondents. in the crowded courtroom registered his opposition.

----------------------------- IN VIEW OF ALL THE FOREGOING, petitioner Miguela Tan Suat is hereby
declared a Filipino citizen by marriage and the Commissioner of Immigration is
G.R. No. L-21665 March 28, 1969 hereby ordered to cancel the necessary alien certificate of registration and
immigrant certificate of residence of the petitioner and to issue the
REPUBLIC OF THE PHILIPPINES and THE COMMISSIONER OF corresponding identification card.lâwphi1.ñet
IMMIGRATION, petitioners,
vs. On the same day the same court rendered another similarly worded, decision in its
HON. MANOLO L. MADDELA, as Judge of the Court of First Instance of Quezon, special Proceeding No. 4013, this time in favor of Chan Po Lan. This second decision
Branch II and CHAN PO LAN, respondents. reads:

First Assistant Solicitor General Esmeraldo Umali and Solicitor Bernardo P. Pardo for This is a petition to have the petitioner Chan Po Lan, a Chinese National, to be
petitioners. declared a Filipino citizen. The Solicitor General has been represented by
De Mesa and De Mesa for respondents. Assistant Fiscal Jose Veluz. During the trial it has been established to the
satisfaction of the Court that sometime in the year 1961, petitioner was legally
MAKALINTAL, J.: married to Cu Bon Piao, a Filipino citizen; and the petitioner has all the
qualifications and more of the disqualifications to become a Filipino citizen.
These are actually two (2) separate petitions for certiorari and prohibition with After the submission of the evidence for the petitioner, the court inquired from
preliminary injunction but are decided jointly because the issues presented proceed Fiscal Veluz if he has any opposition to the petition to which the Fiscal
from the same factual background. answered that he has no opposition, neither has he any evidence to warrant any
opposition. The Court had it announced to the public if there is any opposition
The pertinent facts are not disputed. On April 29, 1963 the Court of First Instance of to the petition of Chan Po Lan to be declared a Filipino citizen and nobody in
Quezon (Branch 11), Hon. Manolo L. Maddela presiding, rendered a decision in its the crowded courtroom registered his position.
Special Proceeding No. 4012, which is hereunder quoted in its entirety:
IN VIEW OF THE FOREGOING, petitioner Chan Po Lan is hereby declared a
This is a petition to have the petitioner Miguela Tan Suat, a Chinese National, Filipino citizen by marriage and the Commissioner of the Bureau of
to be declared a Filipino citizen. The Solicitor General has been represented by Immigration is hereby ordered to cancel the necessary alien certificate of
Assistant Fiscal Jose Veluz. During the trial it has been established to the registration and immigrant certificate of residence of the petitioner and to issue
satisfaction of the Court that sometime in the year 1937 petitioner was legally the corresponding identification card.
25
On July 1, 1963 the Solicitor General 1 filed separate notices of appeal from said Before these cases were submitted for decision, the Solicitor General filed a motion,
decisions, at the same time requesting an extension of ten (10) days within which to file dated February 14, 1964, to cite the Clerk of Court of the Court of First Instance of
the corresponding records on appeal. However, because of the unexplained failure of Quezon for contempt by reason of his failure to forward the records of these cases to
the Clerk of Court of the Court of First Instance of Quezon to forward the records this Court despite our resolution to that effect. It appears, however, that after the said
immediately despite repeated requests therefor by the Solicitor General, the latter, resolution was issued the Clerk did send those records and the same were received here
unable to prepare the records on appeal, filed the instant petitions instead, including the on January 24, 1964. The question of contempt has therefore become moot.
Commissioner of Immigration as co-petitioner in view of the fact that the dispositive
parts of the decisions of the lower court are addressed to him for compliance. WHEREFORE, the writs prayed for are hereby granted; the questioned decisions are
set aside and the writs of preliminary injunction previously issued are made permanent.
On August 10, 1963 we issued in each case a writ of preliminary injunction to restrain Costs against private respondents.
execution and enforcement of the judgment. Thereafter these two cases were submitted
for decision without any answer from the respondents.

Private respondents' identical prayer in the lower court was for a declaration of their
Filipino citizenship and for an order to compel the Commissioner of Immigration to
cancel their respective alien certificates of registration on the ground that they had
married Filipino husbands. In granting the said prayer the lower court was clearly in
error. At that time jurisprudence had already set the question at rest: no person claiming
to be a citizen may get a judicial declaration of citizenship.

Under our laws, there can be no action or proceeding for the judicial
declaration of the citizenship of an individual. Courts of justice exist for the
settlement of justiciable controversies, which imply a given right, legally
demandable and enforceable, an act or omission violative of said right, and a
remedy, granted or sanctioned by law, for said breach of right. As an incident
only of the adjudication of the right of the parties to a controversy, the court
may pass upon, and make a pronouncement relative to, their status. Otherwise,
such a pronouncement is beyond judicial power. Thus, for instance, no action or
proceeding may be instituted for a declaration to the effect that plaintiff or
petitioner is married, or single, or a legitimate child, although a finding thereon
may be made as a necessary premise to justify a given relief available only to
one enjoying said status. At times, the law permits the acquisition of a given
status, such as naturalization by judicial decree. But there is no similar
legislation authorizing the institution of a judicial proceeding to declare that a
given person is part of our citizenry. (Tan v. Republic, L-14159, April 18,
1960).2

26
Moy Ya Lim Yao v. Commissioner Edilberto Aguinaldo Lim, a Filipino citizen of 25 January 1962.
41 SCRA 292 Under Section 15 of Commonwealth Act 473, an alien woman marrying a Filipino, native
Facts: born or naturalized, becomes
On 8 February 1961, Lau Yuen Yeung applied for a passport visa to enter the Philippines as ipso facto a Filipina provided she is not disqualified to be a citizen of the Philippines
a non-immigrant. In the under Section 4 of the same law.
interrogation made in connection with her application for a temporary visitor's visa to enter the Likewise, an alien woman married to an alien who is subsequently naturalized here
Philippines, she stated that she follows the Philippine citizenship of
was a Chinese residing at Kowloon, Hongkong, and that she desired to take a pleasure trip her husband the moment he takes his oath as Filipino citizen, provided that she does not
to the Philippines to visit her suffer from any of the
great-grand-uncle Lau Ching Ping for a period of one month. She was permitted to come disqualifications under said Section 4. Whether the alien woman requires to undergo the
into the Philippines on 13 March naturalization proceedings, Section 15
1961, and was permitted to stay for a period of one month which would expire on 13 April is a parallel provision to Section 16. Thus, if the widow of an applicant for naturalization as
1961. On the date of her arrival, Filipino, who dies during the
Asher Y, Cheng filed a bond in the amount of P1,000.00 to undertake, among others, that said proceedings, is not required to go through a naturalization proceedings, in order to be
Lau Yuen Yeung would actually considered as a Filipino citizen hereof, it
depart from the Philippines on or before the expiration of her authorized period of stay in this should follow that the wife of a living Filipino cannot be denied the same privilege. This is
country or within the period as in plain common sense and there is
his discretion the Commissioner of Immigration or his authorized representative might properly absolutely no evidence that the Legislature intended to treat them differently. As the laws of
allow. After repeated our country, both substantive and
extensions, Lau Yuen Yeung was allowed to stay in the Philippines up to 13 February procedural, stand today, there is no such procedure (a substitute for naturalization
1962. On 25 January 1962, she proceeding to enable the alien wife of a
contracted marriage with Moy Ya Lim Yao alias Edilberto Aguinaldo Lim an alleged Philippine citizen to have the matter of her own citizenship settled and established so that
Filipino citizen. Because of the she may not have to be called
contemplated action of the Commissioner of Immigration to confiscate her bond and order her upon to prove it everytime she has to perform an act or enter into a transaction or
arrest and immediate deportation, business or exercise a right reserved
after the expiration of her authorized stay, she brought an action for injunction with preliminary only to Filipinos), but such is no proof that the citizenship is not vested as of the date of
injunction. At the hearing which marriage or the husband's acquisition of
took place one and a half years after her arrival, it was admitted that Lau Yuen Yeung could not citizenship, as the case may be, for the truth is that the situation obtains even as to native-born
write either English or Tagalog. Filipinos. Everytime the
Except for a few words, she could not speak either English or Tagalog. She could not name any citizenship of a person is material or indispensible in a judicial or administrative case,
Filipino neighbor, with a Filipino Whatever the corresponding court or
name except one, Rosa. She did not know the names of her brothers-in-law, or sisters-in-law. administrative authority decides therein as to such citizenship is generally not considered as res
The Court of First Instance of adjudicata, hence it has to be
Manila (Civil Case 49705) denied the prayer for preliminary injunction. Moya Lim Yao and threshed out again and again as the occasion may demand.
Lau Yuen Yeung appealed. Nota bene:
Issue: There are two laws, which govern the Loss of Philippine citizenship. These are Commonwealth
Whether Lau Yuen Yeung ipso facto became a Filipino citizen upon her marriage to a Filipino Act No. 63 and Commonwealth
citizen. Act No. 473. The former applies to both natural-born and naturalized citizens and the latter
Held: applies only to naturalized citizens.
Lau Yuen Yeung, was declared to have become a Filipino citizen from and by virtue of her As stated in Commonwealth Act. No. 63,
marriage to Moy Ya Lim Yao al as A Filipino citizen may lose his citizenship in any of the following ways and/or events:
27
1. By naturalization in a foreign country; Commonwealth Act No. 473, Section 18 of said law provides that:
2. By express renunciation of citizenship; A naturalization certificate may be cancelled by a competent judge on any of the following
3. By subscribing to an oath of allegiance to support the constitution or laws of a foreign grounds:
country upon attaining twenty-one 1. If it is shown that said naturalization certificate was obtained fraudulently or illegally;
years of age or more: Provided, however, That a Filipino may not divest himself of Philippine 2. If the person naturalized shall, within five years next following the issuance of said
citizenship in any manner naturalization certificated, return
while the Republic of the Philippines is at war with any country; to his native country or to some foreign country and establish his permanent residence
4. By rendering services to, or accepting commission in, the armed forces of a foreign there: Provided, That the
country: Provided, That the rendering fact of the person naturalized remaining for more than one year in his native country or the
of service to, or the acceptance of such commission in, the armed forces of a foreign country, country of his former
and the taking of an oath of nationality, or two years in any other foreign country, shall be considered prima facie evidence
allegiance incident thereto, with the consent of the Republic of the Philippines, shall not divest of his intention of taking
a Filipino of his up his permanent residence in the same;
Philippine citizenship if either of the following circumstances is present: 3. If the petition was made on an invalid declaration of intention
a. The Republic of the Philippines has a defensive and/or offensive pact of alliance with the 4. If it is shown that the minor children of the person naturalized failed to graduate from a
said foreign public high school
country; or recognized by the Office of Private Education of the Philippines, where Philippine history,
b. The said foreign country maintains armed forces on Philippine territory with the consent government and civics
of the Republic are taught as part of the school curriculum, through the fault of their parents either by
of the Philippines: Provided, That the Filipino citizen concerned, at the time of rendering said neglecting to support them or by
service, or transferring them to another school or schools. A certified copy of the decree of naturalization
acceptance of said commission, and taking the oath of allegiance incident thereto, states that he certificate shall be
does so only forwarded by the Clerk of Court of the Department of Interior and the Bureau of Justice.
in connection with his service to said foreign country: And provided, finally, That any Filipino 5. If it is shown that the naturalized citizen has allowed himself to be used as a dummy in
citizen who is violation of the
rendering service to, or is commissioned in, the armed forces of a foreign country under any of constitutional provisions requiring Philippine citizenship as a requisite for the exercise, use
the or enjoyment of a right,
circumstances mentioned in paragraph (a) or (b), shall not be permitted to participate nor vote franchise or privilege.
in any election The main reason why a decision in a naturalization proceeding is not res judicata is
of the Republic of the Philippines during the period of his service to, or commission in, the because such is not a judicial
armed forces of adversarial proceeding. Similarly, estoppel or laches cannot apply to the government in action
said foreign country. Upon his discharge from the service of the said foreign country, he shall for the cancellation of a
be certificate of naturalization, since it is a known principle that the government is never estopped
automatically entitled to the full enjoyment of his civil and political rights as a Filipino citizen; by the mistakes on the part of its
5. By cancellation of the of the certificates of naturalization; agents.
6. By having been declared by competent authority, a deserter of the Philippine armed forces However, according to the Constitution, marriage to an alien would not automatically
in time of war, unless divest a person of his citizenship,
subsequently, a plenary pardon or amnesty has been granted; and unless he or she performs certain acts or omission which would result to the loss of his or
7. In the case of a woman, upon her marriage to a foreigner if, by virtue of the laws in her citizenship. This provision,
force in her husband's country, she however, is not retroactive; thus, does not repatriate those who lost their Philippine citizenship
acquires his nationality. by marriage under the 1935 and
28
1973 Constitutions.

29
which would expire on April 13, 1961. On the date of her arrival, Asher Y, Cheng filed
MOY YA LIM YAO alias EDILBERTO AGUINALDO LIM and LAU YUEN a bond in the amount of P1,000.00 to undertake, among others that said Lau Yuen
YEUNG, Petitioners-Appellants, v. THE COMMISSIONER OF Yeung would actually depart from the Philippines on or before the expiration of her
IMMIGRATION, respondent-appellee. authorized period of stay in this country or within the period as in his discretion the
Commissioner of Immigration or his authorized representative might properly allow.
Aruego, Mamaril & Associates for petitioners-appellants. After repeated extensions, petitioner Lau Yuen Yeung was allowed to stay in the
Philippines up to February 13, 1962 (Exhibit "4"). On January 25, 1962, she contracted
Office of the Solicitor General Arturo A. Alafriz, Assistant Solicitor General Frine' C. marriage with Moy Ya Lim Yao alias Edilberto Aguinaldo Lim an alleged Filipino
Zaballero and Solicitor Sumilang V. Bernardo for respondent-appellee. citizen. Because of the contemplated action of respondent to confiscate her bond and
order her arrest and immediate deportation, after the expiration of her authorized stay,
BARREDO, J.: she brought this action for injunction with preliminary injunction. At the hearing which
took place one and a half years after her arrival, it was admitted that petitioner Lau
Appeal from the following decision of the Court of First Instance of Manila in its Civil Yuen Yeung could not write either English or Tagalog. Except for a few words, she
Case No. 49705 entitled Moy Ya Lim Yao, etc., et al. vs. The Commissioner of could not speak either English or Tagalog. She could not name any Filipino neighbor,
Immigration which, brief as it is, sufficiently depicts the factual setting of and the with a Filipino name except one, Rosa. She did not know the names of her brothers-in-
fundamental issues involved in this case thus: law, or sisters-in-law.

In the instant case, petitioners seek the issuance of a writ of injunction against the Under the facts unfolded above, the Court is of the considered opinion, and so holds,
Commissioner of Immigration, "restraining the latter and/or his authorized that the instant petition for injunction cannot be sustained for the same reason as set
representative from ordering plaintiff Lau Yuen Yeung to leave the Philippines and forth in the Order of this Court, dated March 19, 1962, the pertinent portions of which
causing her arrest and deportation and the confiscation of her bond, upon her failure to read:chanrobles virtual law library
do so."chanrobles virtual law library
First, Section 15 of the Revised Naturalization Law provides:
The prayer for preliminary injunction embodied in the complaint, having been denied,
the case was heard on the merits and the parties submitted their respective Effect of the naturalization on wife and children. - Any woman who is now or may
evidence.chanroblesvirtualawlibrarychanrobles virtual law library hereafter be married to a citizen of the Philippines, and who might herself be lawfully
naturalized shall be deemed a citizen of the Philippines.
The facts of the case, as substantially and correctly stated by the Solicitor General are
these: The above-quoted provision is clear and its import unequivocal and hence it should be
held to mean what it plainly and explicitly expresses in unmistakable terms. The clause
On February 8, 1961, Lau Yuen Yeung applied for a passport visa to enter the "who might herself be lawfully naturalized" incontestably implies that an alien woman
Philippines as a non-immigrant. In the interrogation made in connection with her may be deemed a citizen of the Philippines by virtue of her marriage to a Filipino
application for a temporary visitor's visa to enter the Philippines, she stated that she citizen only if she possesses all the qualifications and none of the disqualifications
was a Chinese residing at Kowloon, Hongkong, and that she desired to take a pleasure specified in the law, because these are the explicit requisites provided by law for an
trip to the Philippines to visit her great (grand) uncle Lau Ching Ping for a period of alien to be naturalized. (Lee Suan Ay, Alberto Tan and Lee Chiao vs. Emilio Galang,
one month (Exhibits "l," "1-a," and "2"). She was permitted to come into the etc., G. R. No. L-11855). However, from the allegation of paragraph 3 of the complaint,
Philippines on March 13, 1961, and was permitted to stay for a period of one month to wit:
30
3. That plaintiff Lau Yuen Yeung, Chinese by birth, who might herself be lawfully decided cases of the Supreme Court on the point mentioned above, but also on the very
naturalized as a Filipino citizen (not being disqualified to become such by provisions of Section 9, sub-paragraph (g) of the Philippine Immigration Act of 1940
naturalization), is a Filipino citizen by virtue of her marriage on January 25, 1962 to which reads:
plaintiff MOY YA LIM YAO alias EDILBERTO AGUINALDO LIM, under the
Naturalization Laws of the Philippines. An alien who is admitted as a non-immigrant cannot remain in the Philippines
permanently. To obtain permanent admission, a non-immigrant alien must depart
it can be deduced beyond debate that petitioner Lau Yuen Yeung while claiming not to voluntarily to some foreign country and procure from the appropriate Philippine Consul
be disqualified, does not and cannot allege that she possesses all the qualifications to be the proper visa and thereafter undergo examination by the Officers of the Bureau of
naturalized, naturally because, having been admitted as a temporary visitor only on Immigration at a Philippine port of entry for determination of his admissibility in
March 13, 1961, it is obvious at once that she lacks at least, the requisite length of accordance with the requirements of this Act. (This paragraph is added by Republic Act
residence in the Philippines (Revised Naturalization Law, Sec. 2, Case No. 2, Sec. 3, 503). (Sec. 9, subparagraph (g) of the Philippine Immigration Act of 1940).
Case No. 3).
And fourth, respondent Commissioner of Immigration is charged with the
Were if the intention of the law that the alien woman, to be deemed a citizen of the administration of all laws relating to immigration (Sec. 3, Com. Act No. 613) and in the
Philippines by virtue of marriage to a Filipino citizen, need only be not disqualified performance of his duties in relation to alien immigrants, the law gives the
under the Naturalization Law, it would have been worded "and who herself is not Commissioner of Immigration a wide discretion, a quasi-judicial function in
disqualified to become a citizen of the Philippines." determining cases presented to him (Pedro Uy So vs. Commissioner of Immigration
CA-G. R. No. 23336-R, Dec. 15, 1960), so that his decision thereon may not be
Second, Lau Yuen Yeung, a temporary Chinese woman visitor, whose authorized stay disturbed unless he acted with abuse of discretion or in excess of his
in the Philippines, after repeated extensions thereof, was to expire last February 28, jurisdiction.chanroblesvirtualawlibrarychanrobles virtual law library
1962, having married her co-plaintiff only on January 25, 1962, or just a little over one
month before the expiry date of her stay, it is evident that said marriage was effected It may also be not amiss to state that wife Lau Yuen Yeung, while she barely and
merely for convenience to defeat or avoid her then impending compulsory departure, insufficiently talk in broken Tagalog and English, she admitted that she cannot write
not to say deportation. This cannot be permitted.chanroblesvirtualawlibrarychanrobles either language.
virtual law library
The only matter of fact not clearly passed upon by His Honor which could have some
Third, as the Solicitor General has well stated: bearing in the resolution of this appeal is the allegation in the brief of petitioners-
appellants, not denied in the governments brief, that "in the hearing ..., it was shown
5. That petitioner Lau Yuen Yeung, having been admitted as a temporary alien visitor thru the testimony of the plaintiff Lau Yuen Yeung that she does not possess any of the
on the strength of a deliberate and voluntary representation that she will enter and stay disqualifications for naturalization." Of course, as an additional somehow relevant
only for a period of one month and thereby secured a visa, cannot go back on her factual matter, it is also emphasized by said appellants that during the hearing in the
representation to stay permanently without first departing from the Philippines as she lower court, held almost ten months after the alleged marriage of petitioners, "Lau
had promised. (Chung Tiao Bing, et al. vs. Commissioner of Immigration, G. R. No. L- Yuen Yeung was already carrying in her womb for seven months a child by her
9966, September 29, 1956; Ong Se Lun vs. Board of Commissioners, G. R. No. L- husband."chanrobles virtual law library
6017, September 16, 1954; Sec. 9, last par., Phil. Immigration Law).
Appellants have assigned six errors allegedly committed by the court a quo,
The aforequoted argument of the Solicitor General is well buttressed not only by the thus:chanrobles virtual law library
31
I THE LOWER COURT ERRED IN DISMISSING PLAINTIFFS-APPELLANTS'
COMPLAINT AND IN REFUSING TO PERMANENTLY ENJOIN THE
THE LOWER COURT ERRED IN HOLDING THAT THE CLAUSE "WHO MIGHT COMMISSIONER FROM ORDERING PLAINTIFF LAU YUEN YEUNG TO
HERSELF BE LAWFULLY NATURALIZED" (OF SECTION 15, REVISED LEAVE THE PHILIPPINES AS A TEMPORARY VISITOR WHICH SHE IS NOT.
NATURALIZATION LAW) INCONTESTABLY IMPLIES THAT AN ALIEN
WOMAN MAY BE DEEMED A CITIZEN OF THE PHILIPPINES BY VIRTUE OF VI
HER MARRIAGE TO A FILIPINO CITIZEN, ONLY IF SHE POSSESSES ALL THE
QUALIFICATIONS AND NONE OF THE DISQUALIFICATIONS SPECIFIED IN THE LOWER COURT ERRED IN REFUSING TO GRANT PLAINTIFFS-
THE LAW. APPELLANTS' MOTION FOR PRELIMINARY INJUNCTION EMBODIED IN
THEIR COMPLAINT, IN AN ORDER DATED MARCH 19, 1962. (PAGES 36-41,
II RECORD ON APPEAL) .

THE LOWER COURT ERRED IN HOLDING THAT A WOMAN FOREIGNER We need not discuss these assigned errors separately. In effect, the above decision
WHO DOES NOT POSSESS ANY OF THE DISQUALIFICATIONS FOR upheld the two main grounds of objection of the Solicitor General to the petition in the
CITIZENSHIP AND WHO MARRIED A FILIPINO CITIZEN IS STILL court below, viz:
CONSIDERED AN ALIEN EVEN AFTER SUCH MARRIAGE AS TO FALL
WITHIN THE REQUIREMENT OF SECTION 9, SUB-PARAGRAPH (9) OF THE That petitioner Lau Yuen Yeung, having been admitted as a temporary alien visitor on
PHILIPPINE IMMIGRATION ACT OF 1940. the strength of a deliberate and voluntary representation that she will enter and stay
only for a period of one month and thereby secured a visa, cannot go back on her
III representation to stay permanently without first departing from the Philippines as she
had promised. (Chung Tiao Bing, et al. vs. Commissioner of Immigration, G.R. No. L-
THE COURT ERRED IN CONCLUDING THAT LAU YUEN YEUNG'S 9966, September 29, 1956; Ong Se Lun vs. Board of Commissioners, G.R. No. L-6017,
MARRIAGE TO A FILIPINO CITIZEN WAS ONLY FOR CONVENIENCE, Sept. 16, 1954, Sec. 9, last par. Phil. Immigration Law);chanrobles virtual law library
MERELY BECAUSE THE SAME WAS CELEBRATED JUST OVER A MONTH
BEFORE THE EXPIRY DATE OF HER AUTHORIZED STAY. That the mere marriage of a Filipino citizen to an alien does not automatically confer
on the latter Philippine citizenship. The alien wife must possess all the qualifications
IV required by law to become a Filipino citizen by naturalization and none of the
disqualifications. (Lee Suan Ay, Alberto Tan and Lee Chiao vs. Galang, etc., G. R. No.
THE LOWER COURT ERRED IN FAILING TO FIND THAT THE L-11855, Dec. 25, 1959)
COMMISSIONER OF IMMIGRATION ACTED WITH ABUSE OF DISCRETION
OR IN EXCESS OF HIS JURISDICTION WHEN SAID OFFICER THREATENED It is obvious from the nature of these objection that their proper resolution would
TO SEND OUT OF THE COUNTRY PLAINTIFF LAU YUEN YEUNG WITH necessarily cover all the points raised in appellants' assignments of error, hence, We
WARNING THAT HER FAILURE TO DO SO WOULD MEAN CONFISCATION OF will base our discussions, more or less, on said
HER BOND, ARREST AND IMMEDIATE DEPORTATION, IN SPITE OF THE objections.chanroblesvirtualawlibrarychanrobles virtual law library
FACT THAT LAU YUEN YEUNG IS NOW A FILIPINO CITIZEN.
Ichanrobles virtual law library
V
32
The first objection of the Solicitor General which covers the matters dealt with in Commissioners, 95 PMI. 785, said:
appellants' second and fourth assignments of error does not require any lengthy
discussion. As a matter of fact, it seem evident that the Solicitor General's pose that an ... It is clear that if an alien gains admission to the Islands on the strength of a deliberate
alien who has been admitted into the Philippines as a non-immigrant cannot remain and voluntary representation that he will enter only for a limited time, and thereby
here permanently unless he voluntarily leaves the country first and goes to a foreign secures the benefit of a temporary visa, the law will not allow him subsequently to go
country to secure thereat from the appropriate Philippine consul the proper visa and back on his representation and stay permanently, without first departing from the
thereafter undergo examination by officers of the Bureau of Immigration at a Philippine Philippines as he had promised. No officer can relieve him of the departure
port of entry for determination of his admissibility in accordance with the requirements requirements of section 9 of the Immigration Act, under the guise of "change" or
of the Philippine Immigration Act of 1940, as amended by Republic Act 503, is "correction", for the law makes no distinctions, and no officer is above the law. Any
premised on the assumption that petitioner Lau Yuen Yeung is not a Filipino citizen. We other ruling would, as stated in our previous decision, encourage aliens to enter the
note the same line of reasoning in the appealed decision of the court a quo. Islands on false pretences; every alien so permitted to enter for a limited time, might
Accordingly, it is but safe to assume that were the Solicitor General and His Honor of then claim a right to permanent admission, however flimsy such claim should be, and
the view that said petitioner had become ipso facto a Filipina by virtue of her marriage thereby compel our government to spend time, money and effort to examining and
to her Filipino husband, they would have held her as entitled to assume the status of a verifying whether or not every such alien really has a right to take up permanent
permanent resident without having to depart as required of aliens by Section 9 (g) of residence here. In the meanwhile, the alien would be able to prolong his stay and evade
the law.chanroblesvirtualawlibrarychanrobles virtual law library his return to the port whence he came, contrary to what he promised to do when he
entered. The damages inherent in such ruling are self-evident.
In any event, to set this point at rest, We hereby hold that portion of Section 9 (g) of the
Immigration Act providing: On the other hand, however, We cannot see any reason why an alien who has been here
as a temporary visitor but who has in the meanwhile become a Filipino should be
An alien who is admitted as a non-immigrant cannot remain in the Philippines required to still leave the Philippines for a foreign country, only to apply thereat for a
permanently. To obtain permanent admission, a non-immigrant alien must depart re-entry here and undergo the process of showing that he is entitled to come back, when
voluntarily to some foreign country and procure from the appropriate Philippine consul after all, such right has become incontestible as a necessary concomitant of his
the proper visa and thereafter undergo examination by the officers of the Bureau of assumption of our nationality by whatever legal means this has been conferred upon
Immigration at a Philippine port of entry for determination of his admissibility in him. Consider for example, precisely the case of the minor children of an alien who is
accordance with the requirements of this Act. naturalized. It is indubitable that they become ipso facto citizens of the Philippines.
Could it be the law that before they can be allowed permanent residence, they still have
does not apply to aliens who after coming into the Philippines as temporary visitors, to be taken abroad so that they may be processed to determine whether or not they have
legitimately become Filipino citizens or acquire Filipino citizenship. Such change of a right to have permanent residence here? The difficulties and hardships which such a
nationality naturally bestows upon their the right to stay in the Philippines permanently requirement entails and its seeming unreasonableness argue against such a rather
or not, as they may choose, and if they elect to reside here, the immigration authorities absurd construction. Indeed, as early as 1957, in Ly Giok Ha vs. Galang, 101 Phil. 459,
may neither deport them nor confiscate their bonds. True it is that this Court has Mr. Justice Concepcion, our present Chief Justice, already ruled thus:
vehemently expressed disapproval of convenient ruses employed by alien to convert
their status from temporary visitors to permanent residents in circumvention of the ... (P)etitioners allege that, upon her marriage to a Filipino, Ly Giok Ha became also a
procedure prescribed by the legal provision already mentioned, such as in Chiong Tiao citizen of the Philippines. Indeed, if this conclusion were correct, it would follow that,
Bing vs. Commissioner of Immigration, 99 Phil. 1020, wherein, thru Mr. Justice J.B.L. in consequence of her marriage, she had been naturalized as such citizen, and, hence
Reyes, the Court, reiterating the ruling in Ong Se Lun vs. Board of Immigration the decision appealed from would have to be affirmed, for section 40(c) of
33
Commonwealth Act 613 provides that "in the event of the naturalization as a Philippine apply for naturalization in accordance with the procedure prescribed by the Revised
citizen ... of the alien on whose behalf the bond deposit is given, the bond shall be Naturalization Law and prove in said naturalization proceeding not only that she has all
cancelled or the sum deposited shall be returned to the depositor or his legal the qualifications and none of the disqualifications provided in the law but also that she
representative." (At. pp. 462-463) has complied with all the formalities required thereby like any other applicant for
naturalization, 2albeit said decision is not yet part of our jurisprudence inasmuch as the
In other words, the applicable statute itself more than implies that the naturalization of motion for its reconsideration is still pending resolution. Appellants are in effect urging
an alien visitor as a Philippine citizen logically produces the effect of conferring upon Us, however, in their first and second assignments of error, not only to
him ipso facto all the rights of citizenship including that of being entitled to reconsider Burca but to even reexamine Lee Suan Ay which, as a matter of fact, is the
permanently stay in the Philippines outside the orbit of authority of the Commissioner prevailing rule, having been reiterated in all subsequent decisions up to Go Im
of Immigration vis-a-vis aliens, if only because by its very nature and express Ty. 3chanrobles virtual law library
provisions, the Immigration Law is a law only for aliens and is inapplicable to citizens
of the Philippines. In the sense thus discussed therefore, appellants' second and fourth Actually, the first case in which Section 15 of the Naturalization Law, Commonwealth
assignments of error are well taken.chanroblesvirtualawlibrarychanrobles virtual law Act 473, underwent judicial construction was in the first Ly Giok Ha case, 4one almost
library identical to the one at bar. Ly Giok Ha, a woman of Chinese nationality, was a
temporary visitor here whose authority to stay was to expire on March 14, 1956. She
IIchanrobles virtual law library filed a bond to guaranty her timely departure. On March 8, 1956, eight days before the
expiration of her authority to stay, she married a Filipino by the name of Restituto
Precisely, the second objection, of the Solicitor General sustained by the trial judge is Lacasta. On March 9, 1956, her husband notified the Commissioner of Immigration of
that appellant Lau Yuen Yeung's marriage to appellant Moya Lim Yao alias Edilberto said marriage and, contending that his wife had become a Filipina by reason of said
Aguinaldo whose Filipino citizenship is not denied did not have the effect of making marriage, demanded for the cancellation of her bond, but instead of acceding to such
her a Filipino, since it has not been shown that she "might herself be lawfully request, the Commissioner required her to leave, and upon her failure to do so, on
naturalized," it appearing clearly in the record that she does not possess all the March 16, 1956, the Commissioner confiscated her bond; a suit was filed for the
qualifications required of applicants for naturalization by the Revised Naturalization recovery of the bond; the lower court sustained her contention that she had no
Law, Commonwealth Act 473, even if she has proven that she does not suffer from any obligation to leave, because she had become Filipina by marriage, hence her bond
of the disqualifications thereunder. In other words, the Solicitor General implicitly should be returned. The Commissioner appealed to this Court. In the said appeal, Mr.
concedes that had it been established in the proceedings below that appellant Lau Yuen Justice Roberto Concepcion, our present Chief Justice, spoke for the Court, thus:
Yeung possesses all the qualifications required by the law of applicants for
naturalization, she would have been recognized by the respondent as a Filipino citizen The next and most important question for determination is whether her marriage to a
in the instant case, without requiring her to submit to the usual proceedings for Filipino justified or, at least, excused the aforesaid failure of Ly Giok Ha to depart from
naturalization.chanroblesvirtualawlibrarychanrobles virtual law library the Philippines on or before March 14, 1956. In maintaining the affirmative view,
petitioners alleged that, upon her marriage to a Filipino, Ly Giok Ha became, also, a
To be sure, this position of the Solicitor General is in accord with what used to be the citizen of the Philippines. Indeed, if this conclusion were correct, it would follow that,
view of this Court since Lee Suan Ay, et al. v. Emilio Galang, etc., et al., G.R. No. L- in consequence of her marriage, she had been naturalized as such citizen, and, hence,
11855, promulgated December 23, 1959, 106 Phil., 706,713, 1for it was only in Zita the decision appealed from would have to be affirmed, for section 40(c) of
Ngo Burca vs. Republic, G.R. NO. L-24252 which was promulgated on January 30, Commonwealth Act No. 613 provides that "in the event of the naturalization as a
1967 (19 SCRA 186), that over the pen of Mr. Justice Conrado Sanchez, this Court held Philippine citizen ... of the alien on whose behalf the bond deposit is given, the bond
that for an alien woman who marries a Filipino to be deemed a Filipina, she has to shall be cancelled or the sum deposited shall be returned to the depositor or his legal
34
representative." Thus the issue boils down to whether an alien female who marries a (g) Citizens or subjects of nations with whom the ... Philippines are at war, during the
male citizen of the Philippines follows ipso facto his political period of such war;chanrobles virtual law library
status.chanroblesvirtualawlibrarychanrobles virtual law library
(h) Citizens or subjects of a foreign country other than the United States, whose laws
The pertinent part of section 15 of Commonwealth Act No. 473, upon which petitioners does not grant Filipinos the right to become naturalized citizens or subjects thereof.
rely, reads:
In the case at bar, there is neither proof nor allegation in the pleadings that Ly Giok Ha
Any woman who is now or may hereafter be married to a citizen of the Philippines, and does not fall under any of the classes disqualified by law. Moreover, as the parties who
who might herself be lawfully naturalized shall be deemed a citizen of the Philippines. claim that, despite her failure to depart from the Philippines within the period specified
in the bond in question, there has been no breach thereof, petitioners have the burden of
Pursuant thereto, marriage to a male Filipino does not vest Philippine citizenship to his proving her alleged change of political status, from alien to citizen. Strictly speaking,
foreign wife, unless she "herself may be lawfully naturalized." As correctly held in an petitioners have not made out, therefore a case against the respondents-
opinion of the Secretary of Justice (Op. No. 52, series of 1950),* this limitation of appellants.chanroblesvirtualawlibrarychanrobles virtual law library
section 15 excludes, from the benefits of naturalization by marriage, those disqualified
from being naturalized as citizens of the Philippines under section 4 of said Considering, however, that neither in the administrative proceedings, nor in the lower
Commonwealth Act No. 473, namely: court, had the parties seemingly felt that there was an issue on whether Ly Giok Ha
may "be lawfully naturalized," and this being a case of first impression in our courts,
(a) Persons opposed to organized government or affiliated with any association or we are of the opinion that, in the interest of equity and justice, the parties herein should
group of persons who uphold and teach doctrines opposing all organized be given an opportunity to introduce evidence, if they have any, on said issue. (At pp.
governments;chanrobles virtual law library 462-464.) .

(b) Persons defending or teaching the necessity or propriety of violence, personal As may be seen, although not specifically in so many words, no doubt was left in the
assault, or assassination for the success and predominance of their ideas;chanrobles above decision as regards the following
virtual law library propositions: .chanroblesvirtualawlibrarychanrobles virtual law library

(c) Polygamists or believers in the practice of polygamy;chanrobles virtual law library 1. That under Section 15 of Commonwealth Act 473, the Revised Naturalization Law,
the marriage of an alien woman to a Filipino makes her a Filipina, if she "herself might
(d) Persons convicted of crimes involving moral turpitude;chanrobles virtual law be lawfully naturalized";chanrobles virtual law library
library
2. That this Court declared as correct the opinion of the Secretary of Justice that the
(e) Persons suffering from mental alienation or incurable contagious limitation of Section 15 of the Naturalization Law excludes from the benefits of
diseases;chanrobles virtual law library naturalization by marriage, only those disqualified from being naturalized under
Section 4 of the law qouted in the decision;chanrobles virtual law library
(f) Persons who, during the period of their residence in the Philippines, have not
mingled socially with the Filipinos, or who have not evinced a sincere desire to learn 3. That evidence to the effect that she is not disqualified may be presented in the action
and embrace the customs, traditions, and ideals of the Filipinos;chanrobles virtual law to recover her bond confiscated by the Commissioner of Immigration;chanrobles
library virtual law library
35
4. That upon proof of such fact, she may be recognized as Filipina; andchanrobles which reads in part as follows:
virtual law library
Any woman who is now or may hereafter be married to a citizen of the Philippines, and
5. That in referring to the disqualification enumerated in the law, the Court somehow who might herself be lawfully naturalized shall be deemed a citizen of the Philippines.
left the impression that no inquiry need be made as to qualifications, 5specially
considering that the decision cited and footnotes several opinions of the Secretary of The phrase "who might herself be lawfully naturalized", as contained in the above
Justice, the immediate superior of the Commissioner of Immigration, the most provision, means that the woman who is married to a Filipino citizen must not belong
important of which are the following: to any of the disqualified classes enumerated in Section 4 of the Naturalization Law
(Ops., Sec. of Jus., No. 28, s. 1950; No. 43, s. 1948, No. 95, s. 1941; Nos. 79 and 168,
Paragraph (a), section 13 of Act No. 2927, as amended, (now section 15, s. 1940). Under the facts stated in the within papers, Mrs. Machura does not appear to
Commonwealth Act No. 473), provided that "any woman who is now or may hereafter be among the disqualified classes mentioned in the
be married to a citizen of the Philippines, and who might herself be lawfully law.chanroblesvirtualawlibrarychanrobles virtual law library
naturalized shall be deemed a citizen of the Philippines." A similar provision in the
naturalization law of the United States has been construed as not requiring the woman It having been shown that Arce Machura or Arsenio Guevara was born as an
to have the qualifications of residence, good character, etc., as in the case of illegitimate of a Filipino mother, he should be considered as a citizen of the Philippines
naturalization by judicial proceedings, but merely that she is of the race of persons who in consonance with the well-settled rule that an illegitimate child follows the
may be naturalized. (Kelly v. Owen [Dist. Col. 1868] 7 Wall 496, 5F, 11, 12; ex citizenship of his only legally recognized parent, the mother (Op., Sec. of Jus., Nos. 58,
parte Tryason [D. C. Wash. 1914] 215 F. 449, 27 Op. Atty. Gen. 507). (Op. No. 168, s. 98 & 281, s. 1948; No. 96, s. 1949). Her husband being a Filipino, Mrs. Machura must
1940 of Justice Sec. Jose Abad Santos.)chanrobles virtual law library necessarily be deemed as a citizen of the Philippines by marriage (Sec. 15, Com. Act
No. 473.) (Op. No. 52, s. 1950 of Justice Sec. Ricardo Nepomuceno.)
In a previous opinion rendered for your Office, I stated that the clause "who might
herself be lawfully naturalized", should be construed as not requiring the woman to The logic and authority of these opinions, compelling as they are, must have so
have the qualifications of residence, good character, etc., as in cases of naturalization appealed to this Court that five days later, on May 22, 1957, in Ricardo Cua v. The
by judicial proceedings, but merely that she is of the race of persons who may be Board of Commissioners, 101 Phil. 521, Mr. Justice J.B.L. Reyes, reiterated the same
naturalized. (Op. No. 79, s. 1940)chanrobles virtual law library ruling on the basis of the following facts:chanrobles virtual law library

Inasmuch as the race qualification has been removed by the Revised Naturalization Tjioe Wu Suan, an Indonesian, arrived in Manila on November 1, 1952, but it turned
Law, it results that any woman who married a citizen of the Philippines prior to or after out that her passport was forged. On December 10, 1953, a warrant was issued for her
June 17, 1939, and the marriage not having been dissolved, and on the assumption that arrest for purpose of deportation. Later, on December 20, 1953, she married Ricardo
she possesses none of the disqualifications mentioned in Section 4 of Commonwealth Cua, a Filipino, and because of said marriage, the Board of Special Inquiry considered
Act No. 473, follows the citizenship of her husband. (Op. No. 176, s. 1940 of Justice her a Filipina. Upon a review of the case, however, the Board of Immigration
Sec. Jose Abad Santos.)chanrobles virtual law library Commissioners insisted on continuing with the deportation proceedings and so, the
husband filed prohibition and mandamus proceedings. The lower court denied the
From the foregoing narration of facts, it would seem that the only material point of petition. Although this Court affirmed said decision, it held, on the other hand, that:
inquiry is as to the citizenship of Arce Machura. If he shall be found to be a citizen of
the Philippines, his wife, Mrs. Lily James Machura, shall likewise be deemed a citizen Granting the validity of marriage, this Court has ruled in the recent case of Ly Giok Ha
of the Philippines pursuant to the provision of Section 15, Commonwealth Act No. 473, v. Galang, supra, p. 459, that the bare fact of a valid marriage to a citizen does not
36
suffice to confer his citizenship upon the wife. Section 15 of the Naturalization Law proceedings, is not necessary. (See: Leonard v. Grant, 5 Fed. 11; 27 Ops. Atty. Gen
requires that the alien woman who marries a Filipino must show, in addition, that she [U.S.] 507; Ops. Sec. of Justice, No. 776, s. 1940, and No. 111, s.
"might herself be lawfully naturalized" as a Filipino citizen. As construed in the 1953.chanroblesvirtualawlibrarychanrobles virtual law library
decision cited, this last condition requires proof that the woman who married a Filipino
is herself not disqualified under section 4 of the Naturalization This view finds support in the case of Ly Giok Ha et al. v. Galang et al., G.R. No. L-
Law.chanroblesvirtualawlibrarychanrobles virtual law library 10760, promulgated May 17, 1957, where the Supreme Court, construing the
abovequoted section of the Naturalization Law, held that "marriage to a male Filipino
No such evidence appearing on record, the claim of assumption of Filipino citizenship does not vest Philippine citizenship to his foreign wife," unless she "herself may be
by Tjioe Wu Suan, upon her marriage to petitioner, is untenable. The lower court, lawfully naturalized," and that "this limitation of Section 15 excludes, from the benefits
therefore, committed no error in refusing to interfere with the deportation proceedings, of naturalization by marriage, those disqualified from being naturalized as citizens of
where she can anyway establish the requisites indispensable for her acquisition of the Philippines under Section 4 of said Commonwealth Act No. 473." In other words,
Filipino citizenship, as well as the alleged validity of her Indonesian passport. (Ricardo disqualification for any of the causes enumerated in Section 4 of the Act is the decisive
Cua v. The Board of Immigration Commissioners, G. R. No. L-9997, May 22, 1957, factor that defeats the right of the foreign wife of a Philippine citizen to acquire
101 Phil. 521, 523.) [Emphasis supplied] . Philippine citizenship.chanroblesvirtualawlibrarychanrobles virtual law library

For emphasis, it is reiterated that in the above two cases, this Court expressly gave the xxx xxx xxxchanrobles virtual law library
parties concerned opportunity to prove the fact that they were not suffering from any of
the disqualifications of the law without the need of undergoing any judicial Does petitioner, Lim King Bian, belong to any of these groups The Commissioner of
naturalization proceeding. It may be stated, therefore, that according to the above Immigration does not say so but merely predicates his negative action on the ground
decisions, the law in this country, on the matter of the effect of marriage of an alien that a warrant of deportation for "overstaying" is pending against the
woman to a Filipino is that she thereby becomes a Filipina, if it can be proven that at petitioner.chanroblesvirtualawlibrarychanrobles virtual law library
the time of such marriage, she does not possess any of the disqualifications enumerated
in Section 4 of the Naturalization Law, without the need of submitting to any We do not believe the position is well taken. Since the grounds for disqualification for
naturalization proceedings under said law.chanroblesvirtualawlibrarychanrobles virtual naturalization are expressly enumerated in the law, a warrant of deportation not based
law library on a finding of unfitness to become naturalized for any of those specified causes may
not be invoked to negate acquisition of Philippine citizenship by a foreign wife of a
It is to be admitted that both of the above decisions made no reference to qualifications, Philippine citizen under Section 15 of the Naturalization Law. (Inclusio unius est
that is, as to whether or not they need also to be proved, but, in any event, it is a fact exclusio alterius) (Op. No. 12, s. 1958 of Justice Undersec. Jesus G.
that the Secretary of Justice understood them to mean that such qualifications need not Barrera.)chanrobles virtual law library
be possessed nor proven. Then Secretary of Justice Jesus Barrera, who later became a
distinguished member of this Court, 6 so ruled in opinions rendered by him subsequent Regarding the steps that should be taken by an alien woman married to a Filipino
to Ly Giok Ha, the most illustrative of which held: . citizen in order to acquire Philippine citizenship, the procedure followed in the Bureau
of Immigration is as follows: The alien woman must file a petition for the cancellation
At the outset it is important to note that an alien woman married to a Filipino citizen of her alien certificate of registration alleging, among other things, that she is married
needs only to show that she "might herself be lawfully naturalized" in order to acquire to a Filipino citizen and that she is not disqualified from acquiring her husband's
Philippine citizenship. Compliance with other conditions of the statute, such as those citizenship pursuant to section 4 of Commonwealth Act No. 473, as amended. Upon the
relating to the qualifications of an applicant for naturalization through judicial filing of said petition, which should be accompanied or supported by the joint affidavit
37
of the petitioner and her Filipino husband to the effect that the petitioner does not receipt of notice, otherwise the bond will be confiscated(Annex 1). For failure of the
belong to any of the groups disqualified by the cited section from becoming naturalized bondsman to comply with the foregoing order, on 1 April 1955. the Commissioner of
Filipino citizen (please see attached CEB Form 1), the Bureau of Immigration conducts Immigration ordered the cash bond confiscated (Annex E). Therefore, there was an
an investigation and thereafter promulgates its order or decision granting or denying the order issued by the Commissioner of Immigration confiscating or forfeiting the cash
petition. (Op. No. 38, s. 19058 of Justice Sec. Jesus G. Barrera.)chanrobles virtual law bond. Unlike in forfeiture of bail bonds in criminal proceedings, where the Court must
library enter an order forfeiting the bail bond and the bondsman must be given an opportunity
to present his principal or give a satisfactory reason for his inability to do so, before
This view finds support in the case of Ly Giok Ha et al., v. Galang et al. (G.R. No. L- final judgment may be entered against the bondsman,(section 15, Rule 110; U.S. v.
10760, promulgated May 17, 1957), where the Supreme Court, construing the above- Bonoan, 22 Phil. 1.) in forfeiture of bonds posted for the temporary stay of an alien in
quoted section in the Revised Naturalization Law, held that "marriage to a male the Philippines, no court proceeding is necessary. Once a breach of the terms and
Filipino does not vest Philippine citizenship to his foreign wife, unless she herself may conditions of the undertaking in the bond is committed, the Commissioner of
be lawfully naturalized," and that "this limitation of Section 15 excludes, from the Immigration may, under the terms and conditions thereof, declare it forfeited in favor
benefits of naturalization by marriage, those disqualified from being naturalized as of the Government. (In the meanwhile, on April 1, 1955, Lee Suan Ay and Alberto Tan,
citizens of the Philippines under Section 4 of said Commonwealth Act No. 473." In a Filipino, were joined in marriage by the Justice of the Peace of Las Piñas, Rizal.)
other words, disqualification for any of the causes enumerated in section 4 of the Act is
the decisive factor that defeats the right of an alien woman married to a Filipino citizen Mr. Justice Sabino Padilla speaking for a unanimous court which included Justices
to acquire Philippine citizenship. (Op. 57, s. 1958 of Justice Sec. Jesus G. Concepcion and Reyes who had penned Ly Giok Ha, and Ricardo Cua, ruled thus:
Barrera.)chanrobles virtual law library
The fact that Lee Suan Ay (a Chinese) was married to a Filipino citizen does not relieve
The contention is untenable. The doctrine enunciated in the Ly Giok Ha case is not a the bondsman from his liability on the bond. The marriage took place on 1 April 1955,
new one. In that case, the Supreme Court held that under paragraph I of Section 15 Of and the violation of the terms and conditions of the undertaking in the bond - failure to
Commonwealth Act No. 473, 'marriage to a male Filipino does not vest Philippine depart from the Philippines upon expiration of her authorized period of temporary stay
citizenship to his foreign wife unless she "herself may be lawfully naturalized"', and, in the Philippines (25 March 1955) and failure to report to the Commissioner of
quoting several earlier opinions of the Secretary of Justice, namely: No. 52, s. 1950; Immigration within 24 hours from receipt of notice - were committed before the
No. 168, s. 1940; No. 95, s. 1941; No. 63, s. 1948; No. 28. s. 1950, "this limitation of marriage. Moreover, the marriage of a Filipino citizen to an alien does not
section 15 excludes from the benefits of naturalization by marriage, those disqualified automatically confer Philippine citizenship upon the latter. She must possess the
from being naturalized as citizens of the Philippines under section 4 of said qualifications required by law to become a Filipino citizen by naturalization.* There is
Commonwealth Act No. 473." (Op. 134, s. 1962 of Justice Undersec. Magno S. no showing that the appellant Lee Suan Ay possesses all the qualifications and none of
Gatmaitan.) the disqualifications provided for by law to become a Filipino citizen by naturalization.

It was not until more than two years later that, in one respect, the above construction of Pertinently to be noted at once in this ruling, which, to be sure, is the one relied upon in
the law was importantly modified by this Court in Lee Suan Ay, supra, in which the the appealed decision now before Us, is the fact that the footnote of the statement
facts were as follows: therein that the alien wife "must possess the qualifications required by law to become a
Filipino citizen by naturalization" makes reference to Section 15, Commonwealth Act
Upon expiration of the appellant Lee Suan Ay's authorized period of temporary stay in 473 and precisely, also to Ly Giok Ha v. Galang, supra. As will be recalled, on the other
the Philippines (25 March 1955), on 26 March 1955 the Commissioner of Immigration hand, in the opinions of the Secretary of Justice explicitly adopted by the Court in Ly
asked the bondsman to present her to the Bureau of Immigration within 24 hours from Giok Ha, among them, Opinion No. 176, Series of 1940, above-quoted, it was clearly
38
held that "(I)n a previous opinion rendered for your Office, I stated that the clause "who Naturalization Law, Commonwealth Act 473. When the case reached the court, the trial
might herself be lawfully naturalized", should be construed as not requiring the woman judge held for the government that in addition to not having any of the disqualifications
to have the qualifications of residence, good character, etc., as in cases of naturalization referred to, there was need that Lo San Tuang should have also possessed all the
by judicial proceedings but merely that she is of the race by persons who may be qualifications of residence, moral character, knowledge of a native principal dialect,
naturalized. (Op. No. 79, s. 1940)chanrobles virtual law library etc., provided by the law. Recognizing that the issue squarely to be passed upon was
whether or not the possession of all the qualifications were indeed needed to be shown
Since Justice Padilla gave no reason at all for the obviously significant modification of apart from non-disqualification, Justice Regala held affirmatively for the Court,
the construction of the law, it could be said that there was need for clarification of the reasoning out thus: .
seemingly new posture of the Court. The occasion for such clarification should have
been in Kua Suy, etc., et al. vs. The Commissioner of Immigration, G.R. No. L-13790, It is to be noted that the petitioner has anchored her claim for citizenship on the basis of
October 31, 1963, penned by Mr. Justice J.B.L. Reyes, who had rendered the opinion in the decision laid down in the case of Leonard v. Grant, 5 Swy. 603, 5 F 11, where the
Ricardo Cua, supra, which followed that in Ly Giok Ha, supra, but apparently seeing no Circuit Court of Oregon held that it was only necessary that the woman "should be a
immediate relevancy in the case on hand then of the particular point in issue now, since person of the class or race permitted to be naturalized by existing laws, and that in
it was not squarely raised therein similarly as in Lee Suan Ay, hence, anything said on respect of the qualifications arising out of her conduct or opinions, being the wife of a
the said matter would at best be no more than obiter dictum, Justice Reyes limited citizen, she is to be regarded as qualified for citizenship, and therefore considered a
himself to holding that "Under Section 15 of the Naturalization Act, the wife is deemed citizen." (In explanation of its conclusion, the Court said: "If, whenever during the life
a citizen of the Philippines only if she "might herself be lawfully naturalized," so that of the woman or afterwards, the question of her citizenship arises in a legal proceeding,
the fact of marriage to a citizen, by itself alone, does not suffice to confer citizenship, the party asserting her citizenship by reason of her marriage with a citizen must not
as this Court has previously ruled in Ly Giok Ha v. Galang, 54 O.G. 356, and in Cua v. only prove such marriage, but also that the woman then possessed all the further
Board of Immigration Commissioners, 53 O.G. 8567; and there is here no evidence of qualifications necessary to her becoming naturalized under existing laws, the statute
record as to the qualifications or absence of disqualifications of appellee Kua Suy", will be practically nugatory, if not a delusion and a share. The proof of the facts may
without explaining the apparent departure already pointed out from Ly Giok Ha and have existed at the time of the marriage, but years after, when a controversy arises upon
Ricardo Cua. Even Justice Makalintal, who wrote a separate concurring and dissenting the subject, it may be lost or difficult to find.")chanrobles virtual law library
opinion merely lumped together Ly Giok Ha, Ricardo Cua and Lee Suan Ay and opined
that both qualifications and non-disqualifications have to be shown without elucidating In other words, all that she was required to prove was that she was a free white woman
on what seemed to be departure from the said first two or a woman of African descent or nativity, in order to be deemed an American citizen,
decisions.chanroblesvirtualawlibrarychanrobles virtual law library because, with respect to the rest of the qualifications on residence, moral character, etc.,
she was presumed to be qualified.chanroblesvirtualawlibrarychanrobles virtual law
It was only on November 30, 1963 that to Mr. Justice Roberto Regala fell the task of library
rationalizing the Court's position. In Lo San Tuang v. Galang, G.R. No. L-18775,
November 30, 1963, 9 SCRA 638, the facts were simply these: Lo San Tuang, a Like the law in the United States, our former Naturalization Law (Act No. 2927, as
Chinese woman, arrived in the Philippines on July 1, 1960 as a temporary visitor with amended by Act No. 3448) specified the classes of persons who alone might become
authority to stay up to June 30, 1961. She married a Filipino on January 7, 1961, almost citizens of the Philippines, even as it provided who were disqualified. Thus, the
six months before the expiry date of her permit, and when she was requested to leave pertinent provisions of that law provided:
after her authority to stay had expired, she refused to do so, claiming she had become a
Filipina by marriage, and to bolster her position, she submitted an affidavit stating Section 1. Who may become Philippine citizens - Philippine citizenship may be
explicitly that she does not possess any of the disqualifications enumerated in the acquired by (a) natives of the Philippines who are not citizens thereof under the Jones
39
Law; (b) natives of the Insular possessions of the United States; (c) citizens of the Fourth. To speak and write English, Spanish, or some native
United States, or foreigners who under the laws of the United States may become tongue.chanroblesvirtualawlibrarychanrobles virtual law library
citizens of said country if residing therein.chanroblesvirtualawlibrarychanrobles virtual
law library In case the petitioner is a foreign subject, he shall, besides, declare in writing and under
oath his intention of renouncing absolutely and perpetually all faith and allegiance to
Section 2. Who are disqualified. - The following cannot be naturalized as Philippine the foreign authority, state or sovereignty of which he was a native, citizen or subject.
citizens: (a) Persons opposed to organized government or affiliated with any
association or group of persons who uphold and teach doctrines opposing all organized Applying the interpretation given by Leonard v. Grant supra, to our law as it then stood,
government; (b) persons defending or teaching the necessity or propriety of violence, alien women married to citizens of the Philippines must, in order to be deemed citizens
personal assault or assassination for the success and predominance of their ideas; (c) of the Philippines, be either (1) natives of the Philippines who were not citizens thereof
polygamists or believers in the practice of polygamy; (d) persons convicted of crimes under the Jones Law, or (2) natives of other Insular possessions of the United States, or
involving moral turpitude; (e) persons suffering from mental alienation or incurable (3) citizens of the United States or foreigners who under the laws of the United States
contagious diseases; (f) citizens or subjects of nations with whom the United States and might become citizens of that country if residing therein. With respect to the
the Philippines are at war, during the period of such qualifications set forth in Section 3 of the former law, they were deemed to have the
war.chanroblesvirtualawlibrarychanrobles virtual law library same for all intents and purposes.chanroblesvirtualawlibrarychanrobles virtual law
library
Section 3. Qualifications. - The persons comprised in subsection (a) of section one of
this Act, in order to be able to acquire Philippine citizenship, must be not less than But, with the approval of the Revised Naturalization Law (Commonwealth Act No.
twenty-one years of age on the day of the hearing of their 473) on June 17, 1939, Congress has since discarded class or racial consideration from
petition.chanroblesvirtualawlibrarychanrobles virtual law library the qualifications of applicants for naturalization (according to its proponent, the
purpose in eliminating this consideration was, first, to remove the features of the
The persons comprised in subsections (b) and (c) of said section one shall, in addition existing naturalization act which discriminated in favor of the Caucasians and against
to being not less than twenty-one years of age on the day of the hearing of the petition, Asiatics who are our neighbors, and are related to us by racial affinity and, second, to
have all and each of the following qualifications:chanrobles virtual law library foster amity with all nations [Sinco, Phil. Political Law 502 - 11 ed.]), even as it
retained in Section 15 the phrase in question. The result is that the phrase "who might
First. Residence in the Philippine Islands for a continuous period of not less than five herself be lawfully naturalized" must be understood in the context in which it is now
years, except as provided in the next following section;chanrobles virtual law library found, in a setting so different from that in which it was found by the Court in Leonard
v. Grant.chanroblesvirtualawlibrarychanrobles virtual law library
Second. To have conducted themselves in a proper and irreproachable manner during
the entire period of their residence in the Philippine Islands, in their relation with the The only logical deduction from the elimination of class or racial consideration is that,
constituted government as well as with the community in which they are as the Solicitor General points out, the phrase "who might herself be lawfully
living;chanrobles virtual law library naturalized" must now be understood as referring to those who under Section 2 of the
law are qualified to become citizens of the
Third. To hold in the Philippine Islands real estate worth not less than one thousand Philippines.chanroblesvirtualawlibrarychanrobles virtual law library
pesos, Philippine currency, or have some known trade or profession; andchanrobles
virtual law library There is simply no support for the view that the phrase "who might herself be lawfully
naturalized" must now be understood as requiring merely that the alien woman must
40
not belong to the class of disqualified persons under Section 4 of the Revised Furthermore, the fact that a decision was favorably made on the naturalization petition
Naturalization Law. Such a proposition misreads the ruling laid down in Leonard v. of her husband is no assurance that he (the husband) would become a citizen, as to
Grant. A person who is not disqualified is not necessarily qualified to become a citizen make a basis for the extension of her temporary stay.
of the Philippines, because the law treats "qualifications" and "disqualifications" in
separate sections. And then it must not be lost sight of that even under the interpretation On the same day, in Tong Siok Sy v. Vivo, G.R. No. L-21136, December 27, 1963, 9
given to the former law, it was to be understood that the alien woman was not SCRA 876, Justice Barrera reiterated the same ruling and citing particularly Lo San
disqualified under Section 2 of that law. Leonard v. Grant did not rule that it was Tuang and Kua Suy, held that the marriage of Tong Siok Sy to a Filipino on November
enough if the alien woman does not belong to the class of disqualified persons in order 12, 1960 at Taichung, Taiwan and her taking oath of Filipino citizenship before the
that she may be deemed to follow the citizenship of her husband: What that case held Philippine Vice-Consul at Taipeh, Taiwan on January 6, 1961 did not make her a
was that the phrase "who might herself be lawfully naturalized, merely means that she Filipino citizen, since she came here only in 1961 and obviously, she had not had the
belongs to the class or race of persons qualified to become citizens by naturalization - necessary ten-year residence in the Philippines required by the
the assumption being always that she is not otherwise law.chanroblesvirtualawlibrarychanrobles virtual law library
disqualified.chanroblesvirtualawlibrarychanrobles virtual law library
Such then was the status of the jurisprudential law on the matter under discussion when
We therefore hold that under the first paragraph of Section 15 of the Naturalization Justice Makalintal sought a reexamination thereof in Choy King Tee v. Galang, G.R.
Law, an alien woman, who is married to a citizen of the Philippines, acquires the No. L-18351, March 26, 1965, 13 SCRA 402. Choy King Tee's husband was granted
citizenship of her husband only if she has all the qualifications and none of the Philippine citizenship on January 13, 1959 and took the oath on January 31 of the same
disqualifications provided by law. Since there is no proof in this case that petitioner has year. Choy King Tee first came to the Philippines in 1955 and kept commuting between
all the qualifications and is not in any way disqualified, her marriage to a Filipino Manila and Hongkong since then, her last visa before the case being due to expire on
citizen does not automatically make her a Filipino citizen. Her affidavit to the effect February 14, 1961. On January 27, 1961, her husband asked the Commissioner of
that she is not in any way disqualified to become a citizen of this country was correctly Immigration to cancel her alien certificate of registration, as well as their child's, for the
disregarded by the trial court, the same being self-serving. reason that they were Filipinos, and when the request was denied as to the wife, a
mandamus was sought, which the trial court granted. Discussing anew the issue of the
Naturally, almost a month later in Sun Peck Yong v. Commissioner of Immigration, need for qualifications, Justice Makalintal not only reiterated the arguments of Justice
G.R. No. L-20784, December 27, 1963, 9 SCRA 875, wherein the Secretary of Foreign Regala in Lo San Tuang but added further that the ruling is believed to be in line with
Affairs reversed a previous resolution of the preceding administration to allow Sun the national policy of selective admission to Philippine citizenship. 7chanrobles virtual
Peck Yong and her minor son to await the taking of the oath of Filipino citizenship of law library
her husband two years after the decision granting him nationalization and required her
to leave and this order was contested in court, Justice Barrera held: No wonder, upon this authority, in Austria v. Conchu, G.R. No. L-20716, June 22,
1965, 14 SCRA 336, Justice J.P. Bengzon readily reversed the decision of the lower
In the case of Lo San Tuang v. Commissioner of Immigration (G.R. No. L-18775, court granting the writs of mandamus and prohibition against the Commissioner of
promulgated November 30, 1963; Kua Suy vs. Commissioner of Immigration, L- Immigration, considering that Austria's wife, while admitting she did not possess all the
13790, promulgated October 31, 1963), we held that the fact that the husband became a qualifications for naturalization, had submitted only an affidavit that she had none of
naturalized citizen does not automatically make the wife a citizen of the Philippines. It the disqualifications therefor. So also did Justice Dizon similarly hold eight days later
must also be shown that she herself possesses all the qualifications, and none of the in Brito v. Commissioner, G.R. No. L-16829, June 30, 1965, 14 SCRA
disqualifications, to become a citizen. In this case, there is no allegation, much less 539.chanroblesvirtualawlibrarychanrobles virtual law library
showing, that petitioner-wife is qualified to become a Filipino citizen herself.
41
Then came the second Ly Giok Ha case 8wherein Justice J. B. L. Reyes took occasion to virtual law library
expand on the reasoning of Choy King Tee by illustrating with examples "the danger of
relying exclusively on the absence of disqualifications, without taking into account the A child born outside of the Philippines after the naturalization of his parent, shall be
other affirmative requirements of the law." 9chanrobles virtual law library considered a Philippine citizen unless within one year after reaching the age of majority
he fails to register himself as a Philippine citizen at the American Consulate of the
Lastly, in Go Im Ty v. Republic, G.R. No. L-17919, decided on July 30, 1966, 10Justice country where he resides, and to take the necessary oath of allegiance.
Zaldivar held for the Court that an alien woman who is widowed during the
dependency of the naturalization proceedings of her husband, in order that she may be is it necessary, in order that an alien woman who marries a Filipino or who is married
allowed to take the oath as Filipino, must, aside from proving compliance with the to a man who subsequently becomes a Filipino, may become a Filipino citizen herself,
requirements of Republic Act 530, show that she possesses all the qualifications and that, aside from not suffering from any of the disqualifications enumerated in the law,
does not suffer from any of the disqualifications under the Naturalization Law, citing in she must also possess all the qualifications required by said law? if nothing but the
the process the decision to such effect discussed above, 11even as he impliedly unbroken line from Lee Suan Ay to Go Im Ty, as recounted above, were to be
reversed pro tanto the ruling in Tan Lin v. Republic, G.R. No. L-13786, May 31, 1961, considered, it is obvious that an affirmative answer to the question would be inevitable,
2 SCRA 383.chanroblesvirtualawlibrarychanrobles virtual law library specially, if it is noted that the present case was actually submitted for decision on
January 21, 1964 yet, shortly after Lo San Tuang, Tong Siok Sy and Sun Peck Yong,
Accordingly, in Burca, Justice Sanchez premised his opinion on the assumption that the all supra, and even before Choy King Tee, supra, were decided. There are other
point now under discussion is settled law.chanroblesvirtualawlibrarychanrobles virtual circumstances, however, which make it desirable, if not necessary, that the Court take
law library up the matter anew. There has been a substantial change in the membership of the
Court since Go Im Ty, and of those who were in the Court already when Burca was
In the case now at bar, the Court is again called upon to rule on the same issue. Under decided, two members, Justice Makalintal and Castro concurred only in the result,
Section 15 of the Naturalization Law, Commonwealth Act 473, providing that: precisely, according to them, because (they wanted to leave the point now under
discussion open in so far as they are concerned. 12Truth to tell, the views and arguments
SEC. 15. Effect of the naturalization on wife and children. - Any woman, who is now discussed at length with copious relevant authorities, in the motion for reconsideration
or may hereafter be married to a citizen of the Philippines, and who might herself be as well as in the memorandum of the amici curae 13in the Burca case cannot just be
lawfully naturalized shall be deemed a citizen of the taken lightly and summarily ignored, since they project in the most forceful manner,
Philippines.chanroblesvirtualawlibrarychanrobles virtual law library not only the legal and logical angles of the issue, but also the imperative practical
aspects thereof in the light of the actual situation of the thousands of alien wives of
Minor children of persons naturalized under this law who have been born in the Filipinos who have so long, even decades, considered themselves as Filipinas and have
Philippines shall be considered citizens thereof.chanroblesvirtualawlibrarychanrobles always lived and acted as such, officially or otherwise, relying on the long standing
virtual law library continuous recognition of their status as such by the administrative authorities in charge
of the matter, as well as by the courts. Under these circumstances, and if only to afford
A foreign-born minor child, if dwelling in the Philippines at the time of the the Court an opportunity to consider the views of the five justices who took no part in
naturalization of the parent, shall automatically become a Philippine citizen, and a Go Im Ty (including the writer of this opinion), the Court decided to further reexamine
foreign-born child, who is not in the Philippines at the time the parent is naturalized, the matter. After all, the ruling first laid in Lee Suan Ay, and later in Lo San Tuang,
shall be deemed a Philippine citizen only during his minority, unless he begins to reside Choy King Tee stand the second (1966) Ly Giok Ha, did not categorically repudiate the
permanently in the Philippines when still a minor, in which case, he will continue to be opinions of the Secretary of Justice relied upon by the first (1959) Ly Giok Ha.
a Philippine citizen even after becoming of age.chanroblesvirtualawlibrarychanrobles Besides, some points brought to light during the deliberations in this case would seem
42
to indicate that the premises of the later cases can still bear further not come within the foregoing provisions, the natives of other insular possessions of the
consideration.chanroblesvirtualawlibrarychanrobles virtual law library United States, and such other persons residing in the Philippine Islands who would
become citizens of the United States, under the laws of the United States, if residing
Whether We like it or not, it is undeniably factual that the legal provision We are therein.
construing, Section 15, aforequoted, of the Naturalization Law has been taken directly,
copied and adopted from its American counterpart. To be more accurate, said provision The Jones Law reenacted these provisions substantially: .
is nothing less than a reenactment of the American provision. A brief review of its
history proves this beyond per adventure of SECTION 2. That all inhabitants of the Philippine Islands who were Spanish subjects
doubt.chanroblesvirtualawlibrarychanrobles virtual law library on the eleventh day of April, eighteen hundred and ninety-nine, and then resided in said
islands, and their children born subsequent thereto, shall be deemed and held to be
The first Naturalization Law of the Philippines approved by the Philippine Legislature citizens of the Philippine Islands, except such as shall have elected to preserve their
under American sovereignty was that of March 26, 1920, Act No. 2927. Before then, as allegiance to the Crown of Spain in accordance with the provisions of the treaty of
a consequence of the Treaty of Paris, our citizenship laws were found only in the peace between the United States and Spain, signed at Paris December tenth, eighteen
Organic Laws, the Philippine Bill of 1902, the Act of the United States Congress of hundred and ninety-eight and except such others as have since become citizens of some
March 23, 1912 and later the Jones Law of 1916. In fact, Act No. 2927 was enacted other country: Provided, That the Philippine Legislature, herein provided for, is hereby
pursuant to express authority granted by the Jones Law. For obvious reasons, the authorized to provide by law for the acquisition of Philippine citizenship by those
Philippines gained autonomy on the subjects of citizenship and immigration only after natives of the Philippine Islands who do not come within the foregoing provisions, the
the effectivity of the Philippine Independence Act. This made it practically impossible natives of the insular possessions of the United States, and such other persons residing
for our laws on said subject to have any perspective or orientation of our own; in the Philippine Islands who are citizens of the United States under the laws of the
everything was American.chanroblesvirtualawlibrarychanrobles virtual law library United States if residing therein.

The Philippine Bill of 1902 provided pertinently: . For aught that appears, there was nothing in any of the said organic laws regarding the
effect of marriage to a Filipino upon the nationality of an alien woman, albeit under the
SECTION 4. That all inhabitants of the Philippine Islands continuing to reside herein Spanish Civil Code provisions on citizenship, Articles 17 to 27, which were, however,
who were Spanish subjects on the eleventh day of April, eighteen-hundred and ninety- abrogated upon the change of sovereignty, it was unquestionable that the citizenship of
nine, and then resided in said Islands, and their children born subsequent thereto, shall the wife always followed that of the husband. Not even Act 2927 contained any
be deemed and held to be citizens of the Philippine Islands and as such entitled to the provision regarding the effect of naturalization of an alien, upon the citizenship of his
protection of the United States, except such as shall have elected to preserve their alien wife, nor of the marriage of such alien woman with a native born Filipino or one
allegiance to the Crown of Spain in accordance with the provisions of the treaty of who had become a Filipino before the marriage, although Section 13 thereof provided
peace between the United States and Spain signed at Paris December tenth, eighteen thus: .
hundred and ninety-eight.
SEC. 13. Right of widow and children of petitioners who have died. - In case a
This Section 4 of the Philippine Bill of 1902 was amended by Act of Congress of petitioner should die before the final decision has been rendered, his widow and minor
March 23, 1912, by adding a provision as follows: children may continue the proceedings. The decision rendered in the case shall, so far
as the widow and minor children are concerned, produce the same legal effect as if it
Provided, That the Philippine Legislature is hereby authorized to provide by law for the had been rendered during the life of the petitioner.
acquisition of Philippine citizenship by those natives of the Philippine Islands who do
43
It was not until November 30, 1928, upon the approval of Act 3448, amending Act that Justice Regala reasoned out why the possession of the qualifications provided by
2977, that the following provisions were added to the above Section 13: the law should also be shown to be possessed by the alien wife of a Filipino, for her to
become a Filipina by marriage.chanroblesvirtualawlibrarychanrobles virtual law library
SECTION 1. The following new sections are hereby inserted between sections thirteen
and fourteen of Act Numbered Twenty-nine hundred and Twenty-seven: As may be recalled, the basic argument advanced by Justice Regala was briefly as
follows: That "like the law in the United States, our Naturalization Law specified the
SEC. 13(a). Any woman who is now or may hereafter be married to a citizen of the classes of persons who alone might become citizens, even as it provided who were
Philippine Islands and who might herself be lawfully naturalized, shall be deemed a disqualified," and inasmuch as Commonwealth Act 473, our Naturalization Law since
citizen of the Philippine Islands.chanroblesvirtualawlibrarychanrobles virtual law 1939 did not reenact the section providing who might become citizens, allegedly in
library order to remove racial discrimination in favor of Caucasians and against Asiatics, "the
only logical deduction ... is that the phrase "who might herself be lawfully naturalized"
SEC. 13(b). Children of persons who have been duly naturalized under this law, being must now be understood as referring to those who under Section 2 of the law are
under the age of twenty-one years at the time of the naturalization of their parents, qualified to become citizens of the Philippines" and "there is simply no support for the
shall, if dwelling in the Philippine Islands, be considered citizens view that the phrase "who might herself be lawfully naturalized" must now be
thereof.chanroblesvirtualawlibrarychanrobles virtual law library understood as requiring merely that the alien woman must not belong to the class of
disqualified persons under Section 4 of the Revised Naturalization Law." 14chanrobles
SEC. 13(c). Children of persons naturalized under this law who have been born in the virtual law library
Philippine Islands after the naturalization of their parents shall be considered citizens
thereof. A similar line of reasoning was followed in Choy King Tee, which for ready reference
may be qouted:
When Commonwealth Act 473, the current naturalization law, was enacted on June 17,
1939, the above Section 13 became its Section 15 which has already been quoted The question has been settled by the uniform ruling of this Court in a number of cases.
earlier in this decision. As can be seen, Section 13 (a) abovequoted was re-enacted The alien wife of a Filipino citizen must first prove that she has all the qualifications
practically word for word in the first paragraph of this Section 15 except for the change required by Section 2 and none of the disqualifications enumerated in Section 4 of the
of Philippine Islands to Philippines. And it could not have been on any other basis than Naturalization Law before she may be deemed a Philippine citizen (Lao Chay v.
this legislative history of our naturalization law that each and everyone of the decisions Galang, L-190977, Oct. 30, 1964, citing Lo San Tuang v. Galang, L-18775, Nov. 30,
of this Court from the first Ly Giok Ha to Go Im Ty, discussed above, were 1963; Sun Peck Yong v. Commissioner of Immigration, L-20784, December 27, 1963;
rendered.chanroblesvirtualawlibrarychanrobles virtual law library Tong Siok Sy v. Vivo, L-21136, December 27, 1963). The writer of this opinion has
submitted the question anew to the court for a possible reexamination of the said ruling
As stated earlier, in the opinion of Chief Justice Concepcion in the first Ly Giok Ha, it in the light of the interpretation of a similar law in the United States after which Section
was quite clear that for an alien woman who marries a Filipino to become herself a 15 of our Naturalization Law was patterned. That law was section 2 of the Act of
Filipino citizen, there is no need for any naturalization proceeding because she February 10, 1855 (Section 1994 of the Revised Statutes of the U.S.). The local law,
becomes a Filipina ipso facto from the time of such marriage, provided she does not Act No. 3448, was passed on November 30, 1928 as an amendment to the former
suffer any of the disqualifications enumerated in Section 4 of Commonwealth Act 473, Philippine Naturalization Law, Act No. 2927, which was approved on March 26, 1920.
with no mention being made of whether or not the qualifications enumerated in Section Under this Naturalization Law, acquisition of Philippine citizenship was limited to
2 thereof need be shown. It was only in Lee Suan Ay in 1959 that the possession of three classes of persons, (a) Natives of the Philippines who were not citizens thereof;
qualifications were specifically required, but it was not until 1963, in Lo San Tuang, (b) natives of the other insular possessions of the United States; and (c) citizens of the
44
United States, or foreigners who, under the laws of the United States, may become On cross-examination, she (Ly Giok Ha) failed to establish that: (1) she has been
citizens of the latter country if residing therein. The reference in subdivision (c) to residing in the Philippines for a continuous period of at least (10) years (p. 27,
foreigners who may become American Citizens is restrictive in character, for only t.s.n., id.); (2) she has a lucrative trade, profession, or lawful occupation (p. 13,
persons of certain specified races were qualified thereunder. In other words, in so far as t.s.n., id.); and (3) she can speak and write English, or any of the principal Philippine
racial restrictions were concerned there was at the time a similarity between the languages (pp. 12, 13, t.s.n., id.).chanroblesvirtualawlibrarychanrobles virtual law
naturalization laws of the two countries and hence there was reason to accord here library
persuasive force to the interpretation given in the United States to the statutory
provision concerning the citizenship of alien women marrying American While the appellant Immigration Commissioner contends that the words emphasized
citizens.chanroblesvirtualawlibrarychanrobles virtual law library indicate that the present Naturalization Law requires that an alien woman who marries
a Filipino husband must possess the qualifications prescribed by section 2 in addition to
This Court, however, believes that such reason has ceased to exist since the enactment not being disqualified under any of the eight ("a" to "h") subheadings of section 4 of
of the Revised Naturalization Law, (Commonwealth Act No. 473) on June 17, 1939. Commonwealth Act No. 473, in order to claim our citizenship by marriage, both the
The racial restrictions have been eliminated in this Act, but the provision found in Act appellee and the court below (in its second decision) sustain the view that all that the
No. 3448 has been maintained. It is logical to presume that when Congress chose to law demands is that the woman be not disqualified under section
retain the said provision - that to be deemed a Philippine citizen upon marriage the 4.chanroblesvirtualawlibrarychanrobles virtual law library
alien wife must be one "who might herself be lawfully naturalized," the reference is no
longer to the class or race to which the woman belongs, for class or race has become At the time the present case was remanded to the court of origin (1960) the question at
immaterial, but to the qualifications and disqualifications for naturalization as issue could be regarded as not conclusively settled, there being only the concise
enumerated in Sections 2 and 4 of the statute. Otherwise the requirement that the pronouncement in Lee Suan Ay, et al. v. Galang, G. R. No. L-11855, Dec. 23, 1959, to
woman "might herself be lawfully naturalized" would be meaningless surplusage, the effect that:
contrary to settled norms of statutory
construction.chanroblesvirtualawlibrarychanrobles virtual law library The marriage of a Filipino citizen to an alien does not automatically confer Philippine
citizenship upon the latter. She must possess the qualifications required by law to
The rule laid down by this Court in this and in other cases heretofore decided is become a Filipino citizen by naturalization.
believed to be in line with the national policy of selective admission to Philippine
citizenship, which after all is a privilege granted only to those who are found worthy Since that time, however, a long line of decisions of this Court has firmly established
thereof, and not indiscriminately to anybody at all on the basis alone of marriage to a the rule that the requirement of section 15 of Commonwealth Act 473 (the
man who is a citizen of the Philippines, irrespective of moral character, ideological Naturalization Act), that an alien woman married to a citizen should be one who "might
beliefs, and identification with Filipino ideals, customs and herself be lawfully naturalized," means not only woman free from the disqualifications
traditions.chanroblesvirtualawlibrarychanrobles virtual law library enumerated in section 4 of the Act but also one who possesses the qualifications
prescribed by section 2 of Commonwealth Act 473 (San Tuan v. Galang, L-18775, Nov.
Appellee here having failed to prove that she has all the qualifications for 30, 1963; Sun Peck Yong v. Com. of Immigration, L-20784, Dee. 27, 1963; Tong Siok
naturalization, even, indeed, that she has none of the disqualifications, she is not Sy v. Vivo, L-21136, Dec. 27, 1963; Austria v. Conchu, L-20716, June 22, 1965; Choy
entitled to recognition as a Philippine citizen. King Tee v. Galang, L-18351, March 26, 1965; Brito v. Com. of Immigration, L-16829,
June 30, 1965).chanroblesvirtualawlibrarychanrobles virtual law library
In the second Ly Giok Ha, the Court further fortified the arguments in favor of the
same conclusion thus: Reflection will reveal why this must be so. The qualifications prescribed under section
45
2 of the Naturalization Act, and the disqualifications enumerated in its section 4 are not Galang, L-18351, March 26, 1965.
mutually exclusive; and if all that were to be required is that the wife of a Filipino be
not disqualified under section 4, the result might well be that citizenship would be It is difficult to minimize the persuasive force of the foregoing rationalizations, but a
conferred upon persons in violation of the policy of the statute. For example, section 4 closer study thereof cannot bat reveal certain relevant considerations which adversely
disqualifies only -chanrobles virtual law library affect the premises on which they are predicated, thus rendering the conclusions arrived
thereby not entirely unassailable.chanroblesvirtualawlibrarychanrobles virtual law
(c) Polygamists or believers in the practice of polygamy; andchanrobles virtual law library
library
1. The main proposition, for instance, that in eliminating Section 1 of Act 2927
(d) Persons convicted of crimes involving moral turpitude,chanrobles virtual law providing who are eligible for Philippine citizenship, the purpose of Commonwealth
library Act 473, the Revised Naturalization Law, was to remove the racial requirements for
naturalization, thereby opening the door of Filipino nationality to Asiatics instead of
so that a blackmailer, or a maintainer of gambling or bawdy houses, not previously allowing the admission thereto of Caucasians only, suffers from lack of exact accuracy.
convicted by a competent court would not be thereby disqualified; still, it is certain that It is important to note, to start with, that Commonwealth Act 473 did away with the
the law did not intend such person to be admitted as a citizen in view of the whole Section 1 of Act 2927 which reads, thus:
requirement of section 2 that an applicant for citizenship "must be of good moral
character."chanrobles virtual law library SECTION 1. Who may become Philippine citizens. - Philippine citizenship may be
acquired by: (a) natives of the Philippines who are not citizens thereof under the Jones
Similarly, the citizen's wife might be a convinced believer in racial supremacy, in Law; (b) natives of the other Insular possessions of the United States; (c) citizens of the
government by certain selected classes, in the right to vote exclusively by certain United States, or foreigners who under the laws of the United States may become
"herrenvolk", and thus disbelieve in the principles underlying the Philippine citizens of said country if residing therein.
Constitution; yet she would not be disqualified under section 4, as long as she is not
"opposed to organized government," nor affiliated to groups "upholding or teaching and not only subdivision (c) thereof. Nowhere in this whole provision was there any
doctrines opposing all organized governments", nor "defending or teaching the mention of race or color of the persons who were then eligible for Philippine
necessity or propriety of violence, personal assault or assassination for the success or citizenship. What is more evident from said provision is that it reflected the inevitable
predominance of their ideas." Et sic de caeteris.chanroblesvirtualawlibrarychanrobles subordination of our legislation during the pre-Commonwealth American regime to the
virtual law library understandable stations flowing from our staffs as a territory of the United States by
virtue of the Treaty of Paris. In fact, Section 1 of Act 2927 was precisely approved
The foregoing instances should suffice to illustrate the danger of relying exclusively on pursuant to express authority without which it could not have been done, granted by an
the absence of disqualifications, without taking into account the other affirmative amendment to Section 4 of the Philippine Bill of 1902 introduced by the Act of the
requirements of the law, which, in the case at bar, the appellee Ly Giok Ha admittedly United States Congress of March 23, 1912 and which was reenacted as part of the
does not possess.chanroblesvirtualawlibrarychanrobles virtual law library Jones Law of 1916, the pertinent provisions of which have already been footed earlier.
In truth, therefore, it was because of the establishment of the Philippine
As to the argument that the phrase "might herself be lawfully naturalized" was derived Commonwealth and in the exercise of our legislative autonomy on citizenship matters
from the U.S. Revised Statutes (section 1994) and should be given the same territorial under the Philippine Independence Act that Section 1 of Act 2927 was eliminated, 15and
and racial significance given to it by American courts, this Court has rejected the same not purposely to eliminate any racial discrimination contained in our Naturalization
in Lon San Tuang v. Galang, L-18775, November 30, 1963; and in Choy King Tee v. Law. The Philippine Legislature naturally wished to free our Naturalization Law from
46
the impositions of American legislation. In other words, the fact that such predicated on the theory that the elimination of Section 1 of Act 2927 by
discrimination was removed was one of the effects rather than the intended purpose of Commonwealth Act 473 was purposely for no other end than the abolition of racial
the amendment.chanroblesvirtualawlibrarychanrobles virtual law library discrimination in our naturalization law has no clear factual basis. 17chanrobles virtual
law library
2. Again, the statement in Choy King Tee to the effect that "the reference in subdivision
(c) (of Section 1 of Act 2927) to foreigners who may become American citizens is 3. In view of these considerations, there appears to be no cogent reason why the
restrictive in character, for only persons of certain specified races were qualified construction adopted in the opinions of the Secretary of Justice referred to in the first
thereunder" fails to consider the exact import of the said subdivision. Explicitly, the Ly Giok Ha decision of the Chief Justice should not prevail. It is beyond dispute that
thrust of the said subdivision was to confine the grant under it of Philippine citizenship the first paragraph of Section 15 of Commonwealth Act 473 is a reenactment of Section
only to the three classes of persons therein mentioned, the third of which were citizens 13(a) of Act 2927, as amended by Act 3448, and that the latter is nothing but an exact
of the United States and, corollarily, persons who could be American citizens under her copy, deliberately made, of Section 1994 of the Raised Statutes of the United States as
laws. The words used in the provision do not convey any idea of favoring aliens of any it stood before its repeal in 1922. 18Before such repeal, the phrase "who might herself
particular race or color and of excluding others, but more accurately, they refer to all be lawfully naturalized" found in said Section 15 had a definite unmistakable
the disqualifications of foreigners for American citizenship under the laws of the construction uniformly foIlowed in all courts of the United States that had occasion to
United States. The fact is that even as of 1906, or long before 1920, when our Act 2927 apply the same and which, therefore, must be considered, as if it were written in the
became a law, the naturalization, laws of the United States already provided for the statute itself. It is almost trite to say that when our legislators enacted said section, they
following disqualifications in the Act of the Congress of June 29, 1906: knew of its unvarying construction in the United States and that, therefore, in adopting
verbatim the American statute, they have in effect incorporated into the provision, as
SEC. 7. That no person who disbelieves in or who is opposed to organized government, thus enacted, the construction given to it by the American courts as well as the Attorney
or who is a member of or affiliated with any organization entertaining and teaching General of the United States and all administrative authorities, charged with the
such disbelief in or opposition to organized government, or who advocates or teaches implementation of the naturalization and immigration laws of that country. (Lo Cham v.
the duty, necessity, or propriety of the unlawful assaulting or killing of any officer or Ocampo, 77 Phil., 635 [1946]; Laxamana v. Baltazar, 92 Phil., 32 [1952]; Hartley v.
officers, either of specific individuals or of officers generally, of the Government of the Commissioner, 295 U.S. 216, 79 L. ed. 1399, 55 S Ct. 756 [19353; Helvering v.
United States, or of any other organized government, because of his or their official Winmill, 305 U.S. 79, 83 L ed. 52, 59 S Ct. 45 [1938]; Helvering v. R. J. Reynolds
character, or who is a polygamist, shall be naturalized or be made a citizen of the Tobacco Co., 306 U.S. 110, 83 L ed. 536, 59 S Ct. 423 [1939]. [p. 32, Memo of Amicus
United States. Curiae]).chanroblesvirtualawlibrarychanrobles virtual law library

and all these disqualified persons were, therefore, ineligible for Philippine citizenship A fairly comprehensive summary of the said construction by the American courts and
under Section 1 of Act 2927 even if they happened to be Caucasians. More importantly, administrative authorities is contained in United States of America ex rel. Dora
as a matter of fact, said American law, which was the first "Act to Establish a Bureau of Sejnensky v. Robert E. Tod, Commissioner of Immigration, Appt., 295 Fed. 523,
Immigration and Naturalization and to provide for a Uniform Rule for Naturalization of decided November 14, 1922, 26 A. L. R. 1316 as follows:
Aliens throughout the United States" contained no racial disqualification requirement,
except as to Chinese, the Act of May 6, 1882 not being among the expressly repealed Section 1994 of the Revised Statutes (Comp. Stat. 3948, 2 Fed. Sta. Anno. 2d ed. p.
by this law, hence it is clear that when Act 2927 was enacted, subdivision (e) of its 117) provides as follows: "Any woman who is now or may hereafter be married to a
Section 1 could not have had any connotation of racial exclusion necessarily, even if it citizen of the United States, and who might herself be lawfully naturalized, shall be
were traced back to its origin in the Act of the United States Congress of 1912 already deemed a citizen."chanrobles virtual law library
mentioned above. 16Thus, it would seem that the rationalization in the qouted decisions
47
Section 1944 of the Revised Stat. is said to originate in the Act of Congress of February class or race which might be lawfully naturalized, and did not refer to any of the other
10, 1855 (10 Stat. at L. 604, chap. 71), which in its second section provided "that any provisions of the naturalization laws as to residence or moral character, or to any of the
woman, who might lawfully be naturalized under the existing laws, married, or who provisions of the immigration laws relating to the exclusion or deportation of
shall be married to a citizen of the United States, shall be deemed and taken to be a aliens.chanroblesvirtualawlibrarychanrobles virtual law library
citizen."chanrobles virtual law library
In 1880, in Leonard v. Grant (C. C.) 5 Fed. 11, District Judge Deady also construed the
And the American Statute of 1855 is substantially a copy of the earlier British Statute 7 Act of 1855, declaring that "any woman who is now or may hereafter be married to a
& 8 Vict. chap. 66, s 16, 1844, which provided that "any woman married, or who shall citizen of the United States, and might herself be lawfully naturalized, shall be deemed
be married, to a natural-born subject or person naturalized, shall be deemed and taken a citizen." He held that "upon the authorities, and the reason, if not the necessity, of the
to be herself naturalized, and have all the rights and privileges of a natural born case," the statute must be construed as in effect declaring that an alien woman, who is
subject."chanrobles virtual law library of the class or race that may be lawfully naturalized under the existing laws, and who
marries a citizen of the United States, is such a citizen also, and it was not necessary
The Act of Congress of September 22, 1922 (42 Stat. at L. 1021, chap. 411, Comp. that it should appear affirmatively that she possessed the other qualifications at the time
Stat. 4358b, Fed. Stat. Anno. Supp. 1922, p. 255), being "An Act Relative to the of her marriage to entitle her to naturalization.chanroblesvirtualawlibrarychanrobles
Naturalization and Citizenship of Married Women," in 2, provides "that any woman virtual law library
who marries a citizen of the United States after the passage of this Act, ... shall not
become a citizen of the United States by reason of such marriage ..."chanrobles virtual In 1882, the Act of 1855 came before Mr. Justice Harlan, sitting in the circuit court,
law library in United States v. Kellar, 13 Fed. 82. An alien woman, a subject of Prussia came to the
United States and married here a naturalized citizen. Mr. Justice Harlan, with the
Section 6 of the act also provides "that 1994 of the Revised Statutes ... are concurrence of Judge Treat, held that upon her marriage she became ipso facto a citizen
repealed." chanrobles virtual law library of the United States as fully as if she had complied with all of the provisions of the
statutes upon the subject of naturalization. He added: "There can be no doubt of this, in
Section 6 also provides that `such repeal shall not terminate citizenship acquired or view of the decision of the Supreme Court of the United, States in Kelly v. Owen, 7
retained under either of such sections, ..." meaning 2 and 6. So that this Act of Wall. 496, 19 L. ed. 283." The alien "belonged to the class of persons" who might be
September 22, 1922, has no application to the facts of the present case, as the marriage lawfully naturalized.chanroblesvirtualawlibrarychanrobles virtual law library
of the relator took place prior to its passage. This case, therefore, depends upon the
meaning to be attached to 1994 of the Revised In 1904, in Hopkins v. Fachant, 65 C. C. A. 1, 130 Fed. 839, an alien woman came to
Statutes.chanroblesvirtualawlibrarychanrobles virtual law library the United States from France and entered the country contrary to the immigration
laws. The immigration authorities took her into custody at the port of New York, with
In 1868 the Supreme Court, in Kelly v. Owen, 7 Wall. 496, 498, 19 L. ed. 283, 284, the view of deporting her. She applied for her release under a writ of habeas corpus,
construed this provision as found in the Act of 1855 as follows: "The term, "who might and pending the disposition of the matter she married a naturalized American citizen.
lawfully be naturalized under the existing laws," only limits the application of the law The circuit court of appeals for the ninth Circuit held, affirming the court below, that
to free white women. The previous Naturalization Act, existing at the time, only she was entitled to be discharged from custody. The court declared: "The rule is well
required that the person applying for its benefits should be "a free white person," and settled that her marriage to a naturalized citizen of the United States entitled her to be
not an alien enemy."chanrobles virtual law library discharged. The status of the wife follows that of her husband, ... and by virtue of her
marriage her husband's domicil became her
This construction limited the effect of the statute to those aliens who belonged to the domicil." .chanroblesvirtualawlibrarychanrobles virtual law library
48
In 1908, the circuit court for the district of Rhode Island in Re Rustigian, 165. Fed. that existed at the time the order of exclusion was made. If the circumstances change
980, had before it the application of a husband for his final decree of naturalization. It prior to the order being carried into effect, it cannot be executed. For example, if an
appeared that at that time his wife was held by the immigration authorities at New York order of exclusion should be based on the ground that the alien was at the time afflicted
on the ground that she was afflicted with a dangerous and contagious disease. Counsel with a contagious disease, and it should be made satisfactorily to appear, prior to actual
on both sides agreed that the effect of the husband's naturalization would be to confer deportation, that the alien had entirely recovered from the disease, we think it plain that
citizenship upon the wife. In view of that contingency District Judge Brown declined to the order could not be carried into effect. So, in this case, if, after the making of the
pass upon the husband's application for naturalization, and thought it best to wait until order of exclusion and while she is permitted temporarily to remain, she in good faith
it was determined whether the wife's disease was curable. He placed his failure to act marries an American citizen, we cannot doubt the validity of her marriage, and that she
on the express ground that the effect of naturalizing the husband might naturalize her. thereby acquired, under international law and under 1994 of the Revised Statutes,
At the same time he express his opinion that the husband's naturalization would not American citizenship, and ceased to be an alien. There upon, the immigration
effect her naturalization, as she was not one who could become lawfully naturalized. authorities lost their jurisdiction over her, as that jurisdiction applies only to aliens, and
"Her own capacity (to become naturalized)," the court stated "is a prerequisite to her not to citizens.chanroblesvirtualawlibrarychanrobles virtual law library
attaining citizenship. If herself lacking in that capacity, the married status cannot confer
it upon her." Nothing, however, was actually decided in that case, and the views In 1910, District Judge Dodge, in Ex parte Kaprielian, 188 Fed. 694, sustained the right
expressed therein are really nothing more than mere dicta. But, if they can be regarded of the officials to deport a woman under the following circumstances: She entered this
as something more than that, we find ourselves, with all due respect for the learned country in July, 1910, being an alien and having been born in Turkey. She was taken
judge, unable to accept them.chanroblesvirtualawlibrarychanrobles virtual law library into custody by the immigration authorities in the following September, and in October
a warrant for her deportation was issued. Pending hearings as to the validity of that
In 1909, in United States ex rel. Nicola v. Williams, 173 Fed. 626, District Judge order, she was paroled in the custody of her counsel. The ground alleged for her
Learned Hand held that an alien woman, a subject of the Turkish Empire, who married deportation was that she was afflicted with a dangerous and contagious disease at the
an American citizen while visiting Turkey, and then came to the United States, could time of her entry. One of the reasons assigned to defeat deportation was that the woman
not be excluded, although she had, at the time of her entry, a disease which under the had married a citizen of the United States pending the proceedings for her deportation.
immigration laws would have been sufficient ground for her exclusion, if she bad not Judge Dodge declared himself unable to believe that a marriage under such
had the status of a citizen. The case was brought into this court on appeal, and in 1911 circumstances "is capable of having the effect claimed, in view of the facts shown." He
was affirmed, in 106 C. C. A. 464, 184 Fed. 322. In that case, however at the time the held that it was no part of the intended policy of 1994 to annul or override the
relators married, they might have been lawfully naturalized, and we said: "Even if we immigration laws, so as to authorize the admission into the country of the wife of a
assume the contention of the district attorney to be correct that marriage will not make naturalized alien not otherwise entitled to enter, and that an alien woman, who is of a
a citizen of a woman who would be excluded under our immigration laws, it does not class of persons excluded by law from admission to the United States does not come
affect these relators."chanrobles virtual law library within the provisions of that section. The court relied wholly upon the dicta contained
in the Rustigian Case. No other authorities were
We held that, being citizens, they could not be excluded as aliens; and it was also said cited.chanroblesvirtualawlibrarychanrobles virtual law library
to be inconsistent with the policy of our law that the husband should be a citizen and
the wife an alien. The distinction between that case and the one now before the court is In 1914, District Judge Neterer, in Ex parte Grayson, 215 Fed. 449, construed 1994 and
that, in the former case, the marriage took place before any order of exclusion had been held that where, pending proceedings to deport an alien native of France as an alien
made, while in this the marriage was celebrated after such an order was made. But such prostitute, she was married to a citizen of the United States, she thereby became a
an order is a mere administrative provision, and has not the force of a judgment of a citizen, and was not subject to deportation until her citizenship was revoked by due
court, and works no estoppel. The administrative order is based on the circumstances process of law. It was his opinion that if, as was contended, her marriage was conceived
49
in fraud, and was entered into for the purpose of evading the immigration laws and Before concluding this opinion, we may add that it has not escaped our observation that
preventing her deportation, such fact should be established in a court of competent Congress, in enacting the Immigration Act of 1917, so as to provide, in 19, "that the
jurisdiction in an action commenced for the purpose. The case was appealed and the marriage to an American citizen of a female of the sexually immoral classes ... shall not
appeal was dismissed. 134 C. C. A. 666, 219 Fed. invest such female with United States citizenship if the marriage of such alien female
1022.chanroblesvirtualawlibrarychanrobles virtual law library shall be solemnized after her arrest or after the commission of acts which make her
liable to deportation under this act."chanrobles virtual law library
It is interesting also to observe the construction placed upon the language of the statute
by the Department of Justice. In 1874, Attorney General Williams, 14 Ops. Atty. Gen. Two conclusions seem irresistibly to follow from the above change in the
402, passing upon the Act of February 10, 1855, held that residence within the United law:chanrobles virtual law library
States for the period required by the naturalization laws was riot necessary in order to
constitute an alien woman a citizen, she having married a citizen of the United States (1) Congress deemed legislation essential to prevent women of the immoral class
abroad, although she never resided in the United States, she and her husband having avoiding deportation through the device of marrying an American
continued to reside abroad after the marriage.chanroblesvirtualawlibrarychanrobles citizen.chanroblesvirtualawlibrarychanrobles virtual law library
virtual law library
(2) If Congress intended that the marriage of an American citizen with an alien woman
In 1909, a similar construction was given to the Immigration Act of May 5, 1907, in an of any other of the excluded classes, either before or after her detention, should not
opinion rendered by Attorney General Wickersham. It appeared an unmarried woman, confer upon her American citizenship, thereby entitling her to enter the country, its
twenty-eight years of age and a native of Belgium, arrived in New York and went at intention would have been expressed, and 19 would not have been confined solely to
once to a town in Nebraska, where she continued to reside. About fifteen months after women of the immoral class.
her arrival she was taken before a United States commissioner by way of instituting
proceedings under the Immigration Act (34 Stat. at L. 898, chap. 1134, Comp. Stat. Indeed, We have examined all the leading American decisions on the subject and We
4242, 3 Fed. Stat. Anno. 2d ed. p. 637) for her deportation, on the ground that she had have found no warrant for the proposition that the phrase "who might herself be
entered this country for the purpose of prostitution, and had been found an inmate of a lawfully naturalized" in Section 1994 of the Revised Statutes was meant solely as a
house of prostitution and practicing the same within three years after landing. It racial bar, even if loose statements in some decisions and other treaties and other
appeared, however, that after she was taken before the United States commissioner, but writings on the subject would seem to give such impression. The case of Kelley v.
prior to her arrest under a warrant by the Department of Justice, she was lawfully Owen, supra, which appears to be the most cited among the first of the
married to a native-born citizen of the United States. The woman professed at the time decisions 19simply held:
of her marriage an intention to abandon her previous mode of life and to remove with
her husband to his home in Pennsylvania. He knew what her mode of life had been, but As we construe this Act, it confers the privileges of citizenship upon women married to
professed to believe in her good intentions. The question was raised as to the right to citizens of the United States, if they are of the class of persons for whose naturalization
deport her, the claim being advance that by her marriage she bad become an American the previous Acts of Congress provide. The terms "married" or "who shall be married,"
citizen and therefore could not be deported. The Attorney General ruled against the do not refer in our judgment, to the time when the ceremony of marriage is celebrated,
right to deport her as she had become an American citizen. He held that the words, but to a state of marriage. They mean that, whenever a woman, who under previous
"who might herself be lawfully naturalized," refer to a class or race who might be Acts might be naturalized, is in a state of marriage to a citizen, whether his citizenship
lawfully naturalized, and that compliance with the other conditions of the naturalization existed at the passage of the Act or subsequently, or before or after the marriage, she
laws was not required. 27 Ops. Atty. Gen. 507.chanroblesvirtualawlibrarychanrobles becomes, by that fact, a citizen also. His citizenship, whenever it exists, confers, under
virtual law library the Act, citizenship upon her. The construction which would restrict the Act to women
50
whose husbands, at the time of marriage, are citizens, would exclude far the greater 4. As already stated, in Lo San Tuang, Choy King Tee and the second Ly Giok Ha, the
number, for whose benefit, as we think, the Act was intended. Its object, in our opinion, Court drew the evidence that because Section 1 of Act 2927 was eliminated by
was to allow her citizenship to follow that of her husband, without the necessity of any Commonwealth Act 473, it follows that in place of the said eliminated section
application for naturalization on her part; and, if this was the object, there is no reason particularly its subdivision (c), being the criterion of whether or not an alien wife "may
for the restriction suggested.chanroblesvirtualawlibrarychanrobles virtual law library be lawfully naturalized," what should be required is not only that she must not be
disqualified under Section 4 but that she must also possess the qualifications
The terms, "who might lawfully be naturalized under the existing laws," only limit the enumerated in Section 2, such as those of age, residence, good moral character,
application of the law to free white women. The previous Naturalization Act, existing adherence to the underlying principles of the Philippine Constitution, irreproachable
at the time only required that the person applying for its benefits should be "a free conduct, lucrative employment or ownership of real estate, capacity to speak and write
white person," and not an alien enemy. Act of April 14th, 1802, 2 Stat. at L. English or Spanish and one of the principal local languages, education of children in
153.chanroblesvirtualawlibrarychanrobles virtual law library certain schools, etc., thereby implying that, in effect, sails Section 2 has been purposely
intended to take the place of Section 1 of Act 2927. Upon further consideration of the
A similar construction was given to the Act by the Court of Appeals of New York, proper premises, We have come, to the conclusion that such inference is not sufficiently
in Burton v. Burton, 40 N. Y. 373; and is the one which gives the widest extension to its justified.chanroblesvirtualawlibrarychanrobles virtual law library
provisions.
To begin with, nothing extant in the legislative history, which We have already
Note that write the court did say that "the terms, "who might lawfully be naturalized explained above of the mentioned provisions has been shown or can be shown to
under existing laws" only limit the application to free white women" 20it hastened to indicate that such was the clear intent of the legislature. Rather, what is definite is that
add that "the previous Naturalization Act, existing at the time, ... required that the Section 15 is, an exact copy of Section 1994 of the Revised Statutes of the United
person applying for its benefits should be (not only) a "free white person" (but also) ... States, which, at the time of the approval of Commonwealth Act 473 had already a
not an alien enemy." This is simply because under the Naturalization Law of the United settled construction by American courts and administrative
States at the time the case was decided, the disqualification of enemy aliens had already authorities.chanroblesvirtualawlibrarychanrobles virtual law library
been removed by the Act of July 30, 1813, as may be seen in the corresponding
footnote hereof anon. In other words, if in the case of Kelly v. Owen only the race Secondly, as may be gleaned from the summary of pertinent American decisions quoted
requirement was mentioned, the reason was that there was no other non-racial above, there can be no doubt that in the construction of the identically worded
requirement or no more alien enemy disqualification at the time; and this is provision in the Revised Statutes of the United States, (Section 1994, which was taken,
demonstrated by the fact that the court took care to make it clear that under the from the Act of February 10, 1855) all authorities in the United States are unanimously
previous naturalization law, there was also such requirement in addition to race. This is agreed that the qualifications of residence, good moral character, adherence to the
impotent, since as stated in re Rustigian, 165 Fed. Rep. 980, "The expression used by Constitution, etc. are not supposed to be considered, and that the only eligibility to be
Mr. Justice Field, (in Kelly v. Owen) the terms "who might lawfully be naturalized taken into account is that of the race or class to which the subject belongs, the
under existing laws" only limit the application of the law to free white women, must be conceptual scope of which, We have just discussed. 21In the very case of Leonard v.
interpreted in the application to the special facts and to the incapacities under the then Grant, supra, discussed by Justice Regala in Lo San Tuang, the explanation for such
existing laws," (at p. 982) meaning that whether or not an alien wife marrying a citizen posture of the American authorities was made thus:
would be a citizen was dependent, not only on her race and nothing more necessarily,
but on whether or not there were other disqualifications under the law in force at the The phrase, "shall be deemed a citizen" in section 1994 Rev. St., or as it was in the Act
time of her marriage or the naturalization of her of 1855, supra, "shall be deemed and taken to be a citizen" while it may imply that the
husband.chanroblesvirtualawlibrarychanrobles virtual law library person to whom it relates has not actually become a citizen by ordinary means or in the
51
usual way, as by the judgment of a competent court, upon a proper application and Fourthly, it is difficult to conceive that the phrase "who might be lawfully naturalized"
proof, yet it does not follow that such person is on that account practically any the less in Section 15 could have been intended to convey a meaning different than that given
a citizen. The word "deemed" is the equivalent of "considered" or "judged"; and, to it by the American courts and administrative authorities. As already stated, Act 3448
therefore, whatever an act of Congress requires to be "deemed" or "taken" as true of which contained said phrase and from which it was taken by Commonwealth Act 473,
any person or thing, must, in law, be considered as having been duly adjudged or was enacted in 1928. By that, time, Section 1994 of the Revised Statutes of the United
established concerning "such person or thing, and have force and effect accordingly. States was no longer in force because it had been repealed expressly the Act of
When, therefore, Congress declares that an alien woman shall, under certain September 22, 1922 which did away with the automatic naturalization of alien wives of
circumstances, be "deemed' an American citizen, the effect when the contingency American citizens and required, instead, that they submit to regular naturalization
occurs, is equivalent to her being naturalized directly by an act of Congress, or in the proceedings, albeit under more liberal terms than those of other applicants. In other
usual mode thereby prescribed. words, when our legislature adopted the phrase in question, which, as already
demonstrated, had a definite construction in American law, the Americans had already
Unless We disregard now the long settled familiar rule of statutory construction that in abandoned said phraseology in favor of a categorical compulsion for alien wives to be
a situation like this wherein our legislature has copied an American statute word for natural judicially. Simple logic would seem to dictate that, since our lawmakers, at the
word, it is understood that the construction already given to such statute before its time of the approval of Act 3448, had two choices, one to adopt the phraseology of
being copied constitute part of our own law, there seems to be no reason how We can Section 1994 with its settled construction and the other to follow the new posture of the
give a different connotation or meaning to the provision in question. At least, We have Americans of requiring judicial naturalization and it appears that they have opted for
already seen that the views sustaining the contrary conclusion appear to be based on in the first, We have no alternative but to conclude that our law still follows the old or
accurate factual premises related to the real legislative background of the framing of previous American Law On the subject. Indeed, when Commonwealth Act 473 was
our naturalization law in its present form.chanroblesvirtualawlibrarychanrobles virtual approved in 1939, the Philippine Legislature, already autonomous then from the
law library American Congress, had a clearer chance to disregard the old American law and make
one of our own, or, at least, follow the trend of the Act of the U.S. Congress of 1922,
Thirdly, the idea of equating the qualifications enumerated in Section 2 of but still, our legislators chose to maintain the language of the old law. What then is
Commonwealth Act 473 with the eligibility requirements of Section 1 of Act 2927 significantly important is not that the legislature maintained said phraseology after
cannot bear close scrutiny from any point of view. There is no question that Section 2 Section 1 of Act 2927 was eliminated, but that it continued insisting on using it even
of Commonwealth Act 473 is more or less substantially the same as Section 3 of Act after the Americans had amended their law in order to provide for what is now
2927. In other words, Section 1 of Act 2927 co-existed already with practically the contended to be the construction that should be given to the phrase in question. Stated
same provision as Section 2 of Commonwealth Act 473. If it were true that the phrase differently, had our legislature adopted a phrase from an American statute before the
"who may be lawfully naturalized" in Section 13 (a) of Act 2927, as amended by Act American courts had given it a construction which was acquiesced to by those given
3448, referred to the so-called racial requirement in Section 1 of the same Act, without upon to apply the same, it would be possible for Us to adopt a construction here
regard to the provisions of Section 3 thereof, how could the elimination of Section 1 different from that of the Americans, but as things stand, the fact is that our legislature
have the effect of shifting the reference to Section 3, when precisely, according to the borrowed the phrase when there was already a settled construction thereof, and what is
American jurisprudence, which was prevailing at the time Commonwealth Act 473 was more, it appears that our legislators even ignored the modification of the American law
approved, such qualifications as were embodied in said Section 3, which had their and persisted in maintaining the old phraseology. Under these circumstances, it would
counterpart in the corresponding American statutes, are not supposed to be taken into be in defiance of reason and the principles of Statutory construction to say that Section
account and that what should be considered only are the requirements similar to those 15 has a nationalistic and selective orientation and that it should be construed
provided for in said Section 1 together with the disqualifications enumerated in Section independently of the previous American posture because of the difference of
4?chanrobles virtual law library circumstances here and in the United States. It is always safe to say that in the
52
construction of a statute, We cannot fall on possible judicial fiat or perspective when qualifications and, as already demonstrated, in American jurisprudence, qualifications
the demonstrated legislative point of view seems to indicate had never been considered to be of any relevance in determining "who might be
otherwise.chanroblesvirtualawlibrarychanrobles virtual law library lawfully naturalized," as such phrase is used in the statute governing the status of alien
wives of American citizens, and our law on the matter was merely copied verbatim
5. Viewing the matter from another angle, there is need to emphasize that in reality and from the American statutes.chanroblesvirtualawlibrarychanrobles virtual law library
in effect, the so called racial requirements, whether under the American laws or the
Philippine laws, have hardly been considered as qualifications in the same sense as 6. In addition to these arguments based on the applicable legal provisions and judicial
those enumerated in Section 3 of Act 2927 and later in Section 2 of Commonwealth Act opinions, whether here or in the United States, there are practical considerations that
473. More accurately, they have always been considered as disqualifications, in the militate towards the same conclusions. As aptly stated in the motion for reconsideration
sense that those who did not possess them were the ones who could not "be lawfully of counsel for petitioner-appellee dated February 23, 1967, filed in the case of Zita Ngo
naturalized," just as if they were suffering from any of the disqualifications under Burca v. Republic, supra:
Section 2 of Act 2927 and later those under Section 4 of Commonwealth Act 473,
which, incidentally, are practically identical to those in the former law, except those in Unreasonableness of requiring alien wife to prove "qualifications" -chanrobles virtual
paragraphs (f) and (h) of the latter. 22Indeed, such is the clear impression anyone will law library
surely get after going over all the American decisions and opinions quoted and/or cited
in the latest USCA (1970), Title 8, section 1430, pp. 598-602, and the first decisions of There is one practical consideration that strongly militates against a construction that
this Court on the matter, Ly Giok Ha (1959) and Ricardo Cua, citing with approval the Section 15 of the law requires that an alien wife of a Filipino must affirmatively prove
opinions of the secretary of Justice. 23Such being the case, that is, that the so-called that she possesses the qualifications prescribed under Section 2, before she may be
racial requirements were always treated as disqualifications in the same light as the deemed a citizen. Such condition, if imposed upon an alien wife, becomes
other disqualifications under the law, why should their elimination not be viewed or unreasonably onerous and compliance therewith manifestly difficult. The
understood as a subtraction from or a lessening of the disqualifications? Why should unreasonableness of such requirement is shown by the following:
such elimination have instead the meaning that what were previously considered as
irrelevant qualifications have become disqualifications, as seems to be the import of the 1. One of the qualifications required of an Applicant for naturalization under Section 2
holding in Choy King Tee to the effect that the retention in Section 15 of of the law is that the applicant "must have resided in the Philippines for a continuous
Commonwealth Act 473 of the same language of what used to be Section 13 (a) of Act period of not less than ten years." If this requirement is applied to an alien wife married
2927 (as amended by Act 3448), notwithstanding the elimination of Section 1 of the to a Filipino citizen, this means that for a period of ten years at least, she cannot hope to
latter, necessarily indicates that the legislature had in mind making the phrase in acquire the citizenship of her husband. If the wife happens to be a citizen of a country
question "who may be lawfully naturalized" refer no longer to any racial whose law declares that upon her marriage to a foreigner she automatically loses her
disqualification but to the qualification under Section 2 of Commonwealth Act 473? citizenship and acquires the citizenship of her husband, this could mean that for a
Otherwise stated, under Act 2927, there were two groups of persons that could not be period of ten years at least, she would be stateless. And even after having acquired
naturalized, namely, those falling under Section 1 and those falling under Section 2, continuous residence in the Philippines for ten years, there is no guarantee that her
and surely, the elimination of one group, i.e. those belonging to Section 1, could not petition for naturalization will be granted, in which case she would remain stateless for
have had, by any process of reasoning, the effect of increasing, rather than decreasing, an indefinite period of time.chanroblesvirtualawlibrarychanrobles virtual law library
the disqualifications that used to be before such elimination. We cannot see by what
alchemy of logic such elimination could have convicted qualifications into 2. Section 2 of the law likewise requires of the applicant for naturalization that he
disqualifications specially in the light of the fact that, after all, these are "must own real estate in the Philippines worth not less than five thousand pesos,
disqualifications clearly set out as such in the law distinctly and separately from Philippine currency, or must have some known lucrative trade, profession, or lawful
53
occupation." Considering the constitutional prohibition against acquisition by an alien "shall be understood as reduced to five years for any petitioner (who is) married to a
of real estate except in cases of hereditary succession (Art. XIII, Sec. 5, Constitution), Filipino woman." It is absurd that an alien male married to a Filipino wife should be
an alien wife desiring to acquire the citizenship of her husband must have to prove that required to reside only for five years in the Philippines to qualify for citizenship,
she has a lucrative income derived from a lawful trade, profession or occupation. The whereas an alien woman married to a Filipino husband must reside for ten years.
income requirement has been interpreted to mean that the petitioner herself must be the
one to possess the said income. (Uy v. Republic, L-19578, Oct. 27, 1964; Tanpa Ong Thus under the interpretation given by this Court, it is more difficult for an alien wife
vs. Republic, L-20605, June 30, 1965; Li Tong Pek v. Republic, L-20912, November related by marriage to a Filipino citizen to become such citizen, than for a foreigner
29, 1965). In other words, the wife must prove that she has a lucrative income derived who is not so related. And yet, it seems more than clear that the general purpose of the
from sources other than her husband's trade, profession or calling. It is of common first paragraph of Section 15 was obviously to accord to an alien woman, by reason of
knowledge, and judicial notice may be taken of the fact that most wives in the her marriage to a Filipino, a privilege not similarly granted to other aliens. It will be
Philippines do not have gainful occupations of their own. Indeed, Philippine law, recalled that prior to the enactment of Act No. 3448 in 1928, amending Act No. 2927
recognizing the dependence of the wife upon the husband, imposes upon the latter the (the old Naturalization Law), there was no law granting any special privilege to alien
duty of supporting the former. (Art. 291, Civil Code). It should be borne in mind that wives of Filipinos. They were treated as any other foreigner. It was precisely to remedy
universally, it is an accepted concept that when a woman marries, her primary duty is to this situation that the Philippine legislature enacted Act No. 3448. On this point, the
be a wife, mother and housekeeper. If an alien wife is not to be remiss in this duty, how observation made by the Secretary of Justice in 1941 is enlightening:
can she hope to acquire a lucrative income of her own to qualify her for citizenship?
chanrobles virtual law library It is true that under, Article 22 of the (Spanish) Civil Code, the wife follows the
nationality of the husband; but the Department of State of the United States on October
3. Under Section 2 of the law, the applicant for naturalization "must have enrolled his 31, 1921, ruled that the alien wife of a Filipino citizen is not a Filipino citizen, pointing
minor children of school age, in any of the public schools or private schools recognized out that our Supreme Court in the leading case of Roa v. Collector of Customs (23 Phil.
by the Office of the Private Education of the Philippines, where Philippine history, 315) held that Articles 17 to 27 of the Civil Code being political have been abrogated
government and civics are taught or prescribed as part of the school curriculum during upon the cession of the Philippine Islands to the United States. Accordingly, the stated
the entire period of residence in the Philippines required of him prior to the hearing of taken by the Attorney-General prior to the envictment of Act No. 3448, was that
his petition for naturalization as Philippine citizen." If an alien woman has minor marriage of alien women to Philippine citizens did not make the former citizens of this
children by a previous marriage to another alien before she marries a Filipino, and such counting. (Op. Atty. Gen., March 16, 1928) .
minor children had not been enrolled in Philippine schools during her period of
residence in the country, she cannot qualify for naturalization under the interpretation To remedy this anomalous condition, Act No. 3448 was enacted in 1928 adding section
of this Court. The reason behind the requirement that children should be enrolled in 13(a) to Act No. 2927 which provides that "any woman who is now or may hereafter be
recognized educational institutions is that they follow the citizenship of their father. married to a citizen of the Philippine Islands, and who might herself be lawfully
(Chan Ho Lay v. Republic, L-5666, March 30, 1954; Tan Hi v. Republic, 88 Phil. 117 naturalized, shall be deemed a citizen of the Philippine Islands. (Op. No. 22, s. 1941;
[1951]; Hao Lian Chu v. Republic, 87 Phil. 668 [1950]; Yap Chin v. Republic, L-4177, emphasis ours).
May 29, 1953; Lim Lian Hong v. Republic, L-3575, Dec. 26, 1950). Considering that
said minor children by her first husband generally follow the citizenship of their alien If Section 15 of the, Revised Naturalization Law were to be interpreted, as this Court
father, the basis for such requirement as applied to her does not exist. Cessante ratione did, in such a way as to require that the alien wife must prove the qualifications
legis cessat ipsa lex.chanroblesvirtualawlibrarychanrobles virtual law library prescribed in Section 2, the privilege granted to alien wives would become illusory. It is
submitted that such a construction, being contrary to the manifested object of the
4. Under Section 3 of the law, the 10-year continuous residence prescribed by Section 2 statute must be rejected.
54
A statute is to be construed with reference to its manifest object, and if the language is farfetched to believe that in joining a Filipino family the alien woman is somehow
susceptible of two constructions, one which will carry out and the other defeat such disposed to assimilate the customs, beliefs and ideals of Filipinos among whom, after
manifest object, it should receive the former construction. (In re National Guard, 71 Vt. all, she has to live and associate, but surely, no one should expect her to do so even
493, 45 A. 1051; Singer v. United States, 323 U.S. 338, 89 L. ed. 285. See also, U.S. v. before marriage. Besides, it may be considered that in reality the extension of
Navarro, 19 Phil. 134 [1911]; U. S. v. Toribio, 15 Phil. 85 citizenship to her is made by the law not so much for her sake as for the husband.
[1910).chanroblesvirtualawlibrarychanrobles virtual law library Indeed, We find the following observations anent the national policy rationalization in
Choy King Tee and Ly Giok Ha (the second) to be quite persuasive:
... A construction which will cause objectionable results should be avoided and the
court will, if possible, place on the statute a construction which will not result in We respectfully suggest that this articulation of the national policy begs the question.
injustice, and in accordance with the decisions construing statutes, a construction which The avowed policy of "selectives admission" more particularly refers to a case where
will result in oppression, hardship, or inconveniences will also be avoided, as will a citizenship is sought to be acquired in a judicial proceeding for naturalization. In such a
construction which will prejudice public interest, or construction resulting in case, the courts should no doubt apply the national policy of selecting only those who
unreasonableness, as well as a construction which will result in absurd are worthy to become citizens. There is here a choice between accepting or rejecting
consequences.chanroblesvirtualawlibrarychanrobles virtual law library the application for citizenship. But this policy finds no application in cases where
citizenship is conferred by operation of law. In such cases, the courts have no choice to
So a construction should, if possible, be avoided if the result would be an apparent accept or reject. If the individual claiming citizenship by operation of law proves in
inconsistency in legislative intent, as has been determined by the judicial decisions, or legal proceedings that he satisfies the statutory requirements, the courts cannot do
which would result in futility, redundancy, or a conclusion not contemplated by the otherwise than to declare that he is a citizen of the Philippines. Thus, an individual who
legislature; and the court should adopt that construction which will be the least likely to is able to prove that his father is a Philippine citizen, is a citizen of the Philippines,
produce mischief. Unless plainly shown to have been the intention of the legislature, an "irrespective of his moral character, ideological beliefs, and identification with Filipino
interpretation which would render the requirements of the statute uncertain and vague ideals, customs, and traditions." A minor child of a person naturalized under the law,
is to be avoided, and the court will not ascribe to the legislature an intent to confer an who is able to prove the fact of his birth in the Philippines, is likewise a citizen,
illusory right. ... (82 C.J.S., Statutes, sec. 326, pp. 623-632). regardless of whether he has lucrative income, or he adheres to the principles of the
Constitution. So it is with an alien wife of a Philippine citizen. She is required to prove
7. In Choy King Tee and the second Ly Giok Ha, emphasis was laid on the need for only that she may herself be lawfully naturalized, i.e., that she is not one of the
aligning the construction of Section 15 with "the national policy of selective admission disqualified persons enumerated in Section 4 of the law, in order to establish her
to Philippine citizenship." But the question may be asked, is it reasonable to suppose citizenship status as a fact.chanroblesvirtualawlibrarychanrobles virtual law library
that in the pursuit of such policy, the legislature contemplated to make it more difficult
if not practically impossible in some instances, for an alien woman marrying a Filipino A paramount policy consideration of graver import should not be overlooked in this
to become a Filipina than any ordinary applicant for naturalization, as has just been regard, for it explains and justifies the obviously deliberate choice of words. It is
demonstrated above? It seems but natural and logical to assume that Section 15 was universally accepted that a State, in extending the privilege of citizenship to an alien
intended to extend special treatment to alien women who by marrying a Filipino wife of one of its citizens could have had no other objective than to maintain a unity of
irrevocably deliver themselves, their possessions, their fate and fortunes and all that allegiance among the members of the family. (Nelson v. Nelson, 113 Neb. 453, 203 N.
marriage implies to a citizen of this country, "for better or for worse." Perhaps there can W. 640 [1925]; see also "Convention on the Nationality of Married Women: Historical
and will be cases wherein the personal conveniences and benefits arising from Background and Commentary." UNITED NATIONS, Department of Economic and
Philippine citizenship may motivate such marriage, but must the minority, as such cases Social Affairs E/CN, 6/399, pp. 8 et seq.). Such objective can only be satisfactorily
are bound to be, serve as the criterion for the construction of law? Moreover, it is not achieved by allowing the wife to acquire citizenship derivatively through the husband.
55
This is particularly true in the Philippines where tradition and law has placed the developing country whose Constitution is nationalistic almost in the come. Certainly,
husband as head of the family, whose personal status and decisions govern the life of the writer of this opinion cannot be the last in rather passionately insisting that our
the family group. Corollary to this, our laws look with favor on the unity and solidarity jurisprudence should speak our own concepts and resort to American authorities, to be
of the family (Art. 220, Civil Code), in whose preservation of State as a vital and sure, entitled to admiration, and respect, should not be regarded as source of pride and
enduring interest. (See Art. 216, Civil Code). Thus, it has been said that by tradition in indisputable authority. Still, We cannot close our eyes to the undeniable fact that the
our country, there is a theoretic identity of person and interest between husband and provision of law now under scrutiny has no local origin and orientation; it is purely
wife, and from the nature of the relation, the home of one is that of the other. (See De la American, factually taken bodily from American law when the Philippines was under
Viña v. Villareal, 41 Phil. 13). It should likewise be said that because of the theoretic the dominating influence of statutes of the United States Congress. It is indeed a sad
identity of husband and wife, and the primacy of the husband, the nationality of commentary on the work of our own legislature of the late 1920's and 1930's that given
husband should be the nationality of the wife, and the laws upon one should be the law the opportunity to break away from the old American pattern, it took no step in that
upon the other. For as the court, in Hopkins v. Fachant (9th Cir., 1904) 65 C.C.A., 1, direction. Indeed, even after America made it patently clear in the Act of Congress of
130 Fed. 839, held: "The status of the wife follows that of the husband, ... and by virtue September 22, 1922 that alien women marrying Americans cannot be citizens of the
of her marriage her husband's domicile became her domicile." And the presumption United States without undergoing naturalization proceedings, our legislators still chose
under Philippine law being that the property relations of husband and wife are under to adopt the previous American law of August 10, 1855 as embodied later in Section
the regime of conjugal partnership (Art. 119, Civil Code), the income of one is also that 1994 of the Revised Statutes of 1874, Which, it is worth reiterating, was consistently
of the other.chanroblesvirtualawlibrarychanrobles virtual law library and uniformly understood as conferring American citizenship to alien women marrying
Americans ipso facto, without having to submit to any naturalization proceeding and
It is, therefore, not congruent with our cherished traditions of family unity and identity without having to prove that they possess the special qualifications of residence, moral
that a husband should be a citizen and the wife an alien, and that the national treatment character, adherence to American ideals and American constitution, provided they show
of one should be different from that of the other. Thus, it cannot be that the husband's they did not suffer from any of the disqualifications enumerated in the American
interests in property and business activities reserved by law to citizens should not form Naturalization Law. Accordingly, We now hold, all previous decisions of this Court
part of the conjugal partnership and be denied to the wife, nor that she herself cannot, indicating otherwise notwithstanding, that under Section 15 of Commonwealth Act
through her own efforts but for the benefit of the partnership, acquire such interests. 473, an alien woman marrying a Filipino, native born or naturalized, becomes ipso
Only in rare instances should the identity of husband and wife be refused recognition, facto a Filipina provided she is not disqualified to be a citizen of the Philippines under
and we submit that in respect of our citizenship laws, it should only be in the instances Section 4 of the same law. Likewise, an alien woman married to an alien who is
where the wife suffers from the disqualifications stated in Section 4 of the Revised subsequently naturalized here follows the Philippine citizenship of her husband the
Naturalization Law. (Motion for Reconsideration, Burca vs. Republic, supra.) moment he takes his oath as Filipino citizen, provided that she does not suffer from any
of the disqualifications under said Section 4.chanroblesvirtualawlibrarychanrobles
With all these considerations in mind, We are persuaded that it is in the best interest of virtual law library
all concerned that Section 15 of the Naturalization Law be given effect in the same way
as it was understood and construed when the phrase "who may be lawfully As under any other law rich in benefits for those coming under it, doubtless there will
naturalized," found in the American statute from which it was borrowed and copied be instances where unscrupulous persons will attempt to take advantage of this
verbatim, was applied by the American courts and administrative authorities. There is provision of law by entering into fake and fictitious marriages or mala
merit, of course in the view that Philippine statutes should be construed in the light of fide matrimonies. We cannot as a matter of law hold that just because of these
Philippine circumstances, and with particular reference to our naturalization laws. We possibilities, the construction of the provision should be otherwise than as dictated
should realize the disparity in the circumstances between the United States, as the so- inexorably by more ponderous relevant considerations, legal, juridical and practical.
called "melting pot" of peoples from all over the world, and the Philippines as a There can always be means of discovering such undesirable practice and every case can
56
be dealt with accordingly as it arises.chanroblesvirtualawlibrarychanrobles virtual law 3. We treat the present petition as one for naturalization. Or, in the words of law, a
library "petition for citizenship". This is as it should be. Because a reading of the petition will
reveal at once that efforts were made to set forth therein, and to prove afterwards,
III.chanroblesvirtualawlibrarychanrobles virtual law library compliance with Sections 2 and 4 of the Revised Naturalization law. The trial court
itself apparently considered the petition as one for naturalization, and, in fact, declared
The third aspect of this case requires necessarily a re-examination of the ruling of this petitioner "a citizen of the Philippines."
Court in Burca, supra, regarding the need of judicial naturalization proceedings before
the alien wife of a Filipino may herself be considered or deemed a Filipino. If this case In other words, under this holding, in order for an alien woman marrying a Filipino to
which, as already noted, was submitted for decision in 1964 yet, had only been decided be vested with Filipino citizenship, it is not enough that she possesses the qualifications
earlier, before Go Im Ty, the foregoing discussions would have been sufficient to prescribed by Section 2 of the law and none of the disqualifications enumerated in its
dispose of it. The Court could have held that despite her apparent lack of qualifications, Section 4. Over and above all these, she has to pass thru the whole process of judicial
her marriage to her co-petitioner made her a Filipina, without her undergoing any naturalization apparently from declaration of intention to oathtaking, before she can
naturalization proceedings, provided she could sustain, her claim that she is not become a Filipina. In plain words, her marriage to a Filipino is absolutely of no
disqualified under Section 4 of the law. But as things stand now, with the Burca ruling, consequence to her nationality vis-a-vis that of her Filipino husband; she remains to be
the question We have still to decide is, may she be deemed a Filipina without the national of the country to which she owed allegiance before her marriage, and if she
submitting to a naturalization proceeding?chanrobles virtual law library desires to be of one nationality with her husband, she has to wait for the same time that
any other applicant for naturalization needs to complete, the required period of ten year
Naturally, if Burca is to be followed, it is clear that the answer to this question must residence, gain the knowledge of English or Spanish and one of the principle local
necessarily be in the affirmative. As already stated, however, the decision in Burca has languages, make her children study in Filipino schools, acquire real property or engage
not yet become final because there is still pending with Us a motion for its in some lawful occupation of her own independently of her husband, file her
reconsideration which vigorously submits grounds worthy of serious consideration by declaration of intention and after one year her application for naturalization, with the
this Court. On this account, and for the reasons expounded earlier in this opinion, this affidavits of two credible witnesses of her good moral character and other
case is as good an occasion as any other to re-examine the qualifications, etc., etc., until a decision is ordered in her favor, after which, she has to
issue.chanroblesvirtualawlibrarychanrobles virtual law library undergo the two years of probation, and only then, but not before she takes her oath as
citizen, will she begin to be considered and deemed to be a citizen of the Philippines.
In the said decision, Justice Sanchez held for the Court: Briefly, she can become a Filipino citizen only by judicial
declaration.chanroblesvirtualawlibrarychanrobles virtual law library
We accordingly rule that: (1) An alien woman married to a Filipino who desires to be a
citizen of this country must apply therefore by filing a petition for citizenship reciting Such being the import of the Court's ruling, and it being quite obvious, on the other
that she possesses all the qualifications set forth in Section 2 and none of the hand, upon a cursory reading of the provision, in question, that the law intends by it to
disqualifications under Section 4, both of the Revised Naturalization Law; (2) Said spell out what is the "effect of naturalization on (the) wife and children" of an alien, as
petition must be filed in the Court of First Instance where petitioner has resided at least plainly indicated by its title, and inasmuch as the language of the provision itself
one year immediately preceding the filing of the petition; and (3) Any action by any clearly conveys the thought that some effect beneficial to the wife is intended by it,
other office, agency, board or official, administrative or otherwise - other than the rather than that she is not in any manner to be benefited thereby, it behooves Us to take
judgment of a competent court of justice - certifying or declaring that an alien wife of a second hard look at the ruling, if only to see whether or not the Court might have
the Filipino citizen is also a Filipino citizen, is hereby declared null and overlooked any relevant consideration warranting a conclusion different from that
void.chanroblesvirtualawlibrarychanrobles virtual law library complained therein. It is undeniable that the issue before Us is of grave importance,
57
considering its consequences upon tens of thousands of persons affected by the ruling To be sure, this appeal can be no less than what this Court attended to in Gan Tsitung
therein made by the Court, and surely, it is for Us to avoid, whenever possible, that Our vs. Republic, G.R. No. L-20819, Feb. 21, 1967, 19 SCRA 401 - when Chief Justice
decision in any case should produce any adverse effect upon them not contemplated Concepcion observed:
either by the law or by the national policy it seeks to
endorse.chanroblesvirtualawlibrarychanrobles virtual law library The Court realizes, however, that the rulings in the Barretto and Delgado cases -
although referring to situations the equities of which are not identical to those obtaining
AMICI CURIAE in the Burca case, respectable and impressive by their number and in the case at bar - may have contributed materially to the irregularities committed
standing in the Bar and well known for their reputation for intellectual integrity, legal therein and in other analogous cases, and induced the parties concerned to believe,
acumen and incisive and comprehensive resourcefulness in research, truly evident in although erroneously, that the procedure followed was valid under the
the quality of the memorandum they have submitted in said case, invite Our attention to law.chanroblesvirtualawlibrarychanrobles virtual law library
the impact of the decision therein thus:
Accordingly, and in view of the implications of the issue under consideration, the
The doctrine announced by this Honorable Court for the first time in the present case -- Solicitor General was required, not only, to comment thereon, but, also, to state "how
that an alien woman who marries a Philippine citizen not only does not ipso many cases there are, like the one at bar, in which certificates of naturalization have
facto herself become a citizen but can acquire such citizenship only through ordinary been issued after notice of the filing of the petition for naturalization had been
naturalization proceedings under the Revised Naturalization Law, and that all published in the Official Gazette only once, within the periods (a) from January 28,
administrative actions "certifying or declaring such woman to be a Philippine citizen 1950" (when the decision in Delgado v. Republic was promulgated) "to May 29, 1957"
are null and void" - has consequences that reach far beyond the confines of the present (when the Ong Son Cui was decided) "and (b) from May 29, 1957 to November 29,
case. Considerably more people are affected, and affected deeply, than simply Mrs. Zita 1965" (when the decision in the present case was
N. Burca. The newspapers report that as many as 15 thousand women married to rendered).chanroblesvirtualawlibrarychanrobles virtual law library
Philippine citizens are affected by this decision of the Court. These are women of many
and diverse nationalities, including Chinese, Spanish, British, American, Columbian, After mature deliberation, and in the light of the reasons adduced in appellant's motion
Finnish, Japanese, Chilean, and so on. These members of the community, some of for reconsideration and in the reply thereto of the Government, as well as of the data
whom have been married to citizens for two or three decades, have all exercised rights contained in the latter, the Court holds that the doctrine laid down in the Ong Son Cui
and privileges reserved by law to Philippine citizens. They will have acquired, case shall apply and affect the validity of certificates of naturalization issued after, not
separately or in conjugal partnership with their citizen husbands, real property, and they on or before May 29, 1957.
will have sold and transferred such property. Many of these women may be in
professions membership in which is limited to citizens. Others are doubtless Here We are met again by the same problem. In Gan Tsitung, the Court had to
stockholders or officers or employees in companies engaged in business activities for expressly enjoin the prospective application of its construction of the law made in a
which a certain percentage of Filipino equity content is prescribed by law. All these previous decision, 24which had already become final, to serve the ends of justice and
married women are now faced with possible divestment of personal status and of rights equity. In the case at bar, We do not have to go that far. As already observed, the
acquired and privileges exercised in reliance, in complete good faith, upon a reading of decision in Burca still under reconsideration, while the ruling in Lee Suan Ay, Lo San
the law that has been accepted as correct for more than two decades by the very Tuang, Choy King Tee and others that followed them have at the most become the law
agencies of government charged with the administration of that law. We must of the case only for the parties thereto. If there are good grounds therefor, all We have
respectfully suggest that judicial doctrines which would visit such comprehensive and to do now is to reexamine the said rulings and clarify or modify
far-reaching injury upon the wives and mothers of Philippine citizens deserve intensive them.chanroblesvirtualawlibrarychanrobles virtual law library
scrutiny and reexamination.
58
For ready reference, We requote Section 15: said status if they transfer their permanent residence to a foreign country before
becoming of age; (c) all such minor children, if born outside of the Philippines after
Sec. 15. Effect of the naturalization on wife and children. - Any woman who is now or such naturalization, shall also be "considered" Filipino citizens, unless they expatriate
may hereafter be married to a citizen of the Philippines, and who might herself be themselves by failing to register as Filipinos at the Philippine (American) Consulate of
lawfully naturalized shall be deemed a citizen of the the country where they reside and take the necessary oath of allegiance; and (d) as to
Philippines.chanroblesvirtualawlibrarychanrobles virtual law library the wife, she "shall be deemed a citizen of the Philippines" if she is one "who might
herself be lawfully naturalized". 26chanrobles virtual law library
Minor children of persons naturalized under this law who have been born in the
Philippines shall be considered citizens thereof.chanroblesvirtualawlibrarychanrobles No doubt whatever is entertained, so Burca holds very correctly, as to the point that the
virtual law library minor children, falling within the conditions of place and time of birth and residence
prescribed in the provision, are vested with Philippine citizenship directly by legislative
A foreign-born minor child, if dwelling in the Philippines at the time of naturalization fiat or by force of the law itself and without the need for any judicial proceeding or
of the parents, shall automatically become a Philippine citizen, and a foreign-born declaration. (At p. 192, 19 SCRA). Indeed, the language of the provision, is not
minor child, who is not in the Philippines at the time the parent is naturalized, shall be susceptible of any other interpretation. But it is claimed that the same expression "shall
deemed a Philippine citizen only during his minority, unless he begins to reside be deemed a citizen of the Philippines" in reference to the wife, does not necessarily
permanently in the Philippines when still a minor, in which case, he will continue to be connote the vesting of citizenship status upon her by legislative fiat because the
a Philippine citizen even after becoming of age.chanroblesvirtualawlibrarychanrobles antecedent phrase requiring that she must be one "who might herself be lawfully
virtual law library naturalized" implies that such status is intended to attach only after she has undergone
the whole process of judicial naturalization required of any person desiring to become a
A child born outside of the Philippines after the naturalization of his parent, shall be Filipino. Stated otherwise, the ruling in Burca is that while Section 15 envisages and
considered a Philippine citizen, unless within one year after reaching the age of intends legislative naturalization as to the minor children, the same section deliberately
majority, he fails to register himself as a Philippine citizen at the American Consulate treats the wife differently and leaves her out for the ordinary judicial
of the country where he resides, and to take the necessary oath of allegiance. naturalization.chanroblesvirtualawlibrarychanrobles virtual law library

It is obvious that the main subject-matter and purpose of the statute, the Revised Of course, it goes without saying that it is perfectly within the constitutional authority
Naturalization Law or Commonwealth Act 473, as a whole, is to establish a complete of the Congress of the Philippines to confer or vest citizenship status by legislative fiat.
procedure for the judicial conferment of the status of citizenship upon qualified aliens. (U.S. v. Wong Kim Ark, 169 U.S. 649, 42 L ed. 890 [1898]; See, 1 Tañada & Carreon,
After laying out such a procedure, remarkable for its elaborate and careful inclusion of Political Law of the Philippines 152 [1961 ed.]) In fact, it has done so for particular
all safeguards against the possibility of any undesirable persons becoming a part of our individuals, like two foreign religious prelates, 27hence there is no reason it cannot do it
citizenry, it carefully but categorically states the consequence of the naturalization of an for classes or groups of persons under general conditions applicable to all of the
alien undergoing such procedure it prescribes upon the members of his immediate members of such class or group, like women who marry Filipinos, whether native-born
family, his wife and children, 25and, to that end, in no uncertain terms it ordains that: (a) or naturalized. The issue before Us in this case is whether or not the legislature hag
all his minor children who have been born in the Philippines shall be "considered done so in the disputed provisions of Section 15 of the Naturalization Law. And Dr.
citizens" also; (b) all such minor children, if born outside the Philippines but dwelling Vicente G. Sinco, one of the most respect authorities on political law in the
here at the time of such naturalization "shall automatically become" Filipinos also, but Philippines 28observes in this connection thus: "A special form of naturalization is often
those not born in the Philippines and not in the Philippines at the time of such observed by some states with respect to women. Thus in the Philippines a foreign
naturalization, are also redeemed citizens of this country provided that they shall lose woman married to a Filipino citizen becomes ipso facto naturalized, if she belongs to
59
any of the classes who may apply for naturalization under the Philippine Laws." (Sinco, Obviously, these considerations leave Us no choice. Much as this Court may feel that
Phil. Political Law 498-499 [10th ed. 1954]; emphasis ours; this comment is as the United States herself has evidently found it to be an improvement of her national
substantially reiterated in the 1962 edition, citing Ly Giok Ha and Ricardo policy vis-a-vis the alien wives of her citizens to discontinue their automatic
Cua, supra.)chanrobles virtual law library incorporation into the body of her citizenry without passing through the judicial
scrutiny of a naturalization proceeding, as it used to be before 1922, it seems but
More importantly, it may be stated, at this juncture, that in construing the provision of proper, without evidencing any bit of colonial mentality, that as a developing country,
the United States statutes from which our law has been copied, 28a the American courts the Philippines adopt a similar policy, unfortunately, the manner in which our own
have held that the alien wife does not acquire American citizenship by choice but by legislature has enacted our laws on the subject, as recounted above, provides no basis
operation of law. "In the Revised Statutes the words "and taken" are omitted. The effect for Us to construe said law along the line of the 1922 modification of the American
of this statute is that every alien woman who marries a citizen of the United States Law. For Us to do so would be to indulge in judicial legislation which it is not
becomes perforce a citizen herself, without the formality of naturalization, and institutionally permissible for this Court to do. Worse, this court would be going
regardless of her wish in that respect." (USCA 8, p. 601 [1970 ed.], citing Mackenzie v. precisely against the grain of the implicit Legislative
Hare, 1913, 134 P. 713, 165 Cal. 766, affirmed 36 S. Ct. 106, 239 U.S. 299, 60 L ed. intent.chanroblesvirtualawlibrarychanrobles virtual law library
297.) .chanroblesvirtualawlibrarychanrobles virtual law library
There is at least one decision of this Court before Burca wherein it seems it is quite
We need not recount here again how this provision in question was first enacted as clearly implied that this Court is of the view that under Section 16 of the Naturalization
paragraph (a) of Section 13, by way of an insertion into Act 2927 by Act 3448 of Law, the widow and children of an applicant for naturalization who dies during the
November 30, 1928, and that, in turn, and paragraph was copied verbatim from Section proceedings do not have to submit themselves to another naturalization proceeding in
1994 of the Revised Statutes of the United States, which by that time already had a order to avail of the benefits of the proceedings involving the husband. Section 16
long accepted construction among the courts and administrative authorities in that provides: .
country holding that under such provision an alien woman who married a citizen
became, upon such marriage, likewise a citizen by force of law and as a consequence of SEC. 16. Right of widow and children of petitioners who have died. - In case a
the marriage itself without having to undergo any naturalization proceedings, provided petitioner should die before the final decision has been rendered, his widow and minor
that, it could be shown that at the time of such marriage, she was not disqualified to be children may continue the proceedings. The decision rendered in the case shall, so far
naturalized under the laws then in force. To repeat the discussion We already made of as the widow and minor children are concerned, produce the same legal effect as if it
these undeniable facts would unnecessarily make this decision doubly extensive. The had been rendered during the life of the petitioner.
only point which might be reiterated for emphasis at this juncture is that whereas in the
United States, the American Congress, recognizing the construction, of Section 1994 of In Tan Lin v. Republic, G.R. No. L-13706, May 31, 1961, 2 SCRA 383, this Court
the Revised Statutes to be as stated above, and finding it desirable to avoid the effects held:
of such construction, approved the Act of September 22, 1922 Explicitly requiring all
such alien wives to submit to judicial naturalization albeit under more liberal terms Invoking the above provisions in their favor, petitioners-appellants argue (1) that under
than those for other applicants for citizenship, on the other hand, the Philippine said Sec. 16, the widow and minor children are allowed to continue the same
Legislature, instead of following suit and adopting such a requirement, enacted Act proceedings and are not substituted for the original petitioner; (2) that the qualifications
3448 on November 30, 1928 which copied verbatim the aforementioned Section 1994 of the original petitioner remain to be in issue and not those of the widow and minor
of the Revised Statutes, thereby indicating its preference to adopt the latter law and its children, and (3) that said Section 16 applies whether the petitioner dies before or after
settled construction rather than the reform introduced by the Act of final decision is rendered, but before the judgment becomes
1922.chanroblesvirtualawlibrarychanrobles virtual law library executory.chanroblesvirtualawlibrarychanrobles virtual law library
60
There is force in the first and second arguments. Even the second sentence of said Legislature intended to treat them differently.chanroblesvirtualawlibrarychanrobles
Section 16 contemplate the fact that the qualifications of the original petitioner remains virtual law library
the subject of inquiry, for the simple reason that it states that "The decision rendered in
the case shall, so far as the widow and minor children are concerned, produce the same Additionally, We have carefully considered the arguments advanced in the motion for
legal effect as if it had been rendered during the life of the petitioner." This phraseology reconsideration in Burca, and We see no reason to disagree with the following views of
emphasizes the intent of the law to continue the proceedings with the deceased as the counsel: .
theoretical petitioner, for if it were otherwise, it would have been unnecessary to
consider the decision rendered, as far as it affected the widow and the minor It is obvious that the provision itself is a legislative declaration of who may be
children.chanroblesvirtualawlibrarychanrobles virtual law library considered citizens of the Philippines. It is a proposition too plain to be disputed that
Congress has the power not only to prescribe the mode or manner under which
xxx xxx xxxchanrobles virtual law library foreigners may acquire citizenship, but also the very power of conferring citizenship by
legislative fiat. (U. S. v. Wong Kim Ark, 169 U. S. 649, 42 L. Ed. 890 [1898] ; see 1
The Chua Chian case (supra), cited by the appellee, declared that a dead person can not Tañada and Carreon, Political Law of the Philippines 152 [1961 ed.]) The Constitution
be bound to do things stipulated in the oath of allegiance, because an oath is a personal itself recognizes as Philippine citizens "Those who are naturalized in accordance with
matter. Therein, the widow prayed that she be allowed to take the oath of allegiance for law" (Section 1[5], Article IV, Philippine Constitution). Citizens by naturalization,
the deceased. In the case at bar, petitioner Tan Lin merely asked that she be allowed to under this provision, include not only those who are naturalized in accordance with
take the oath of allegiance and the proper certificate of naturalization, once the legal proceedings for the acquisition of citizenship, but also those who acquire
naturalization proceedings of her deceased husband, shall have been completed, not on citizenship by "derivative naturalization" or by operation of law, as, for example, the
behalf of the deceased but on her own behalf and of her children, as recipients of the "naturalization" of an alien wife through the naturalization of her husband, or by
benefits of his naturalization. In other words, the herein petitioner proposed to take the marriage of an alien woman to a citizen. (See Tañada & Carreon, op. cit. supra, at 152,
oath of allegiance, as a citizen of the Philippines, by virtue of the legal provision that 172; Velayo, Philippine Citizenship and Naturalization 2 [1965 ed.]; 1 Paras, Civil
"any woman who is now or may hereafter be married to a citizen of the Philippines and Code 186 [1967 ed.]; see also 3 Hackworth, Digest of International Law
who might herself be lawfully naturalized shall be deemed a citizen of the Philippines. 3).chanroblesvirtualawlibrarychanrobles virtual law library
Minor children of persons naturalized under this law who have been born in the
Philippines shall be considered citizens thereof." (Section 15, Commonwealth Act No. The phrase "shall be deemed a citizen of the Philippines" found in Section 14 of the
473). The decision granting citizenship to Lee Pa and the record of the case at bar, do Revised Naturalization Law clearly manifests an intent to confer citizenship.
not show that the petitioning widow could not have been lawfully naturalized, at the Construing a similar phrase found in the old U.S. naturalization law (Revised Statutes,
time Lee Pa filed his petition, apart from the fact that his 9 minor children were all born 1994), American courts have uniformly taken it to mean that upon her marriage, the
in the Philippines. (Decision, In the Matter of the Petition of Lee Pa to be admitted a alien woman becomes by operation of law a citizen of the United States as fully as if
citizen of the Philippines, Civil Case No. 16287, CFI, Manila, Annex A; Record on she had complied with all the provisions of the statutes upon the subject of
Appeal, pp. 8-11). The reference to Chua Chian case is, therefore, premature. naturalization. (U.S. v. Keller, 13 F. 82; U.S. Opinions of the US Attorney General
dated June 4, 1874 [14 Op. 4021, July 20, 1909 [27 Op. 507], December 1, 1910 [28
Section 16, as may be seen, is a parallel provision to Section 15. If the widow of an Op. 508], Jan. 15, 1920 [32 Op. 2091 and Jan. 12, 1923 [23 398]).
applicant for naturalization as Filipino, who dies during the proceedings, is not required
to go through a naturalization preceeding, in order to be considered as a Filipino citizen The phrase "shall be deemed a citizen," in Section 1994 Revised Statute (U.S. Comp.
hereof, it should follow that the wife of a living Filipino cannot be denied the same Stat. 1091, 1268) or as it was in the Act of 1855 (10 Stat. at L. 604, Chapt. 71, Sec. 2),
privilege. This is plain common sense and there is absolutely no evidence that the "shall be deemed and taken to be a citizens" while it may imply that the person to
61
whom it relates has not actually become a citizen by the ordinary means or in the usual facto a citizen of the Philippines from the time the fact of relationship concurs with the
way, as by the judgment of a competent court, upon a proper application and proof, yet fact of citizenship of his parent, and the time when the child became a citizen does not
it does not follow that such person is on that account practically any the less a depend upon the time that he is able to prove that he was born in the Philippines. The
citizen. The word "deemed" is the equivalent of "considered" or "judged," and child may prove some 25 years after the naturalization of his father that he was born in
therefore, whatever an Act of Congress requires to be "deemed" or "taken" as true of the Philippines and should, therefore, be "considered" a citizen thereof. It does not
any person or thing must, in law, be considered as having been duly adjudged or mean that he became a Philippine citizen only at that later time. Similarly, an alien
established concerning such person or thing, and have force and effect accordingly. woman who married a Philippine citizen may be able to prove only some 25 years after
When, therefore, Congress declares that an alien woman shall, under certain her marriage (perhaps, because it was only 25 years after the marriage that her
circumstances, be "deemed" an American citizen, the effect when the contingency citizenship status became in question), that she is one who might herself be lawfully
occurs, is equivalent to her being naturalized directly by an Act of Congress or in the naturalized." It is not reasonable to conclude that she acquired Philippine citizenship
usual mode thereby prescribed. (Van Dyne, Citizenship of the United States 239, cited only after she had proven that she "might herself be lawfully naturalized." It is not
in Velayo, Philippine Citizenship and Naturalization 146-147 [1965 ed.]; emphasis reasonable to conclude that she acquired Philippine citizenship only after she had
ours). proven that she "might herself be lawfully naturalized."chanrobles virtual law library

That this was likewise the intent of the Philippine legislature when it enacted the first The point that bears emphasis in this regard is that in adopting the very phraseology of
paragraph of Section 15 of the Revised Naturalization Law is shown by a textual the law, the legislature could not have intended that an alien wife should not be
analysis of the entire statutory provision. In its entirety, Section 15 reads: deemed a Philippine citizen unless and until she proves that she might herself be
lawfully naturalized. Far from it, the law states in plain terms that she shall be
(See supra). deemed a citizen of the Philippines if she is one "who might herself be lawfully
naturalized." The proviso that she must be one "who might herself be lawfully
The phrases "shall be deemed" "shall be considered," and "shall automatically become" naturalized" is not a condition precedent to the vesting or acquisition of citizenship; it
as used in the above provision, are undoubtedly synonymous. The leading idea or is only a condition or a state of fact necessary to establish her citizenship as a factum
purpose of the provision was to confer Philippine citizenship by operation of law upon probandum, i.e., as a fact established and proved in evidence. The word "might," as
certain classes of aliens as a legal consequence of their relationship, by blood or by used in that phrase, precisely replies that at the time of her marriage to a Philippine
affinity, to persons who are already citizens of the Philippines. Whenever the fact of citizen, the alien woman "had (the) power" to become such a citizen herself under the
relationship of the persons enumerated in the provision concurs with the fact of laws then in force. (Owen v. Kelly, 6 DC 191 [1867], aff'd Kelly v. Owen, 76 US 496,
citizenship of the person to whom they are related, the effect is for said persons to 19 L ed 283 [1869). That she establishes such power long after her marriage does not
become ipso facto citizens of the Philippines. "Ipso facto" as here used does not mean alter the fact that at her marriage, she became a
that all alien wives and all minor children of Philippine citizens, from the mere fact of citizen.chanroblesvirtualawlibrarychanrobles virtual law library
relationship, necessarily become such citizens also. Those who do not meet the
statutory requirements do not ipso facto become citizens; they must apply for (This Court has held) that "an alien wife of a Filipino citizen may not acquire the status
naturalization in order to acquire such status. What it does mean, however, is that in of a citizen of the Philippines unless there is proof that she herself may be lawfully
respect of those persons enumerated in Section 15, the relationship to a citizen of the naturalized" (Decision, pp. 3-4). Under this view, the "acquisition" of citizenship by the
Philippines is the operative fact which establishes the acquisition of Philippine alien wife depends on her having proven her qualifications for citizenship, that is, she is
citizenship by them. Necessarily, it also determines the point of time at which such not a citizen unless and until she proves that she may herself be lawfully naturalized. It
citizenship commences. Thus, under the second paragraph of Section 15, a minor child is clear from the words of the law that the proviso does not mean that she must first
of a Filipino naturalized under the law, who was born in the Philippines, becomes ipso prove that she "might herself be lawfully naturalized" before she shall be deemed (by
62
Congress, not by the courts) a citizen. Even the "uniform" decisions cited by this Court It is true that unless and until the alien wife proves that she might herself be lawfully
(at fn. 2) to support its holding did not rule that the alien wife becomes a citizen naturalized, it cannot be said that she has established her status as a proven fact. But
only after she has proven her qualifications for citizenship. What those decisions ruled neither can it be said that on that account, she did not become a citizen of the
was that the alien wives in those cases failed to prove their qualifications and Philippines. If her citizenship status is not questioned in any legal proceeding, she
therefore they failed to establish their claim to citizenship. Thus in Ly Giok Ha v. obviously has no obligation to establish her status as a fact. In such a case, the
Galang, 101 Phil. 459 [l957], the case was remanded to the lower court for presumption of law should be that she is what she claims to be. (U.S. v. Roxas, 5 Phil.
determination of whether petitioner, whose claim to citizenship by marriage to a 375 [1905]; Hilado v. Assad, 51 O.G. 4527 [1955]). There is a presumption that a
Filipino was disputed by the Government, "might herself be lawfully naturalized," for representation shown to have been made is true. (Aetna Indemnity Co. v. George A.
the purpose of " proving her alleged change of political status from alien to citizen" (at Fuller, Co., 73 A. 738, 74 A. 369, 111 ME. 321).
464). In Cua v. Board, 101 Phil. 521 [1957], the alien wife who was being deported,
claimed she was a Philippine citizen by marriage to a Filipino. This Court finding that The question that keeps bouncing back as a consequence of the foregoing views is,
there was no proof that she was not disqualified under Section 4 of the Revised what substitute is them for naturalization proceedings to enable the alien wife of a
Naturalization Law, ruled that: "No such evidence appearing on record, the claim of Philippine citizen to have the matter of her own citizenship settled and established so
assumption of Philippine citizenship by Tijoe Wu Suan, upon her marriage to that she may not have to be called upon to prove it everytime she has to perform an act
petitioner, is untenable." (at 523) It will be observed that in these decisions cited by this or enter in to a transaction or business or exercise a right reserved only to Filipinos?
Court, the lack of proof that the alien wives "might (themselves) be lawfully The ready answer to such question is that as the laws of our country, both substantive
naturalized" did not necessarily imply that they did not become, in truth and in fact, and procedural, stand today, there is no such procedure, but such paucity is no proof
citizens upon their marriage to Filipinos. What the decisions merely held was that these that the citizenship under discussion is not vested as of the date of marriage or the
wives failed to establish their claim to that status as a proven husband's acquisition of citizenship, as the case may be, for the truth is that the same
fact.chanroblesvirtualawlibrarychanrobles virtual law library situation objections even as to native-born Filipinos. Everytime the citizenship of a
person is material or indispensable in a judicial or administrative case, whatever the
In all instances where citizenship is conferred by operation of law, the time when corresponding court or administrative authority decides therein as to such citizenship is
citizenship is conferred should not be confused with the time when citizenship status is generally not considered as res adjudicata, hence it has to be threshed out again and
established as a proven fact. Thus, even a natural-born citizen of the Philippines, whose again as the occasion may demand. This, as We view it, is the sense in which Justice
citizenship status is put in issue in any proceeding would be required to prove, for Dizon referred to "appropriate proceeding" in Brito v. Commissioner, supra. Indeed,
instance, that his father is a citizen of the Philippines in order to factually establish his only the good sense and judgment of those subsequently inquiring into the matter may
claim to citizenship.* His citizenship status commences from the time of birth, make the effort easier or simpler for the persons concerned by relying somehow on the
although his claim thereto is established as a fact only at a subsequent time. Likewise, antecedent official findings, even if these are not really
an alien woman who might herself be lawfully naturalized becomes a Philippine citizen binding.chanroblesvirtualawlibrarychanrobles virtual law library
at the time of her marriage to a Filipino husband, not at the time she is able to establish
that status as a proven fact by showing that she might herself be lawfully naturalized. It may not be amiss to suggest, however, that in order to have a good starting point and
Indeed, there is no difference between a statutory declaration that a person is deemed a so that the most immediate relevant public records may be kept in order, the following
citizen of the Philippines provided his father is such citizen from a declaration that an observations in Opinion No. 38, series of 1958, of then Acting Secretary of Justice
alien woman married to a Filipino citizen of the Philippines provided she might herself Jesus G. Barrera, may be considered as the most appropriate initial step by the
be lawfully naturalized. Both become citizens by operation of law; the former becomes interested parties:
a citizen ipso facto upon birth; the later ipso facto upon
marriage.chanroblesvirtualawlibrarychanrobles virtual law library Regarding the steps that should be taken by an alien woman married to a Filipino
63
citizen in order to acquire Philippine citizenship, the procedure followed in the Bureau
of Immigration is as follows: The alien woman must file a petition for the cancellation
of her alien certificate of registration alleging, among other things, that she is married
to a Filipino, citizen and that she is not disqualified from acquiring her husband's
citizenship pursuant to section 4 of Commonwealth Act No. 473, as amended. Upon the
filing of said petition, which should be accompanied or supported by the joint affidavit
of the petitioner and her Filipino husband to the effect that the petitioner does not
belong to any of the groups disqualified by the cited section from becoming naturalized
Filipino citizen (please see attached CEB Form 1), the Bureau of Immigration conducts
an investigation and thereafter promulgates its order or decision granting or denying the
petition.

Once the Commissioner of Immigration cancels the subject's registration as an alien,


there will probably be less difficulty in establishing her Filipino citizenship in any other
proceeding, depending naturally on the substance and vigor of the
opposition.chanroblesvirtualawlibrarychanrobles virtual law library

Before closing, it is perhaps best to clarify that this third issue We have passed upon
was not touched by the trial court, but as the point is decisive in this case, the Court
prefers that the matter be settled once and for all
now.chanroblesvirtualawlibrarychanrobles virtual law library

IN VIEW OF ALL THE FOREGOING, the judgment of the Court a quo dismissing
appellants' petition for injunction is hereby reversed and the Commissioner of
Immigration and/or his authorized representative is permanently enjoined from causing
the arrest and deportation and the confiscation of the bond of appellant Lau Yuen
Yeung, who is hereby declared to have become a Filipino citizen from and by virtue of
her marriage to her co-appellant Moy Ya Lim Yao alias Edilberto Aguinaldo Lim, a
Filipino citizen on January 25, 1962. No costs.

64
Oh Hek How v. Republic petitioner shall "solemnly swear," inter alia, that he renounces "absolutely and forever
29 SCRA 94 all allegiance and fidelity to any
Petitioner Oh Hek How having been granted naturalization through his petition filed a foreign prince, potentate" and particularly to the state "of which" he is "a subject or
motion alleging that he had complied with citizen." The obvious purpose of
the requirements of Republic Act No. 530 and praying that he be allowed to take his this requirement is to divest him of his former nationality, before acquiring Philippine
oath of allegiance as such citizen and issued citizenship, because, otherwise,
the corresponding certificate of naturalization. he would have two nationalities and owe allegiance to two (2) distinct sovereignties,
The Court of First Instance of Zamboanga del Norte issued forthwith an order which our laws do not permit,
authorizing the taking of said oath. On that same except that, pursuant to Republic Act No. 2639, "the acquisition of citizenship by a
date, petitioner took it and the certificate of naturalization was issued to him. natural-born Filipino citizen from
The Government seasonably gave notice of its intention to appeal from said order of one of the Iberian and any friendly democratic Ibero-American countries shall not
February 9, 1966 and filed its record on produce loss or forfeiture of his
appeal among the grounds that the oath was taken prior to judgment having been final Philippine citizenship, if the law of that country grants the same privilege to its citizens
and executor. and such had been agreed upon
Issue:
Is the oath valid? by treaty between the Philippines and the foreign country from which citizenship is
Whether or not a permission to renounce citizenship is necessary from the Minister of acquired."
the Interior of Nationalist China.
Held:
First issue:
The order of February 9, 1966 (oath-taking) had not — and up to the present has
not — become final and executory in
view of the appeal duly taken by the Government.
2nd Issue:
It is argued that the permission is not required by our laws and that the
naturalization of an alien, as a citizen of the
Philippines, is governed exclusively by such laws and cannot be controlled by any
foreign law. However, the question of
how a Chinese citizen may strip himself of that status is necessarily governed —
pursuant to Articles 15 and 16 of our
Civil Code — by the laws of China, not by those of the Philippines.
As a consequence, a Chinese national cannot be naturalized as a citizen of the
Philippines, unless he has complied with the laws
of Nationalist China requiring previous permission of its Minister of the Interior for the
renunciation of nationality.
• Section 12 of Commonwealth Act No. 473 provides, however, that before the
naturalization certificate is issued, the
65
At the outset, it is obvious that the oath of allegiance taken by petitioner on November
G.R. No. L-27429 August 27, 1969 28, 1966, and the certificate of naturalization issued to him in pursuance thereof, as
well as the authority given therefor by the lower court, are null and void. Indeed, the
IN THE MATTER OF THE PETITION FOR ADMISSION AS CITIZEN OF THE order of February 9, had not — and up to the present has not — become final and
PHILIPPINES. executory in view of the appeal duly taken by the Government. What is more,
OH HEK HOW, petitioner appellee, petitioner's second oath was taken, not only after the filing of the notice of appeal 1 and
vs. the submission of the record on appeal, but also after the approval thereof. In other
REPUBLIC OF THE PHILIPPINES, oppositor-appellant. words, the lower court had already lost its jurisdiction over the case. 2

Eliezer M. Echavez for petitioner-appellee. Again, petitioner's net income in 1960 and 1961 was P3,945.65 and P5,105.79,
Office of the Solicitor General Antonio P. Barredo, Assistant Solicitor General respectively, or from about P330 to P425 a month. His income tax return for 1962, filed
Felicisimo R. Rosete and Solicitor Santiago M. Kapunan for oppositor-appellant. subsequently to the institution of this case, showed a net income of P6,485.50 for that
year, or about P540 a month. Considering that petitioner has a wife and three (3)
CONCEPCION, C.J.: children, one of them of school age, at the time of the filing of his application for
naturalization, his aforementioned income is not a lucrative one. Indeed, it has been
A decision granting his petition for naturalization as citizen of the Philippines having held that the following incomes are not lucrative, from the viewpoint of our
been rendered on January 16, 1964, petitioner Oh Hek How filed, on January 17, 1966, naturalization laws, namely: (1) P4,200 3 or P5,000 a year 4 for one married, with five
a motion alleging that he had complied with the requirements of Republic Act No. 530 (5) children; 5 (2) P6,000 a year for one married, with two (2) minor children; 5 and (3)
and praying that he be allowed to take his oath of allegiance as such citizen and issued P6,000 6 or P6,300 a year 7 for one married, with only one (1) child.
the corresponding certificate of naturalization. Upon petitioner's testimony, taken on
February 9, 1966, the date set for the hearing of said motion, the Court of First Instance Lastly, it is conceded that petitioner has not required from the Minister of the Interior of
of Zamboanga del Norte issued forthwith an order authorizing the taking of said oath. Nationalist China the permission required by the laws thereof for a valid renunciation
On that same date, petitioner took it and the certificate of naturalization was issued to of his Chinese citizenship. In Go A. Leng v. Republic, 8 a decision granting the
him. application for naturalization of a Chinese national was reversed by this Court, upon
the ground, among others, of "his failure to secure" the aforementioned permission.
The Government seasonably gave notice of its intention to appeal from said order of
February 9, 1966 and filed its record on appeal. Before the same was approved, it also It is argued that the same is not required by our laws and that the naturalization of an
moved to cancel petitioner's certificate of naturalization, upon the ground, among alien, as a citizen of the Philippines, is governed exclusively by such laws and cannot
others, that it was issued and the oath taken before said order of February 9, 1966, had be controlled by any foreign law. Section 12 of Commonwealth Act No. 473 provides,
become final and executory. Acting upon this motion and petitioner's opposition however, that before the naturalization certificate is issued, the petitioner shall
thereto, the court issued, on October 3, 1966, an order granting the motion, but, at the "solemnly swear," inter alia, that he renounces "absolutely and forever all allegiance
same time, authorizing the taking of a new oath by the petitioner and the issuance in his and fidelity to any foreign prince, potentate" and particularly to the state "of which" he
favor of another certificate of naturalization, after thirty (30) days from notice to the is "a subject or citizen." The obvious purpose of this requirement is to divest him of his
Solicitor General. Thereafter, or on November 26, 1966, the court approved the record former nationality, before acquiring Philippine citizenship, because, otherwise, he
on appeal and, once more, authorized the petitioner to "take a new or proper oath to would have two nationalities and owe allegiance to two (2) distinct sovereignties,
validate the first one made on February 9, 1966." The case is now before us on said which our laws do not permit, except that, pursuant to Republic Act No. 2639, "the
record on appeal filed by the Government. acquisition of citizenship by a natural-born Filipino citizen from one of the Iberian and
66
any friendly democratic Ibero-American countries shall not produce loss or forfeiture
of his Philippine citizenship, if the law of that country grants the same privilege to its
citizens and such had been agreed upon by treaty between the Philippines and the
foreign country from which citizenship is acquired." The question of how a Chinese
citizen may strip himself of that status is necessarily governed — pursuant to Articles
15 and 16 of our Civil Code — by the laws of China, not by those of the
Philippines. 9 As a consequence, a Chinese national cannot be naturalized as a citizen
of the Philippines, unless he has complied with the laws of Nationalist China requiring
previous permission of its Minister of the Interior for the renunciation of nationality.

The view to the contrary, adhered to in Parado v. Republic, 10 Chausintek v.


Republic, 11 and Lim So v. Republic 12has been superseded by our ruling in the
subsequent case of Go A. Leng v. Republic 13 which we hereby reiterate.

WHEREFORE, the order appealed from is reversed, and the oath of allegiance taken,
on November 28, 1966, by petitioner Oh Hek How, as well as the certificate of
naturalization issued in pursuance thereto, are hereby declared null and void, with costs
against said petitioner, who is, moreover, directed to surrender the aforementioned
certificate of naturalization to the Clerk of the Court of First Instance of Zamboanga del
Norte, within ten (10) days after this decision shall have become final. It is so ordered.

67
Board of Commissioners v. Dela Rosa subject are presumed to be the same as those of the Philippines. In the case at bar, there
Facts: being no proof of Chinese law
On July 12, 1960, Santiago Gatchalian, grandfather of William Gatchalian, was recognized by relating to marriage, there arises the presumption that it is the same as that of Philippine
the Bureau of Immigration as a law.
native born Filipino citizen following the citizenship of natural mother Mariana Gatchalian. On The lack of proof of Chinese law on the matter cannot be blamed on Santiago Gatchalian much
June 27, 1961, Willian, then more on respondent William
twelve years old, arrives in Manila from Hongkong together with a daughter and a son of Gatchalian who was then a twelve-year old minor. The fact is, as records indicate, Santiago
Santiago. They had with them was not pressed by the Citizenship
certificate of registration and identity issued by the Philippine consulate in Hongkong based on Investigation Board to prove the laws of China relating to marriage, having been content
a cablegram bearing the signature with the testimony of Santiago
of the secretary of foreign affairs, Felixberto Serrano, and sought admission as Filipino that the Marriage Certificate was lost or destroyed during the Japanese occupation of
citizens. China.
On August 15, 1990, the Commission on Immigration and Deportatiion ordered the arrest of The testimonies of Santiago Gatchalian and Francisco Gatchalian before the Philippine
William and was released upon consular and immigration authorities
posting P 200,000 cash bond. Thus on the 29th of the same month, he filed a petition for regarding their marriages, birth and relationship to each other are not self-serving but are
certiorari and prohibition before the admissible in evidence as statements or
RTC of Manila. A motion to dismiss was filed but denied. declarations regarding family reputation or tradition in matters of pedigree (Sec. 34, Rule 130).
Petitioners, claim that respondent is an alien. In support of their position, petitioners point out Philippine law, following the lex loci celebrationis, adheres to the rule that a marriage
that Santiago Gatchalian's marriage formally valid where celebrated is
with Chu Gim Tee in China as well as the marriage of Francisco (father of William) Gatchalian valid everywhere. Referring to marriages contracted abroad, Art. 71 of the Civil Code (now
to Ong Chiu Kiok, likewise in Art. 26 of the Family Code)
China, were not supported by any evidence other than their own self-serving testimony nor was provides that "all marriages performed outside of the Philippines in accordance with the laws in
there any showing what the laws force in the country where they
of China were. It is the postulate advanced by petitioners that for the said marriages to be valid were performed, and valid there as such, shall also be valid in this country . . ."
in this country, it should have And any doubt as to the validity of the matrimonial unity and the extent as to how far the
been shown that they were valid by the laws of China wherein the same were contracted. There validity of such marriage may be
being none, petitioners extended to the consequences of the coverture is answered by Art. 220 of the Civil Code in this
conclude that the aforesaid marriages cannot be considered valid. Hence, Santiago's manner: "In case of doubt, all
children, including Francisco, presumptions favor the solidarity of the family. Thus, every intendment of law or facts
followed the citizenship of their mother, having been born outside of a valid marriage. leans toward the validity of
Similarly, the validity of the marriage, the indissolubility of the marriage bonds, the legitimacy of children, the
Francisco's marriage not having been demonstrated, William and Johnson followed the community of property during
citizenship of their mother, a Chinese marriage, the authority of parents over their children, and the validity of defense for any
national. member of the family in case of
Issue: unlawful aggression." (Emphasis supplied). Bearing in mind the "processual presumption"
Whether or not William Gatchalian is to be declared as a Filipino citizen enunciated in Miciano and other
Held: cases, he who asserts that the marriage is not valid under our law bears the burden of proof to
In Miciano vs. Brimo (50 Phil. 867 [1924]; Lim and Lim vs. Collector of Customs, 36 Phil. present the foreign law.
472; Yam Ka Lim vs. Collector of Having declared the assailed marriages as valid, respondent William Gatchalian follows
Customs, 30 Phil. 46 [1915]), this Court held that in the absence of evidence to the the citizenship of his father
contrary, foreign laws on a particular Francisco, a Filipino, as a legitimate child of the latter. Francisco, in turn is likewise a
68
Filipino being the legitimate child of
Santiago Gatchalian who (the latter) is admittedly a Filipino citizen whose Philippine
citizenship was recognized by the Bureau
of Immigration in an order dated July 12, 1960.
Finally, respondent William Gatchalian belongs to the class of Filipino citizens who became as
such at the time of the adoption of
the Constitution. . . .
Nota bene: for Gatchalian
In Moy Ya Lim vs. Commissioner of Immigration (41 SCRA 292 [1971]) and in Lee vs.
Commissioner of Immigration (supra),
this Court declared that:
Everytime the citizenship of a person is material or indispensable in a judicial or administrative
case, whatever the corresponding
court or administrative authority decides therein as to such citizenship is generally not
considered as res adjudicata, hence it
has to be threshed out again and again as the occasion may demand.
An exception to the above rule was laid by this Court in Burca vs. Republic (51 SCRA 248
[1973]), viz:
We declare it to be a sound rule that where the citizenship of a party in a case is definitely
resolved by a court or by an
administrative agency, as a material issue in the controversy, after a full-blown hearing
with the active participation of the
Solicitor General or his authorized representative, and this finding or the citizenship of the
party is affirmed by this Court,
the decision on the matter shall constitute conclusive proof of such party's citizenship in any
other case or proceeding. But it is
made clear that in no instance will a decision on the question of citizenship in such cases be
considered conclusive or binding in
any other case or proceeding, unless obtained in accordance with the procedure herein stated.
Thus, in order that the doctrine of res judicata may be applied in cases of citizenship, the
following must be present: 1) a
person's citizenship must be raised as a material issue in a controversy where said person
is a party; 2) the Solicitor
General or his authorized representative took active part in the resolution thereof, and 3)
the finding or citizenship is

affirmed by this Court.

69
G.R. Nos. 95122-23 May 31, 1991
BIDIN, J.:
BOARD OF COMMISSIONERS (COMMISSION ON IMMIGRATION AND
DEPORTATION), BOARD OF SPECIAL INQUIRY, COMMISSIONER ANDREA D. This is a petition for certiorari and prohibition filed by the Solicitor General seeking 1)
DOMINGO, ASSOCIATE COMMISSIONER JORGE V. SARMIENTO, ACTING to set aside the Resolution/Temporary Restraining Order dated September 7, 1990,
ASSOCIATE COMMISSIONER REGINO R. SANTIAGO, MEMBERS OF THE issued by respondent Judge de la Rosa in Civil Case No. 90-54214 which denied
BOARD OF SPECIAL INQUIRY, ESTANISLAO CANTA, LEO MAGAHOM and petitioners' motion to dismiss and restrained petitioners from commencing or
BENJAMIN KALAW, petitioners, continuing with any of the proceedings which would lead to the deportation of
vs. respondent William Gatchalian, docketed as D.C. No. 90-523, as well as the Order of
HON. JOSELITO DELA ROSA, Presiding Judge, RTC Manila, Branch 29, WILLIAM respondent Judge Capulong dated September 6, 1990 in Civil Case No. 3431-V-90
T. GATCHALIAN,respondents. which likewise enjoined petitioners from proceeding with the deportation charges
against respondent Gatchalian, and 2) to prohibit respondent judges from further acting
BOARD OF COMMISSIONERS (COMMISSION ON IMMIGRATION AND in the aforesaid civil cases.
DEPORTATION), BOARD OF SPECIAL INQUIRY, COMMISSIONER ANDREA D.
DOMINGO, ASSOCIATE COMMISSIONER JORGE V. SARMIENTO, ACTING On October 23, 1990, respondent Gatchalian filed his Comment with Counter-Petition,
ASSOCIATE COMMISSIONER REGINO R. SANTIAGO, MEMBERS OF THE docketed as G.R. Nos. 96512-13, alleging lack of jurisdiction on the part of respondent
BOARD OF SPECIAL INQUIRY, ESTANISLAO CANTA, LEO MAGAHOM and Board of Commissioners, et al., over his person with prayer that he be declared a
BENJAMIN KALAW, petitioners, Filipino citizen, or in the alternative, to remand the case to the trial court for further
vs. proceedings.
HON. TERESITA DIZON CAPULONG, Presiding Judge, RTC Branch 172,
Valenzuela, Metro Manila, DEE HUA T. GATCHALIAN, SHERWING T. On December 13, 1990, petitioners filed their comment to respondent Gatchalian's
GATCHALIAN, KENNETH T. GATCHALIAN, REXLON T. GATCHALIAN, and counter-petition. The Court considers the comment filed by respondent Gatchalian as
WESLIE T. GATCHALIAN, respondents. answer to the petition and petitioners' comment as answer to the counter-petition and
gives due course to the petitions.
G.R. Nos. 95612-13 May 31, 1991
There is no dispute as to the following facts:
WILLIAM T. GATCHALIAN, petitioner,
vs. On July 12, 1960, Santiago Gatchalian, grandfather of William Gatchalian, was
BOARD OF COMMISSIONERS (COMMISSION ON IMMIGRATION AND recognized by the Bureau of Immigration as a native born Filipino citizen following the
DEPORTATION), et al., respondents. citizenship of his natural mother, Marciana Gatchalian (Annex "1", counter-petition).
Before the Citizenship Evaluation Board, Santiago Gatchalian testified that he has five
The Solicitor General for petitioners. (5) children with his wife Chu Gim Tee, namely: Jose Gatchalian, Gloria Gatchalian,
edesma, Saludo & Associates for respondent William Gatchalian. Francisco Gatchalian, Elena Gatchalian and Benjamin Gatchalian (Annex "2", counter-
Cervo and Tanay Law Office for respondent T.D. Capulong, D.H.T. Gatchalian, et al. petition).

On June 27, 1961, William Gatchali`an, then a twelve-year old minor, arrived in Manila
70
from Hongkong together with Gloria, Francisco, and Johnson, all surnamed Gatchalian. On March 14, 1973, the Board of Special Inquiry recommended to the then Acting
They had with them Certificates of Registration and Identity issued by the Philippine Commissioner Victor Nituda the reversal of the July 6, 1962 decision of the then Board
Consulate in Hongkong based on a cablegram bearing the signature of the then of Commissioners and the recall of the warrants of arrest issued therein (Annex "5",
Secretary of Foreign Affairs, Felixberto Serrano, and sought admission as Filipino counter-petition).
citizens. Gloria and Francisco are the daughter and son, respectively, of Santiago
Gatchalian; while William and Johnson are the sons of Francisco. On March 15, 1973, Acting Commissioner Nituda issued an order reaffirming the July
6, 1961 decision of the Board of Special Inquiry thereby admitting respondent
After investigation, the Board of Special Inquiry No. 1 rendered a decision dated July Gatchalian as a Filipino citizen and recalled the warrant of arrest issued against him
6, 1961, admitting William Gatchalian and his companions as Filipino citizens (Annex (Annex "6", counter-petition).
"C", petition). As a consequence thereof, William Gatchalian was issued Identification
Certificate No. 16135 by the immigration authorities on August 16, 1961 (Annex "D", On June 7, 1990, the acting director of the National Bureau of Investigation wrote the
petition). Secretary of Justice recommending that respondent Gatchalian along with the other
applicants covered by the warrant of exclusion dated July 6, 1962 be charged with
On January 24, 1962, the then Secretary of Justice issued Memorandum No. 9 setting violation of Sec. 37 (a), pars. 1 and 2, in relation to Secs. 45 (c), and (d) and (e) of
aside all decisions purporting to have been rendered by the Board of Commissioners on Commonwealth Act No. 613, as amended, also known as the Immigration Act of 1940
appeal or on review motu proprio of decisions of the Board of Special Inquiry. The (Annex "G", petition).
same memorandum directed the Board of Commissioners to review all cases where
entry was allowed on the ground that the entrant was a Philippine citizen. Among those On August 1, 1990, the Secretary of Justice indorsed the recommendation of the NBI to
cases was that of William and others. the Commissioner of Immigration for investigation and immediate action (Annex "20",
counter-petition).
On July 6, 1962, the new Board of Commissioners, after a review motu proprio of the
proceedings had in the Board of Special Inquiry, reversed the decision of the latter and On August 15, 1990, petitioner Commissioner Domingo of the Commission of
ordered the exclusion of, among others, respondent Gatchalian (Annex "E", petition). A Immigration and Deportation * issued a mission order commanding the arrest of
warrant of exclusion also dated July 6, 1962 was issued alleging that "the decision of respondent William Gatchalian (Annex "18", counter-petition). The latter appeared
the Board of Commissioners dated July 6, 1962 . . . has now become final and before Commissioner Domingo on August 20, 1990 and was released on the same day
executory (Annex "F", petition). upon posting P200,000.00 cash bond.

The actual date of rendition of said decision by the Board of Commissioners (whether On August 29, 1990, William Gatchalian filed a petition for certiorari and prohibition
on July 6, 1962 or July 20, 1962) became the subject of controversy in the 1967 case with injunction before the Regional Trial Court of Manila, Br. 29, presided by
of Arocha vs. Vivo (21 SCRA 532) wherein this Court sustained the validity of the respondent Judge dela Rosa, docketed as Civil Case No. 90-54214.
decision of the new Board of Commissioners having been promulgated on July 6, 1962,
or within the reglementary period for review. On September 4, 1990, petitioners filed a motion to dismiss Civil Case No. 90-54214
alleging that respondent judge has no jurisdiction over the Board of Commissioners
Sometime in 1973, respondent Gatchalian, as well as the others covered by the July 6, and/or the Board of Special Inquiry. Nonetheless, respondent judge dela Rosa issued
1962 warrant of exclusion, filed a motion for re-hearing with the Board of Special the assailed order dated September 7, 1990, denying the motion to dismiss.
Inquiry where the deportion case against them was assigned.
Meanwhile, on September 6, 1990, respondent Gatchalian's wife and minor children
71
filed before the Regional Trial Court of Valenzuela, Metro Manila, Br. 172, presided by Respondent, on the other hand, contends that petitioners are not quasi-judicial agencies
respondent judge Capulong Civil Case No. 3431-V-90 for injunction with writ of and are not in equal rank with Regional Trial Courts.
preliminary injunction. The complaint alleged, among others, that petitioners acted
without or in excess of jurisdiction in the institution of deportation proceedings against Under Sec. 21 (1) of Batas Pambansa Blg. 129, the Regional Trial Courts have
William. On the same day, respondent Capulong issued the questioned temporary concurrent jurisdiction with this Court and the Court of Appeals to issue "writs
restraining order restraining petitioners from continuing with the deportation of certiorari, prohibition, mandamus, quo warranto, habeas corpusand injunction which
proceedings against William Gatchalian. may be enforced in any part of their respective regions, . . ." Thus, the RTCs are vested
with the power to determine whether or not there has been a grave abuse of discretion
The petition is anchored on the following propositions: 1) respondent judges have no on the part of any branch or instrumentality of the government.
jurisdiction over petitioners (Board of Commissioners, et al.,) and the subject matter of
the case, appellate jurisdiction being vested by BP 129 with the Court of Appeals; 2) It is true that under Sec. 9 (3) of Batas Pambansa Blg. 129, the Court of Appeals is
assuming respondent judges have jurisdiction, they acted with grave abuse of discretion vested with —
in preempting petitioners in the exercise of the authority and jurisdiction to hear and
determine the deportation case against respondent Gatchalian, and in the process (3) Exclusive appellate jurisdiction over all final judgments, decisions,
determine also his citizenship; 3) respondent judge dela Rosa gravely abused his resolutions, order, or awards of Regional Trial Courts and quasi-judicial
discretion in ruling that the issues raised in the deportation proceedings are beyond the agencies, instrumentalities, board or commission, except those falling within the
competence and jurisdiction of petitioners, thereby disregarding the cases of Arocha vs. appellate jurisdiction of the Supreme Court in accordance with the Constitution,
Vivo and Vivo vs. Arca (supra), which put finality to the July 6, 1962 decision of the the provisions of this Act, and of sub-paragraph (1) of the third paragraph of
Board of Commissioners that respondent Gatchalian is a Chinese citizen; and 4) and sub-paragraph (4) of the fourth paragraph of Section 17 of the Judiciary Act
respondent judge Capulong should have dismissed Civil Case No. 3431-V-90 for of 1948.
forum-shopping.
It does not provide, however, that said exclusive appellate jurisdiction of the Court of
In his counter-petition, William Gatchalian alleges among others that: 1) assuming that Appeals extends to all quasi-judicial agencies. The quasi-judicial bodies whose
the evidence on record is not sufficient to declare him a Filipino citizen, petitioners decisions are exclusively appealable to the Court of Appeals are those which under the
have no jurisdiction to proceed with the deportation case until the courts shall have law, Republic Act No. 5434, or their enabling acts, are specifically appealable to the
finally resolved the question of his citizenship; 2) petitioners can no longer judiciously Court of Appeals (Presidential Anti-Dollar Salting Task Force vs. Court of Appeals,
and fairly resolve the question of respondent's citizenship in the deportation case 171 SCRA 348 [1989]; Lupangco vs. Court of Appeals, 160 SCRA 848 [1988]). Thus,
because of their bias, pre-judgment and prejudice against him; and 3) the ground for under Republic Act No. 5434, it is specifically provided that the decisions of the Land
which he is sought to be deported has already prescribed. Registration Commission (LRC), the Social Security Commission (SSC), Civil
Aeronautics Board (CAB), the Patent Office and the Agricultural Invention Board are
For purposes of uniformity, the parties herein will be referred to in the order the appealable to the Court of Appeals.
petitions were filed.
In the Presidential Anti-Dollar Salting Task Force (supra), this Court clarified the
Petitioners argue that under Sec. 9 (3) of BP 129, it is the Court of Appeals which has matter when We ruled:
exclusive appellate jurisdiction over all final judgments or orders of quasi-judicial
agencies, boards or commissions, such as the Board of Commissioners and the Board Under our Resolution dated January 11, 1983:
of Special Inquiry.
72
. . . The appeals to the Intermediate Appellate Court (now Court of As a rule, where legislation provides for an appeal from decisions of certain
Appeals) from quasi-judicial bodies shall continue to be governed by the administrative bodies to the Court of Appeals, it means that such bodies are co-
provisions of Republic Act No. 5434 insofar as the same is not equal with the Regional Trial Courts, in terms of rank and stature, and logically,
inconsistent with the provisions of B.P. Blg. 129. beyond the control of the latter. (Emphasis supplied)

The pertinent provisions of Republic Act No. 5434 are as follows: There are quasi-judicial agencies, as the National Labor Relations Commissions, whose
decisions are directly appealable to this Court. It is only when a specific law, as
Sec. 1. Appeals from specified agencies.— Any provision of existing Republic Act No. 5434, provides appeal from certain bodies or commissions to the
law or Rules of Court to the contrary notwithstanding, parties aggrieved Court of Appeals as the Land Registration Commission (LRC), Securities and
by a final ruling, award, order, or decision, or judgment of the Court of Exchange Commission (SEC) and others, that the said commissions or boards may be
Agrarian Relations; the Secretary of Labor under Section 7 of Republic considered co-equal with the RTCs in terms of rank, stature and are logically beyond
Act Numbered Six hundred and two, also known as the "Minimum the control of the latter.
Wage Law"; the Department of Labor under Section 23 of Republic Act
Numbered Eight hundred seventy-five, also known as the "Industrial However, the Bureau of Immigration (or CID) is not among those quasi-judicial
Peace Act"; the Land Registration Commission; the Social Security agencies specified by law whose decisions, orders, and resolutions are directly
Commission; the Civil Aeronautics Board; the Patent Office and the appealable to the Court of Appeals. In fact, its decisions are subject to judicial review
Agricultural Inventions Board, may appeal therefrom to the Court of in accordance with Sec. 25, Chapter 4, Book VII of the 1987 Administrative Code,
Appeals, within the period and in the manner herein provided, whether which provides as follows:
the appeal involves questions of fact, mixed questions of fact and law, or
questions of law, or all three kinds of questions. From final judgments or Sec. 25. Judicial Review.—(1) Agency decisions shall be subject to judicial
decisions of the Court of Appeals, the aggrieved party may appeal review in accordance with this chapter and applicable laws.
by certiorari to the Supreme Court as provided under Rule 45 of the
Rules of Court. xxx xxx xxx

Because of subsequent amendments, including the abolition of various special (6) The review proceeding shall be filed in the court specified in the statute or,
courts, jurisdiction over quasi-judicial bodies has to be, consequently, in the absence thereof, in any court of competent jurisdiction in accordance with
determined by the corresponding amendatory statutes. Under the Labor Code, the provisions on venue of the Rules of Court.
decisions and awards of the National Labor Relations Commission are final and
executory, but, nevertheless, reviewable by this Court through a petition Said provision of the Administrative Code, which is subsequent to B.P. Blg. 129 and
for certiorari and not by way of appeal. which thus modifies the latter, provides that the decision of an agency like the Bureau
of Immigration should be subject to review by the court specified by the statute or in
Under the Property Registration Decree, decision of the Commission of Land the absence thereof, it is subject to review by any court of competent jurisdiction in
Registration, en consulta, are appealable to the Court of Appeals. accordance with the provisions on venue of the Rules of Court.

The decisions of the Securities and Exchange Commission are likewise B.P. Blg. 129 did not intend to raise all quasi-judicial bodies to the same level or rank
appealable to the Appellate Court, and so are decisions of the Social Security of the RTC except those specifically provided for under the law as aforestated. As the
Commission. Bureau of Immigration is not of equal rank as the RTC, its decisions may be appealable
73
to, and may be reviewed through a special civil action for certiorari by, the RTC (Sec. SCRA 413 [1970]; Vivo vs. Montesa, 24 SCRA 155 [1967]). Judicial intervention,
21, (1) BP 129). however, should be granted only in cases where the "claim of citizenship is so
substantial that there are reasonable grounds to believe that the claim is correct. In
True, it is beyond cavil that the Bureau of Immigration has the exclusive authority and other words, the remedy should be allowed only on sound discretion of a competent
jurisdiction to try and hear cases against an alleged alien, and in the process, determine court in a proper proceeding (Chua Hiong vs. Deportation Board, supra; Co. vs.
also their citizenship (Lao Gi vs. Court of Appeals, 180 SCRA 756 [1989]). And a mere Deportation Board, 78 SCRA 107 [1977]). It appearing from the records that
claim of citizenship cannot operate to divest the Board of Commissioners of its respondent's claim of citizenship is substantial, as We shall show later, judicial
jurisdiction in deportation proceedings (Miranda vs. Deportation Board, 94 Phil. 531 intervention should be allowed.
[1954]).
In the case at bar, the competent court which could properly take cognizance of the
However, the rule enunciated in the above-cases admits of an exception, at least insofar proceedings instituted by respondent Gatchalian would nonetheless be the Regional
as deportation proceedings are concerned. Thus, what if the claim to citizenship of the Trial Court and not the Court of Appeals in view of Sec. 21 (1), BP 129, which confers
alleged deportee is satisfactory? Should the deportation proceedings be allowed to upon the former jurisdiction over actions for prohibition concurrently with the Court of
continue or should the question of citizenship be ventilated in a judicial proceeding? Appeals and the Supreme Court and in line with the pronouncements of this Court
In Chua Hiong vs. Deportation Board (96 Phil. 665 [1955]), this Court answered the in Chua Hiong and Co cases.
question in the affirmative, and We quote:
Ordinarily, the case would then be remanded to the Regional Trial Court. But not in the
When the evidence submitted by a respondent is conclusive of his citizenship, case at bar.1âwphi1 Considering the voluminous pleadings submitted by the parties and
the right to immediate review should also be recognized and the courts should the evidence presented, We deem it proper to decide the controversy right at this
promptly enjoin the deportation proceedings. A citizen is entitled to live in instance. And this course of action is not without precedent for "it is a cherished rule of
peace, without molestation from any official or authority, and if he is disturbed procedure for this Court to always strive to settle the entire controversy in a single
by a deportation proceeding, he has the unquestionable right to resort to the proceeding leaving no root or branch to bear the seeds of future litigation. No useful
courts for his protection, either by a writ of habeas corpus or of prohibition, on purpose will be served if this case is remanded to the trial court only to have its
the legal ground that the Board lacks jurisdiction. If he is a citizen and evidence decision raised again to the Court of Appeals and from there to this Court" (Marquez
thereof is satisfactory, there is no sense nor justice in allowing the deportation vs. Marquez, 73 Phil. 74; Keramic Industries, Inc. vs. Guerrero, 61 SCRA 265 [1974])
proceedings to continue, granting him the remedy only after the Board has Alger Electric, Inc. vs. Court of Appeals (135 SCRA 37 [1985]), citing Gayos vs.
finished its investigation of his undesirability. Gayos (67 SCRA 146 [1975]).

. . . And if the right (to peace) is precious and valuable at all, it must also be In Lianga Bay Logging Co., Inc. vs. Court of Appeals (157 SCRA 357 [1988]), We also
protected on time, to prevent undue harassment at the hands of ill-meaning or stated:
misinformed administrative officials. Of what use is this much boasted right to
peace and liberty if it can be availed of only after the Deportation Board has Remand of the case to the lower court for further reception of evidence is not
unjustly trampled upon it, besmirching the citizen's name before the bar of necessary where the court is in a position to resolve the dispute based on the
public opinion? (Emphasis supplied) records before it. On many occasions, the Court, in the public interest and the
expeditious administration of justice, has resolved actions on the merits instead
The doctrine of primary jurisdiction of petitioners Board of Commissioners over of remanding them to the trial court for further proceedings, such as where the
deportation proceedings is, therefore, not without exception (Calacday vs. Vivo, 33 ends of justice would not be subserved by the remand of the case or when
74
public interest demands an early disposition of the case or where the trial court a petition for a writ of habeas corpus filed on July 21, 1965 by Macario Arocha in
had already received all the evidence of the parties (Quisumbing vs. CA, 112 behalf of Pedro Gatchalian. Well settled is the rule that a person not party to a case
SCRA 703; Francisco, et al., vs. The City of Davao, et al., supra; Republic vs. cannot be bound by a decision rendered therein.
Security Credit & Acceptance Corp., et al., 19 SCRA 58; Samal vs. CA, supra;
Republic vs. Central Surety & Insurance Co., 25 SCRA 641). Neither can it be argued that the Board of Commissioners' decision (dated July 6, 1962)
finding respondent's claim to Philippine citizenship not satisfactorily proved,
Likewise in Tejones vs. Gironella (159 SCRA 100 [1988]), We said: constitute res judicata. For one thing, said decision did not make any categorical
statement that respondent Gatchalian is a Chinese. Secondly, the doctrine of res
Sound practice seeks to accommodate the theory which avoids waste of time, judicata does not apply to questions of citizenship (Labo vs. Commission on Elections
effort and expense, both to the parties and the government, not to speak of delay (supra); citing Soria vs. Commissioner of Immigration, 37 SCRA 213; Lee vs.
in the disposal of the case (cf. Fernandez vs. Garcia, 92 Phil. 592, 297). A Commissioner of Immigration, 42 SCRA 561 [1971]; Sia Reyes vs. Deportation Board,
marked characterstic of our judicial set-up is that where the dictates of justice so 122 SCRA 478 [1983]).
demand . . . the Supreme Court should act, and act with finality (Li Siu Liat vs.
Republic, 21 SCRA 1039, 1046, citingSamal vs. CA, 99 Phil. 230 and US vs. In Moy Ya Lim vs. Commissioner of Immigration (41 SCRA 292 [1971]) and in Lee
Gimenez, 34 Phil. 74.) (Beautifont, Inc. vs. Court of appeals, et al., Jan. 29, vs. Commissioner of Immigration (supra), this Court declared that:
1988; See also Labo vs. Commission on Elections, 176 SCRA 1 [1989]).
(e)verytime the citizenship of a person is material or indispensable in a judicial
Respondent Gatchalian has adduced evidence not only before the Regional Trial Court or administrative case, whatever the corresponding court or administrative
but also before Us in the form of public documents attached to his pleadings. On the authority decides therein as to such citizenship is generally not considered as res
other hand, Special Prosecutor Renato Mabolo in his Manifestation (dated September adjudicata, hence it has to be threshed out again and again as the occasion may
6, 1990; Rollo, p. 298, counter-petition) before the Bureau of Immigration already demand.
stated that there is no longer a need to adduce evidence in support of the deportation
charges against respondent. In addition, petitioners invoke that this Court's decision An exception to the above rule was laid by this Court in Burca vs. Republic (51 SCRA
in Arocha vs. Vivo and Vivo vs. Arca (supra), has already settled respondent's alienage. 248 [1973]), viz:
Hence, the need for a judicial determination of respondent's citizenship specially so
where the latter is not seeking admission, but is already in the Philippines (for the past We declare it to be a sound rule that where the citizenship of a party in a case is
thirty [30] years) and is being expelled (Chua Hiong vs. Deportation Board, supra). definitely resolved by a court or by an administrative agency, as a material issue
in the controversy, after a full-blown hearing with the active participation of the
According to petitioners, respondent's alienage has been conclusively settled by this Solicitor General or his authorized representative, and this finding or the
Court in the Arocha and Vivocases, We disagree. It must be noted that in said cases, the citizenship of the party is affirmed by this Court, the decision on the matter
sole issue resolved therein was the actual date of rendition of the July 6, 1962 decision shall constitute conclusive proof of such party's citizenship in any other case or
of the then board of Commissioners, i.e., whether the decision was rendered on July 6, proceeding. But it is made clear that in no instance will a decision on the
1962 or on July 20, 1962 it appearing that the figure (date) "20" was erased and over it question of citizenship in such cases be considered conclusive or binding in any
was superimposed the figure "6" thereby making the decision fall within the one-year other case or proceeding, unless obtained in accordance with the procedure
reglementary period from July 6, 1961 within which the decision may be reviewed. herein stated.
This Court did not squarely pass upon any question of citizenship, much less that of
respondent's who was not a party in the aforesaid cases. The said cases originated from Thus, in order that the doctrine of res judicata may be applied in cases of citizenship,
75
the following must be present: 1) a person's citizenship must be raised as a material As We held in Qua Chee Gan vs. Deportation Board (supra), "(t)he constitution does
issue in a controversy where said person is a party; 2) the Solicitor General or his not distinguish warrants between a criminal case and administrative proceedings. And
authorized representative took active part in the resolution thereof, and 3) the finding or if one suspected of having committed a crime is entitled to a determination of the
citizenship is affirmed by this Court. probable cause against him, by a judge, why should one suspected of a violation of an
administrative nature deserve less guarantee?" It is not indispensable that the alleged
Gauged by the foregoing, We find the pre-conditions set forth in inexistent in alien be arrested for purposes of investigation. If the purpose of the issuance of the
the Arocha and Vivo cases relied upon by petitioners. Indeed, respondent William warrant of arrest is to determine the existence of probable cause, surely, it cannot pass
Gatchalian was not even a party in said cases. the test of constitutionality for only judges can issue the same (Sec. 2, Art. III,
Constitution).
Coming now to the contention of petitioners that the arrest of respondent follows as a
matter of consequence based on the warrant of exclusion issued on July 6, 1962, A reading of the mission order/warrant of arrest (dated August 15, 1990; Rollo, p. 183,
coupled with the Arocha and Vivo cases (Rollo, pp. 33), the Court finds the same counter-petition) issued by the Commissioner of Immigration, clearly indicates that the
devoid of merit. same was issued only for purposes of investigation of the suspects, William Gatchalian
included. Paragraphs 1 and 3 of the mission order directs the Intelligence
Sec. 37 (a) of Commonwealth Act No. 613, as amended, otherwise known as the Agents/Officers to:
Immigration Act of 1940, reads:
xxx xxx xxx
Sec. 37. (a) The following aliens shall be arrested upon the warrant of the
Commissioner of Immigration or of any other officer designated by him for the 1. Make a warrantless arrest under the Rules of Criminal Procedure, Rule 113,
purpose and deported upon the warrant of the Commissioner of Sec. 5, for violation of the Immigration Act, Sec. 37, para. a; Secs. 45 and 46
Immigration after a determination by the Board of Commissioner of the Administrative Code;
existence of the ground for deportation as charged against the alien. (Emphasis
supplied) xxx xxx xxx

From a perusal of the above provision, it is clear that in matters of implementing the 3. Deliver the suspect to the Intelligence Division and immediately conduct
Immigration Act insofar as deportation of aliens are concerned, the Commissioner of custodial interrogation, after warning the suspect that he has a right to remain
Immigration may issue warrants of arrest only after a determination by the Board of silent and a right to counsel; . . .
Commissioners of the existence of the ground for deportation as charged against the
alien. In other words, a warrant of arrest issued by the Commissioner of Immigration, Hence, petitioners' argument that the arrest of respondent was based, ostensibly, on the
to be valid, must be for the sole purpose of executing a final order of deportation. A July 6, 1962 warrant of exclusion has obviously no leg to stand on. The mission
warrant of arrest issued by the Commissioner of Immigration for purposes of order/warrant of arrest made no mention that the same was issued pursuant to a final
investigation only, as in the case at bar, is null and void for being unconstitutional (Ang order of deportation or warrant of exclusion.
Ngo Chiong vs. Galang, 67 SCRA 338 [1975] citing Po Siok Pin vs. Vivo, 62 SCRA
363 [1975]; Vivo vs. Montesa, 24 SCRA 155; Morano vs. Vivo, 20 SCRA 562; Qua But there is one more thing that militates against petitioners' cause. As records indicate,
Chee Gan vs. Deportation Board, 9 SCRA 27 [1963]; Ng Hua To vs. Galang, 10 SCRA which petitioners conveniently omitted to state either in their petition or comment to
411; see also Santos vs. Commissioner of Immigration, 74 SCRA 96 [1976]). the counter-petition of respondent, respondent Gatchalian, along with others previously
covered by the 1962 warrant of exclusion, filed a motion for re-hearing before the
76
Board of Special Inquiry (BSI) sometime in 1973. Nonetheless, in said order it was found that the applicants therein have not
satisfactorily proven that they are the children and/or grandchildren of Santiago
On March 14, 1973, the Board of Special Inquiry, after giving due course to the motion Gatchalian. The status of Santiago Gatchalian as a Filipino was reiterated
for re-hearing, submitted a memorandum to the then Acting Commissioner Victor in Arocha and Arca (supra) where advertence is made to the "applicants being the
Nituda (Annex "5", counter-petition) recommending 1 the reconsideration of the July 6, descendants of one Santiago Gatchalian, a Filipino." (at p. 539).
1962 decision of the then Board of Commissioners which reversed the July 6, 1961
decision of the then Board of Special Inquiry No. 1 and 2 the lifting of the warrants of In the sworn statement of Santiago Gatchalian before the Philippine Consul in
arrest issued against applicants. The memorandum inferred that the "very basis of the Hongkong in 1961 (Annex "1" to the Comment of petitioners to Counter-Petition), he
Board of Commissioners in reversing the decision of the Board of Special Inquiry was reiterated his status as a Philippine citizen being the illegitimate child of Pablo Pacheco
due to a forged cablegram by the then Secretary of Foreign Affairs, . . ., which was and Marciana Gatchalian, the latter being a Filipino; that he was born in Manila on July
dispatched to the Philippine Consulate in Hong Kong authorizing the registration of 25, 1905; and that he was issued Philippine Passport No. 28160 (PA-No. A91196) on
applicants as P.I. citizens." The Board of Special Inquiry concluded that "(i)f at all, the November 18, 1960 by the Department of Foreign Affairs in Manila. In his affidavit of
cablegram only led to the issuance of their Certificate(s) of Identity which took the January 23, 1961 (Annex "5", counter-petition), Santiago reiterated his claim of
place of a passport for their authorized travel to the Philippines. It being so, even if the Philippine citizenship as a consequence of his petition for cancellation of his alien
applicants could have entered illegally, the mere fact that they are citizens of the registry which was granted on February 18, 1960 in C.E.B. No. 3660-L; and that on
Philippines entitles them to remain in the country." July 20, 1960, he was recognized by the Bureau of Immigration as a Filipino and was
issued Certificate No. 1-2123.
On March 15, 1973, then Acting Commissioner Nituda issued an Order (Annex "6",
counter-petition) which affirmed the Board of Special Inquiry No. 1 decision dated July The dissenting opinions of my esteemed brethrens, Messrs. Justices F.P. Feliciano and
6, 1961 admitting respondent Gatchalian and others as Filipino citizens; recalled the H.G. Davide, Jr., proposing to re-open the question of citizenship of Santiago
July 6, 1962 warrant of arrest and revalidated their Identification Certificates. Gatchalian at this stage of the case, where it is not even put in issue, is quite much to
late. As stated above, the records of the Bureau of Immigration show that as of July 20,
The above order admitting respondent as a Filipino citizen is the last official act of the 1960, Santiago Gatchalian had been declared to be a Filipino citizen. It is a final
government on the basis of which respondent William Gatchalian continually exercised decision that forecloses a re-opening of the same 30 years later. Petitioners do not even
the rights of a Filipino citizen to the present. Consequently, the presumption of question Santiago Gatchalian's Philippine citizenship. It is the citizenship of respondent
citizenship lies in favor of respondent William Gatchalian. William Gatchalian that is in issue and addressed for determination of the Court in this
case.
There should be no question that Santiago Gatchalian, grandfather of William
Gatchalian, is a Filipino citizen. As a matter of fact, in the very order of the BOC of Furthermore, petitioners' position is not enhanced by the fact that respondent's arrest
July 6, 1962, which reversed the July 6, 1961 BSI order, it is an accepted fact that came twenty-eight (28) years after the alleged cause of deportation arose. Section 37
Santiago Gatchalian is a Filipino. The opening paragraph of said order states: (b) of the Immigration Act states that deportation "shall not be effected . . . unless the
arrest in the deportation proceedings is made within five (5) years after the cause of
The claim to Filipino citizenship of abovenamed applicants is based on the deportation arises." In Lam Shee vs. Bengzon (93 Phil. 1065 [1953]), We laid down the
citizenship of one Santiago Gatchalian whose Philippine citizenship was consequences of such inaction, thus:
recognized by the Bureau of Immigration in an Order dated July 12, 1960.
(Annex "37", Comment with Counter-Petition). There is however an important circumstance which places this case beyond the
reach of the resultant consequence of the fraudulent act committed by the
77
mother of the minor when she admitted that she gained entrance into the the warrant of exclusion dated July 6, 1962 was already recalled and the Identification
Philippines by making use of the name of a Chinese resident merchant other certificate of respondent, among others, was revalidated on March 15, 1973 by the then
than that of her lawful husband, and that is, that the mother can no longer be the Acting Commissioner Nituda.
subject of deportation proceedings for the simple reason that more than 5 years
had elapsed from the date of her admission. Note that the above irregularity was It is also proposed in the dissenting opinions of Messrs. Justices Feliciano and Davide,
divulged by the mother herself, who in a gesture of sincerity, made an Jr., that the BOC decision dated July 6, 1962 and the warrant of exclusion which was
spontaneous admission before the immigration officials in the investigation found to be valid in Arocha should be applicable to respondent William Gatchalian
conducted in connection with the landing of the minor on September 24, 1947, even if the latter was not a party to said case. They also opined that under Sec. 37 (b) of
and not through any effort on the part of the immigration authorities. And the Immigration Act, the five (5) years limitation is applicable only where the
considering this frank admission, plus the fact that the mother was found to be deportation is sought to be effected under clauses of Sec. 37 (b) other than clauses 2, 7,
married to another Chinese resident merchant, now deceased, who owned a 8, 11 and 12 and that no period of limitation is applicable in deportations under clauses
restaurant in the Philippines valued at P15,000 and which gives a net profit of 2, 7, 8, 11 and 12.
P500 a month, the immigration officials then must have considered the
irregularity not serious enough when, inspire of that finding, they decided to The Court disagrees. Under Sec. 39 of the Immigration Act, it is reiterated that such
land said minor "as a properly documented preference quota immigrant" deportation proceedings should be instituted within five (5) years. Section 45 of the
(Exhibit D). We cannot therefore but wonder why two years later the same Act provides penal sanctions for violations of the offenses therein enumerated
immigration officials would reverse their attitude and would take steps to with a fine of "not more than P1,000.00 and imprisonment for not more than two (2)
institute deportation proceedings against the minor. years and deportation if he is an alien." Thus:

Under the circumstances obtaining in this case, we believe that much as the Penal Provisions
attitude of the mother would be condemned for having made use of an improper
means to gain entrance into the Philippines and acquire permanent residence Sec. 45. Any individual who—
there, it is now too late, not to say unchristian, to deport the minor after having
allowed the mother to remain even illegally to the extent of validating her (a) When applying for an immigration document personates another individual,
residence by inaction, thus allowing the period of prescription to set in and to or falsely appears in the name of deceased individual, or evades the
elapse in her favor. To permit his deportation at this late hour would be to immigration laws by appearing under an assumed name; fictitious name; or
condemn him to live separately from his mother through no fault of his thereby
leaving him to a life of insecurity resulting from lack of support and protection (b) Issues or otherwise disposes of an immigration document, to any person not
of his family. This inaction or oversight on the part of immigration officials has authorized by law to receive such document; or
created an anomalous situation which, for reasons of equity, should be resolved
in favor of the minor herein involved. (Emphasis supplied) (c) Obtains, accepts or uses any immigration document, knowing it to be false;
or
In the case at bar, petitioners' alleged cause of action and deportation against herein
respondent arose in 1962. However, the warrant of arrest of respondent was issued by (d) Being an alien, enters the Philippines without inspection and admission by
Commissioner Domingo only on August 15, 1990 — 28 long years after. It is clear that the immigration officials, or obtains entry into the Philippines by wilful, false,
petitioners' cause of action has already prescribed and by their inaction could not now or misleading representation or wilful concealment of a material fact; or
be validly enforced by petitioners against respondent William Gatchalian. Furthermore,
78
(e) Being an alien shall for any fraudulent purpose represent himself to be a however, considering that it is a harsh and extraordinary administrative proceeding
Philippine citizen in order to evade any requirement of the immigration laws; or affecting the freedom and liberty of a person, the constitutional right of such person to
due process should not be denied. Thus, the provisions of the Rules of Court of the
(f) In any immigration matter shall knowingly make under oath any false Philippines particularly on criminal procedure are applicable to deportation
statement or representations; or proceedings." (Lao Gi vs. Court of Appeals, supra). Under Sec. 6, Rule 39 of the Rules
of Court, a final judgment may not be executed after the lapse of five (5) years from the
(g) Being an alien, shall depart from the Philippines without first securing an date of its entry or from the date it becomes final and executory. Thereafter, it may be
immigration clearance certificates required by section twenty-two of this Act; or enforced only by a separate action subject to the statute of limitations. Under Art. 1144
(3) of the Civil Code, an action based on judgment must be brought within 10 years
(h) Attempts or conspires with another to commit any of the foregoing acts, from the time the right of action accrues.
shall be guilty of an offense, and upon conviction thereof, shall be fined not
more than one thousand pesos, and imprisoned for not more than two years, and In relation to Sec. 37 (b) of the Immigration Act, the rule, therefore, is:
deported if he is an alien. (Emphasis supplied)
1. Deportation or exclusion proceedings should be initiated within five (5) years after
Such offenses punishable by correctional penalty prescribe in 10 years (Art. 90, the cause of deportation or exclusion arises when effected under any other clauses other
Revised Penal Code); correctional penalties also prescribe in 10 years (Art. 92, Revised than clauses 2, 7, 8, 11 and 12 and of paragraph (a) of Sec. 37 of the Immigration Act;
Penal Code). and

It must be noted, however, that under Sec. 1, Act No. 3326 [1926], as amended, 2. When deportation or exclusion is effected under clauses 2, 7, 8, 11 and 12 of
(Prescription for Violations Penalized by Special Acts and Municipal Ordinances) paragraph (a) of Sec. 37, the prescriptive period of the deportation or exclusion
"violations penalized by special acts shall, unless otherwise provided in such acts, proceedings is eight (8) years.
prescribe in accordance with the following rules: . . .c) after eight years for those
punished by imprisonment for two years or more, but less than six years; . . ." In the case at bar, it took petitioners 28 years since the BOC decision was rendered on
July 6, 1962 before they commenced deportation or exclusion proceedings against
Consequently, no prosecution and consequent deportation for violation of the offenses respondent William Gatchalian in 1990. Undoubtedly, petitioners' cause of action has
enumerated in the Immigration Act can be initiated beyond the eight-year prescriptive already prescribed. Neither may an action to revive and/or enforce the decision dated
period, the Immigration Act being a special legislation. July 6, 1962 be instituted after ten (10) years (Art. 1144 [3], Civil Code).

The Court, therefore, holds that the period of effecting deportation of an alien after Since his admission as a Filipino citizen in 1961, respondent William Gatchalian has
entry or a warrant of exclusion based on a final order of the BSI or BOC are not continuously resided in the Philippines. He married Ting Dee Hua on July 1, 1973
imprescriptible. The law itself provides for a period of prescription. Prescription of the (Annex "8", counter-petition) with whom he has four (4) minor children. The marriage
crime is forfeiture or loss of the rights of the State to prosecute the offender after the contract shows that said respondent is a Filipino (Annex "8"). He holds passports and
lapse of a certain time, while prescription of the penalty is the loss or forfeiture by the earlier passports as a Filipino (Annexes "9", "10" & "11", counter-petition). He is a
government of the right to execute the final sentence after the lapse of a certain time registered voter of Valenzuela, Metro Manila where he has long resided and exercised
(Padilla, Criminal Law, Vol. 1, 1974, at p. 855). his right of suffrage (Annex 12, counter-petition). He engaged in business in the
Philippines since 1973 and is the director/officer of the International Polymer Corp.
"Although a deportation proceeding does not partake of the nature of a criminal action, and Ropeman International Corp. as a Filipino (Annexes, "13" & "14", counter-
79
petition). He is a taxpayer. Respondent claims that the companies he runs and in which presumed to be the same as those of the Philippines. In the case at bar, there being no
he has a controlling investment provides livelihood to 4,000 employees and proof of Chinese law relating to marriage, there arises the presumption that it is the
approximately 25,000 dependents. He continuously enjoyed the status of Filipino same as that of Philippine law.
citizenship and discharged his responsibility as such until petitioners initiated the
deportation proceedings against him. The lack of proof of Chinese law on the matter cannot be blamed on Santiago
Gatchalian much more on respondent William Gatchalian who was then a twelve-year
"The power to deport an alien is an act of the State. It is an act by or under the authority old minor. The fact is, as records indicate, Santiago was not pressed by the Citizenship
of the sovereign power. It is a police measure against undesirable aliens whose Investigation Board to prove the laws of China relating to marriage, having been
presence in the country is found to be injurious to the public good and domestic content with the testimony of Santiago that the Marriage Certificate was lost or
tranquility of the people" (Lao Gi vs. Court of Appeals, supra). How could one who has destroyed during the Japanese occupation of China. Neither was Francisco Gatchalian's
helped the economy of the country by providing employment to some 4,000 people be testimony subjected to the same scrutiny by the Board of Special Inquiry. Nevertheless,
considered undesirable and be summarily deported when the government, in its the testimonies of Santiago Gatchalian and Francisco Gatchalian before the Philippine
concerted drive to attract foreign investors, grants Special Resident Visa to any alien consular and immigration authorities regarding their marriages, birth and relationship
who invest at least US$50,000.00 in the country? Even assuming arguendo that to each other are not self-serving but are admissible in evidence as statements or
respondent is an alien, his deportation under the circumstances is unjust and unfair, if declarations regarding family reputation or tradition in matters of pedigree (Sec. 34,
not downright illegal. The action taken by petitioners in the case at bar is diametrically Rule 130). Furtheremore, this salutary rule of evidence finds support in substantive law.
opposed to settled government policy. Thus, Art. 267 of the Civil Code provides:

Petitioners, on the other hand, claim that respondent is an alien. In support of their Art. 267. In the absence of a record of birth, authentic document, final judgment
position, petitioners point out that Santiago Gatchalian's marriage with Chu Gim Tee in or possession of status, legitimate filiation may be proved by any other means
China as well as the marriage of Francisco (father of William) Gatchalian to Ong Chiu allowed by the Rules of Court and special laws. (See also Art. 172 of the
Kiok, likewise in China, were not supported by any evidence other than their own self- Family Code)
serving testimony nor was there any showing what the laws of China were. It is the
postulate advanced by petitioners that for the said marriages to be valid in this country, Consequently, the testimonies/affidavits of Santiago Gatchalian and Francisco
it should have been shown that they were valid by the laws of China wherein the same Gatchalian aforementioned are not self-serving but are competent proof of filiation
were contracted. There being none, petitioners conclude that the aforesaid marriages (Art. 172 [2], Family Code).
cannot be considered valid. Hence, Santiago's children, including Francisco, followed
the citizenship of their mother, having been born outside of a valid marriage. Similarly, Philippine law, following the lex loci celebrationis, adheres to the rule that a marriage
the validity of the Francisco's marriage not having been demonstrated, William and formally valid where celebrated is valid everywhere. Referring to marriages contracted
Johnson followed the citizenship of their mother, a Chinese national. abroad, Art. 71 of the Civil Code (now Art. 26 of the Family Code) provides that "(a)ll
marriages performed outside of the Philippines in accordance with the laws in force in
After a careful consideration of petitioner's argument, We find that it cannot be the country where they were performed, and valid there as such, shall also be valid in
sustained. this country . . ." And any doubt as to the validity of the matrimonial unity and the
extent as to how far the validity of such marriage may be extended to the consequences
In Miciano vs. Brimo (50 Phil. 867 [1924]; Lim and Lim vs. Collector of Customs, 36 of the coverture is answered by Art. 220 of the Civil Code in this manner: "In case of
Phil. 472; Yam Ka Lim vs. Collector of Customs, 30 Phil. 46 [1915]), this Court held doubt, all presumptions favor the solidarity of the family. Thus, every intendment of
that in the absence of evidence to the contrary, foreign laws on a particular subject are law or facts leans toward the validity of marriage, the indissolubility of the marriage
80
bonds, the legitimacy of children, the community of property during marriage, the respondent Gatchalian, he being a Filipino citizen; Civil Cases No. 90-54214 and 3431-
authority of parents over their children, and the validity of defense for any member of V-90 pending before respondent judges are likewise DISMISSED. Without
the family in case of unlawful aggression." (Emphasis supplied). Bearing in mind the pronouncement as to costs.
"processual presumption" enunciated in Miciano and other cases, he who asserts that
the marriage is not valid under our law bears the burden of proof to present the foreign SO ORDERED.
law.
Gutierrez, Jr., Gancayco, Sarmiento, Griño-Aquino and Medialdea, JJ., concur.
Having declared the assailed marriages as valid, respondent William Gatchalian Fernan, C.J., and Narvasa, J., concur in the result.
follows the citizenship of his father Francisco, a Filipino, as a legitimate child of the
latter. Francisco, in turn is likewise a Filipino being the legitimate child of Santiago
Gatchalian who (the latter) is admittedly a Filipino citizen whose Philippine citizenship
was recognized by the Bureau of Immigration in an order dated July 12, 1960.

Finally, respondent William Gatchalian belongs to the class of Filipino citizens


contemplated under Sec. 1, Article IV of the Constitution, which provides:

Sec. 1. The following are citizens of the Philippines:

(1) Those who are citizens of the Philippines at the time of the adoption of this
Constitution. . . .

This forecloses any further question about the Philippine citizenship of respondent
William Gatchalian.

The Court is not unaware of Woong Woo Yiu vs. Vivo (13 SCRA 552 [1965]) relied
upon by petitioners. The ruling arrived thereat, however, cannot apply in the case at bar
for the simple reason that the parties therein testified to have been married in China by
a village leader, which undoubtedly is not among those authorized to solemnize
marriage as provided in Art. 56 of the Civil Code (now Art. 7, Family Code).

Premises considered, the Court deems it unnecessary to resolve the other issues raised
by the parties.

WHEREFORE, G.R. Nos. 95122-23 is DISMISSED for lack of merit; G.R. Nos.
95612-13 is hereby GRANTED and respondent William Gatchalian is declared a
Filipino citizen. Petitioners are hereby permanently enjoined from continuing with the
deportation proceedings docketed as DC No. 90-523 for lack of jurisdiction over
81
NUVAL VS. GURRAY the subscribed affidavit
Facts: Exhibit F-1 before the board of election inspectors of precinct No. 1 of Balaoan, by virtue of
• This appeal was taken by the petitioner Gregorio Nuval from the judgment of the Court of which he was registered as
First Instance of La Union, an elector of the said precinct, having made use of the right of suffrage in said municipality in
upholding the defense of res judicata and dismissing the quo warranto proceedings instituted the general elections of
by the said Gregorio 1925. In his cedula certificates issued by himself as municipal treasurer of Balaoan from the
Nuval against Norbeto Guray and others, with costs against the petitioner. year 1923 to 1928,
• Gregorio Nuval filed, in his dual capacity as a voter duly qualified and registered in the included, he made it appear that his residence was the residential district of Balaoan. In the year
election list of the municipality of 1926, his wife and
Luna and as a duly registered candidate for the office of municipal president of said children who, up to that time, had lived in the municipality of Balaoan, went back to live in the
municipality, a petition against town of Luna in the
Norberto Guray asking for the exclusion of his name from the election list of said house of his wife's parents, due to the high cost of living in that municipality. Norberto Guray
municipality, not being a used to go home to Luna
qualified voter of said municipality and he had not resided therein for six months as house of his wife's parents, due to the high cost of living in that municipality. Norberto Guray
required by section 431 of the used to go home to Luna
said Administrative Code. in the afternoons after office hours, and there he passed the nights with his family. His children
• Norbeto Guray was elected to the office of municipal president of Luna by a plurality of studied in the public
votes, Gregorio Nuval school of Luna. In January, 1927, he commenced the construction of a house of strong
obtaining second place. materials in Luna, which
• Gregorio Nuval filed the present action of quo warranto asking that Norberto Guray be has not yet been completed, and neither be nor his family has lived in it. On February 1,
declared ineligible had a legal 1928, Norberto Guray
residence of one year previous to the election as required by section 2174 of the said applied for and obtained vacation leave to be spent in Luna, and on the 16th of the same month
Administrative Code in order to he filed his resignation
be eligible to an elective municipal office. by telegraph, which was accepted on the same day, also by telegraph. Nothwithstanding that he
• Norberto Guray had resided in the municipality of Luna, his birthplace, where he had was already provided
married and had held the with a cedula by himself as municipal treasurer of Balaoan on January 31, 1928, declaring him
office of municipal treasurer. On that date he was appointed municipal treasurer of Balaoan, resident of said town, he
Province of La Union. obtained another cedula from the municipality of Luna on February 20, 1928, which was dated
The rules of the provincial treasurer of La Union, to which Norberto Guray was subject as such January 15, 1928, in
municipal treasurer, which it is presented that he resided in the barrio of Victoria, municipality of Luna, Province of
require that municipality treasurers live continuously in the municipality where they La Union. On February
perform they official duties, 23, 1928, Norberto Guray applied for and obtained the cancellation of his name in the election
in order to be able to give an account of their acts as such treasurers at any time. In order to list of the municipality
qualify and be in a position of Balaoan, and on April 14, 1928, he applied for registration as a voter in Luna, alleging that
to vote as an elector in Balaoan in the general election of 1925, Norberto Guray asked for the he had been residing in
cancellation of his name said municipality for thirty years. For this purpose he made of the cedula certificate antedated.
in the election lists of Luna, where he had voted in the general elections of 1922, alleging as a Issue:
ground therefore the Whether or not Norberto Guray had the legal residence of one year immediately prior to the
following: "On the ground of transfer of any residence which took place on the 28th day general elections of June 5, 1928, in
of June, 1922. My order to be eligible to the office of municipal president of Luna, Province of La Union?
correct and new address is Poblacion, Balaoan, La Union;" and in order to be registered in Ruling:
82
It is an established rule that "where a voter abandons his residence in a state and acquires
one in another state, he cannot
again vote in the state of his former residence until he has qualified by a new period of
residence" (20 Corpus Juris, p. 71,
par. 28). "The term 'residence' as so used is synonymous with 'domicile,' which imports
not only intention to reside in a
fixed place, but also personal presence in that place, coupled with conduct indicative of
such intention." (People vs.
Bender, 144 N. Y. S., 145.)
Since Norberto Guray abandoned his first residence in the municipality of Luna and acquired
another in Balaoan, in order to vote
and be a candidate in the municipality of Luna, he needed to reacquire residence in the latter
municipality for the length of time
prescribed by the law, and for such purpose, he needed not only the intention to do so, but
his personal presence in said

municipality.

83
Code, as amended by Act No. 3387, Gregorio Nuval filed, in civil case No. 1442 of the
G.R. No. L-30241 December 29, 1928 Court of First Instance of La Union, in his dual capacity as a voter duly qualified and
registered in the election list of the municipality of Luna and as a duly registered
GREGORIO NUVAL, petitioner-appellant, candidate for the office of municipal president of said municipality, a petition against
vs. Norberto Guray asking for the exclusion of his name from the election list of said
NORBERTO GURAY, ET AL., respondents. municipality, not being a qualified voter of said municipality sine he had not resided
NORBERTO GURAY, appelllee. therein for six months as required by section 431 of the said Administrative Code.

Mabanag and Primicias, Gibbs and McDonough, and Mariano Alisangco for appellant. Proceedings were had upon the petition in accordance with sections 437 and 438 of the
Sison and Siguion and Franciscco Ortega for appellee. same Code, as amended by Act No. 3387, and Judge E. Araneta Diaz, rendered
judgment dismissing it because, in his opinion, Norberto Guray was a bona
fide resident of the municipality of Luna from Janury 1, 1927. As that order was not
appealable, Norberto Guray's name remained in the election list of the municipality of
VILLA-REAL, J.: Luna.

This appeal was taken by the petitioner Gregorio Nuval from the judgment of the Court The general election having been held on June 5, 1928, Norbeto Guray was elected to
of First Instance of La Union, upholding the defense of res judicata and dismissing the office of municipal president of Luna by a plurality of votes, Gregorio Nuval
the quo warranto proceedings instituted by the said Gregorio Nuval against Norbeto obtaining second place. On June 7, 1928, the municipal council of Luna, acting as the
Guray and others, with costs against the petitioner. municipal, Norberto Guray, elected to the office of municipal president of the said
municipality of Luna for the next triennium.
In support of his appeal, the appellant assign the following alleged errors as committed
by the trial court in its judgment, to wit: On June 18, 1928, Gregorio Nuval filed the present action of quo warranto as provided
in section 408 of the Administrative Code, as amended by Act No. 3387, asking that
1. The lower court erred in holding that the judgment rendered upon Gregorio Norberto Guray be declared ineligible had a legal residence of one year previuos to the
Nuval's petition for the cancellation of Norbeto Guray's name on the election election as required by section 2174 of the said Administrative Code in order to be
list of Luna is conclude and constitutes res judiata in the present case. eligible to an elective municipal office.

2. The trial court erred in not holding that Norbeto Guray at the time of his The question to be solved under the first assignment of error is whether or not the
election, was ineligible for the office of the residence in said municipality. judgment rendered in the case of the petition for the exclusion of Norberto Guray's
name from the election list of Luna, is res judicata, so as to prevent the institution and
3. The lower court erred in not finding in its judgment that the petitioner is prosecution of an action in quo warranto, which is now before us.
entitled to hold the office in question.
The procedure prescribed by section 437 of the Administrative Code, as amended by
In regard to the first assignment of error, the evidence adduced during the trial of the Act. No. 3387 is of a summary character and the judgment rendered therein is not
case shows: appealable except when the petition is tried before the justice of the peace of the capital
or the circuit judge, in which case it may be appealed to the judge of first instance, with
That on May 11, 1928, and within the period fixed by section 437 of the Administrative whom said two lower judges have concurrent jurisdiction.
84
The petition for execution was presented by Gregorio Nuval in his capacity as qualified facts:
voter of the municipality of Luna, and as a duly registered candidate for the office of
the president of said municipality, against Norberto Guray as a registered voter in the Up to June 27, 1922, Norberto Guray had resided in the municipality of Luna, his
election list of said municipality. The present proceedings of quo warranto was birthplace, where he had married and had held the office of municipal treasurer. On that
intreposed by Gregorio Nuval in his capacity as a registered candidate voted for the date he was appointed municipal treasurer of Balaoan, Province of La Union. The rules
office of municipal president of Luna, against Norberto Guray, as an elected candidate of the provincial treasurer of La Union, to which Norberto Guray was subject as such
for the same office. Therefore, there is no identity of parties in the two cases, since it is municipal treasurer, require that municipality treasurers live continuously in the
not enough that there be an identity of persons, but there must be an identity of municipality where they perform they official duties, in order to be able to give an
capacities in which said persons litigate. (Art. 1259 of the Civil Code; Bowler vs. account of their acts as such treasurers at any time. In order to qualify and be in a
Estate of Alvarez, 23 Phil., 561; 34 Corpus Juris, p. 756, par. 1165.) position to vote as an elector in Balaoan in the general election of 1925, Norberto
Guray asked for the cancellation of his name in the election lists of Luna, where he had
In said case for the petition for the exclusion, the object of the litigation, or the litigious voted in the general elections of 1922, alleging as a ground therefore the following:
matter was the conclusion of Norberto Guray as a voter from the election list of the "On the ground of transfer of any residence which took place on the 28th day of June,
municipality of Luna, while in the present quo warranto proceeding, the object of the 1922. My correct and new address is Poblacion, Balaoan, La Union;" and in order to be
litigation, or the litigious matter in his exclusion or expulsion from the office to which registered in the subscribed affidavit Exhibit F-1 before the board of election inspectors
he has been elected. Neither does there exist, then, any identity in the object of the of precinct No. 1 of Balaoan, by virtue of which he was registered as an elector of the
litigation, or the litigious matter. said precinct, having made use of the right of suffrage in said municipality in the
general elections of 1925. In his cedula certificates issued by himself as municipal
In said case of the petition for exclusion, the cause of action was that Norberto Guray treasurer of Balaoan from the year 1923 to 1928, included, he made it appear that his
had not the six months' legal residence in the municipality of Luna to be a qualified residence was the residential district of Balaoan. In the year 1926, his wife and children
voter thereof, while in the present proceedings of quo warranto, the case of this action who, up to that time, had lived in the municipality of Balaoan, went back to live in the
is that Norberto Guray has not the one year's legal residence required for the eligibility town of Luna in the house of his wife's parents, due to the high cost of living in that
to the office of municipal president of Luna. Neither does there exist, therefore, identity municipality. Norberto Guray used to go home to Luna in the afternoons after office
of causes of action. hours, and there he passed the nights with his family. His children studied in the public
school of Luna. In January, 1927, he commenced the construction of a house of strong
In order that res judicata may exist the following are necessary: (a) Identity of parties; materials in Luna, which has not yet been completed, and neither be nor his family has
(b) identity of things; and (c) identity of issues (Aquino vs. Director of Lands, 39 Phil., lived in it. On February 1, 1928, Norberto Guray applied for and obtained vacation
850). And as in the case of the petition for exclusion and in the present quo leave to be spent in Luna, and on the 16th of the same month he filed his resignation by
warranto proceeding, as there is no identity either of parties, or of things or litigious telegraph, which was accepted on the same day, also by telegraph. Nothwithstanding
matter, or of issues or causes of action, there is no res judicata.1awphi1.net that he was already provided with a cedula by himself as municipal treasurer of
Balaoan on January 31, 1928, declaring him resident of said town, he obtained another
For the above considerations, the trial court erred in holding that the judgment rendered cedula from the municipality of Luna on February 20, 1928, which was dated January
in the case on the petition of Gregorio Nuval asking for the cancellation of Norberto 15, 1928, in which it is presented that he resided in the barrio of Victoria, municipality
Guray's name in the election list of Luna is conclusive and constitutes res judicata in of Luna, Province of La Union. On February 23, 1928, Norberto Guray applied for and
the present case. obtained the cancellation of his name in the election list of the municipality of Balaoan,
and on April 14, 1928, he applied for registration as a voter in Luna, alleging that he
With respect to the second assignment of error, the evidence establishes the following had been residing in said municipality for thirty years. For this purpose he made of the
85
cedula certificate antedated. in the latter's house that only in the month of January, 1927, did he begin the
construction of a house of strong materials, which is not yet completed, nor occupied
In view of the facts just related, the question arises whether or not Norberto Guray had by himself or his family, His aftrenoon tips to Luna, according to his own explanation
the legal residence of one year immediately prior to the general elections of June 5, given to the provincial treasurer, were made for purpose of visiting his sick father. His
1928, in order to be eligible to the office of municipal president of Luna, Province of own act in recording in his cedula certificates for the years 1927 and 1928 issued by
La Union. himself in his favor as municipal treasurer of Balaoan, that his place of residene was
that municipality, and in taking out a new cedula in the municipality of Luna of
There is no question but that when Norberto Guray accepted and assumed the office of February 20, 1928, and having the date of its issuance surreptitiuosly put back to
municipal treasurer of Balaoan, La Union, he transferred his residence from the January 15 1928, show that until the date of his resignation he did not consider himself
municipality of Luna to that of Balaoan. as a resident of the municipality of Luna. The fact that his wife and children lived in
Luna not in his own house but in that of his wife's father since the year 1926, cannot be
The only question to determine refers to the date when he once more established his looked upon as a change of residence, since a change of residence requires an actual
residence in the municipality of Luna. and deliberate abandonment of the former (20 Corpus Juris, p. 71) and one cannot have
two legal residences at the same time.
It is an established rule that "where a voter abandons his residence in a state and
acquires one in another state, he cannot again vote in the state of his former residence The present case is different from that of Doctor Apacible cited by the appellee in his
until he has qualified by a new period of residence" (20 Corpus Juris, p. 71, par. 28). brief. Doctor Apacible never had abandoned his legal residence in the Province of
"The term 'residence' as so used is synonymous with 'domicile,' which imports not only Batangas, nothwithstanding that he had been living with his family in the City of
intention to reside in a fixed place, but also personal presence in that place, coupled Manila, taking out his cedula certificates here, but he never exercised the right of
with conduct indicative of such intention." (People vs. Bender, 144 N. Y. S., 145.) suffrage here. Norberto Guray abandoned his legal residencce in the municipality of
Luna, transferring it to the municipality of Balaoan by reason and an account of the
Since Norberto Guray abandoned his first residence in the municipality of Luna and requirements of the rules of the provincial treasurer of La Union, under whose
acquired another in Balaoan, in order to vote and be a candidate in the municipality of jurisdiction is said municipality, exercising his right of suffrage in the
Luna, he needed to reacquire residence in the latter municipality for the length of time latter.1awphi1.net
prescribed by the law, and for such purpose, he needed not only the intention to do so,
but his personal presence in said municipality. For the foregoing considerations, we are of opinion and so hold in fact and in law
Norberto Guray only abandoned his legal residence in the Municipality of Balaoan, and
By reason of his office as municipal treasurer of Balaoan and on account of the rules of began to acquire another in the municipality of Luna from Febraury 16, 1928, when he
the provincial treasurer of La Union, under whose jurisdiction was such municipality, filed his resignation from the office of municipal treasurer of Balaoan which he had
Norberto Guray had to reside and in fact resided in said municipality until the 6th of been holding, and which resignation was accepted; and on being elected municipal
February, 1928 when he filed his resignation from his office, which was accepted on president of Luna in the general elections of June 5, 1928, he had not reacquired the
the same date. The fact that his family moved to the municipality of Luna in the year legal residence necessary to be validly elected to said office.
1926 in order to live there in view of the high cost of living in balaoan; the fact that his
children studied in the public shool of said town; the fact that on afternoons after hours By virtue whereof, the election of respondent-appellee Norberto Guray to the office of
he went home to the municipality of Luna and there passed the night with his family, municipal president of Luna is hereby held to be unlawful and quashed and, in
are not in themselves alone sufficient to show that from said year he had transfered his consequence, he has no right to take possession of said office, petitioner Gregorio
residence to said municipality, since his wife and children lived with his father-in-law, Nuval being the one legally elected to said office with a right to take possession
86
thereof, having secured second place in the election. With costs against the respondent. candidate occupying the second place has been elected, even if he were eligible, since
So ordered. the law only authorizes a declaration of election in favor of the person who has
obtained a plurality of votes, and has presented his certificate of candidacy. In the
Avanceña, C. J., Ostrand, Johns and Romualdez, JJ., concur. second case, the court determines who has been legally appointed and can and ought to
Villamor, J., dissents. declare who is entitled to occupy the office.

RULING ON THE MOTION FOR RECONSIDERATION In view of the foregoing, we are of opinion that the judgment rendered in this case on
December 29, 1928, should be, and is hereby, amended, eliminating from the
February 1, 1929 dispositive part thereof, the holding that Gregorio Nuval is the one who has been
legally elected, so as to read as follows:
VILLA-REAL, J.:
By virtue whereof, the election of respondent-appellee Norberto Guray to the
This is a motion praying for the reasons given that the judgment rendered in this case office of Municipal president of Luna, is hereby declared unlawful and quashed
on December 29, 1928 be reconsidered, and another rendered affirming the judgment and, consequently, that he has no right to take possession of said office, with
appealed from. costs against said respondent.

In regard to the grounds of the motion with reference to the defence of res judicata, as So ordered.
the movant does not adduce any new argument in support thereof, and inasmuch as this
court has already discussed question at length, we find no sufficient reason to grant the Avanceña, C. J., Malcolm, Johns and Romualdez, JJ., concur.
motion on said grounds.

As to the other grounds touching this court's holding that Gregorio Nuval is the one
who has been legally elected to the office of municipal president of Luna, La Union,
and entitled to take possession thereof, having received second place, we consider them
meritorious, for the reason that 408 of the Election Law, providing the remedy in case a
person not eligible should be elected to a provincial or municipal office, does not
authorize that it be declared who has been legally elected, thus differing from section Separate Opinions
479 of the law, which contains such an authorization, and for the reason, furthermore,
that section 477 of the said law provides that only those who have obtained a plurality
of votes, and have presented their certificates of candidacy may be certified as elected
to municipal offices. Elective offices are by nature different from the appointive VILLAMOR, J., dissenting:
offices. The occupation of the first depends on the will of the elector, while that of the
second depends on the will of the authority providing for it. In quo In consequence of my dissenting opinion from the decision in this case, I am compelled
warranto proceedings referring to offices filled by election, what is to be determined is to dissent likewise from the ruling on the motion for reconsideration. And I take this
the eligibility of the candidate elect, while in quo warranto proceedings referring to opportunity of stating the grounds of my dissent. In the opinion prumulgated on
offices filled by appointment, what is determined is the legality of the appointment. In December 29, 1928, among other things, the following was stated: "Proceedings were
the first case when the person elected is ineligible, the court cannot declare that the had upon the petition in accordance with section 437 and 438 of the same Code, as
87
amended by Act 3387, and Judge E. Araneta Diaz, rendered judgment dismissing it cedula certificate antedated."
because, in his opinion, Norberto Guray was a bona fide resident of the municipality of
Luna from January 1, 1927. As that order was not appealable Norberto Guray's name Considering the facts that related in the majority opinion, I believed that, setting aside
remained in the election list of the municipality of Luna." technicalities, the question of the residence of the protestee-appellee Guray was
decided by the court holding that he was a bona fide resident of the municipality of
The same decision states: "In said case of the petition for the exclusion, the object of Luna, since January 1, 1927. I believe it is plain that Norberto Guray's residence
the litigation, or the litigious matter was the exclusion of Norberto Guray as a voter follows him as the shadow follows the body that casts it, whether it be in his capacity
from the election list of the municipality of Luna, while in the present quo as voter, or in his eligibility for the office of municipal president. The fact is that the
warranto proceeding, the object of the litigation, or the litigious matter in his exclusion Court of La Union found Guray to be a bona fide resident of the municipality of Luna
or expulsion from the office to which he has been elected. Neither does this exist, then, since January 1, 1927. From that date until the general elections (June 5, 1928), I
any identity in the object of the litigation, or the litigious matter. believe more than one year has elapsed, and consequently, Guray had the legal
residence of over one year at least, in the municipality of Luna at the time of his
In said case of the petition for exclusion, the cause of action was that Norberto Guray election as municipal president.
had not the six months' legal residence in the municipality of Luna to be qualified voter
therefor, while in the present proceeding of quo warranto, the cause of action is that In this jurisdiction the courts have ever looked upon resident as purely a matter of
Norberto Guray has not the one year legal residence required for eligibility to the office intention, manifested by the acts, conduct and circumstances of the person choosing a
of the municipal president of Luna. Neithe does there exist, therefore, identity of causes place as his permanent dwelling place and home. It was so understood by the
of action. Philippine Commission in discussing the first election law, whose provisions upon the
residence of voters and eligibles have not been amended up to the present time. The
Further on, it states: "In the year 1926, his wife and children who, up to that time had domicile or legal residence has been define as the place where a person has a principal
lived in the municipality of Balaoan, went back to live in the town of Luna in the house house or habitation, or where he kepts his family and his chief place of business. The
of his wife's parents. Due to the high cost of living in that municipality. Norberto Guray intention in every case is the real object of investigation. If a man leaves his town and
used to go home to Luna in the afternoons after office hours, and there he passed the removes to another by reason of his business, but with the intention to return to it, he
night with his family. His children studied in the public school of Luna. In January, has lost his residence in said town. The mere change of dwelling place does not
1927, he commenced the construction of a house of strong materials in Luna, which has involved a change of residence if it be not accompanied by intention. Hence it has been
not been completed, and neither he nor his family has lived in it. On February 1, 1928, held that if one has removed to another town solely by virtue of his appointment as the
Norberto Guray applied for and obtained vacation leave to be spent in Luna, and on the municipal treasurer, but with the intention to return to his original town, he has not lost
16th of the same month he filed his resignation by telegraph, which was accepted on his residence in his original town. The words good faith accompanying the word
the same day, also by telegraph. Nothwithstanding that he was already provided with a residence must be taken as a description of the state of mind of the person claiming
cedula issued by himself as municipal treasurer of Balaoan on January 31, 1928, residence. For example, if a man removes to province with the sole object of remaining
declaring him a resident of the said town, he obtained another cedula from the one year in order to become a candidate for the governorship, he may be held to be a
municipality of Luna on February 20, 1928, which was dated January 15, 1928, in resident in bad faith, while he may be bodily absent from the province for a long time
which it was presented that he resided in the barrio of Victoria, municipality of Luna, and yet have in his mind the constant intention to return. If it be so, he would be a
Province of La Union. On February 23, 1928, Norberto Guray applied for and obtained resident in good faith of the place he had left.
the cancellation of his name in the election list of the municipality of Balaoan, and on
April 14, 1928, he applied for the registration as a voter in Luna, alleging that he had It is evident that bodily presence in a place, accompanied by the intention to live
been residing in said municipality for thirty years. For this purpose he made use of the therein forever, establishes domicile or residence. But bodily presence in this place is
88
not residence; it is a mere proof, if you wish, tending to demonstrate residence. At provisions of section 408 of the Election Law, as amended, by the vice-
most, personal presence is presumptive proof that a person intends to reside where he president elect of the municipality, who challenged the right of the municipal
is. But such a presumption may be rebutted by other acts, conduct, and circumstances president elect, to the position to which elected on the ground that the municipal
of the person, clearly showing his intention to establish his home elsewhere. The herein president was ineligible, cannot be successfully maintained.
protestee-appellee Guray, at least January, 1927, kept his family in the municipality of
Luna, he went there at night to sleep with them, his children studied in the public 2. The Election Law makes use of the terms 'qualified voter in his municipality,'
school of said municipality, and he had a house of strong materials built in Luna, and qualified elector therein. To be a qualified voter, does not necessarily mean
although it was not yet completed, as stated in the majority opinion. But all these acts that a person must be a registered voter. It is sufficient for the candidate to
show his intention to establish his legal residence in the municipality of Luna. possess all of the qualifications prescribed in sectio 431 and none of the
disqualifications prescribed in section 432. The fact that a candidate failed to
The fact of having paid his personal cedula in Balaoan is of trivial importance in register as an elector in the municipality does not deprive him of the right to
showing legal residence, for, as Governor-General Smith said in the resolution of the become a candidate to be voted for.
protest against the election of the provincial governor of Batangas, Apacible, hundreds
of persons have taken out their cedulas outside of their residential province without 3. One may be qualified voter without exercising the right to vote. Registering
having lost their legal residence as a result thereof. Governor-General Smith himself, does not confer the right; it is but a condition presented to the exercise of the
who lived nine years in the Philippines, and took out his cedula in Manila, had his right. Registration regulates the exercise of the right of the suffrage. It is not a
residence in San Francisco, California, because his intention was to go back and live qualification of such right.
there permanently.
4. The question of the residence for the purposes of the Election Law is largely
There is no further need of amassing citations, but let a recently decided case be one of intention.
remembered, namely, the quo warranto proceeding of Ira vs. Abano (p. 380. ante). The
facts in that case are: Maximo Abano is a native of the municipality of Meycauayan, In another, and yet a more recent case (Vivero vs. Murillo, G. R. No. 30271) 1, it was
Bulacan. At the competent age he removed to Manila to complete his education. While held: "A student living with his parents in a certain barrio of a municipality, which
living temporarily in Manila, Abano registered as a voter there. Shortly after having barrio is latter separated to be organized as an independent municipality, who for
been admitted to the bar and after the death of his father, Abano went back to live in several years pursues his studies in several provinces of the archipelago, supported by
Meycauayan. From May 10, 1927, up to the present Abano has considered himself a his parents, returning to the latters' home during his vacations in the newly organized
resident of Meycauayan. When in 1928 the election where about to be held, he filed a municipality, does not lose his residence in said municipality, either on account of
petition for the cancellation of his registration in Manila, which was dated April 3, having resided in different provinces as a student, or of having registered as a voter in
1928, but his petition was denied by the city officials because it had not been deposited the former municipality and is eligible as municipal president of the new municipality
in the mails on or before April 4, 1928. Nevertheless, Abano presented himself as a even if his registration as a voter in the municipality to which the new one originally
candidate of the municipal presidency in Meycauayan in the 1928 elections and was by belonged has not been cancelled."
popular vote elected thereto. In that case it was held:
The same view of the question of legal residence has been sustained by the Philippine
1. A candidate who was elected to the office of municipal president and who at Assembly in the protest against the election of the representative of the southern district
the time the election was registered as a voter of Manila and not the of Manila, Fernando Guerrero, and in that of the representative for the second district
municipality in which he was a candidate, is nevertheless eligible to the office, of Manila, Luciano Cinco. (Journal of Sessions of the Assembly, December 6, 1917,
and proceedings in the nature od quo warranto instituted by virtue of the and November 29, 1927, respectively.)
89
Wherefore, I am of opinion that the motion for reconsideration should be granted on
the ground that the protestee and appellee had the legal residence to become eligible to
the office of the municipal president of the municipality of Luna. With respect the
second ground of the motion for reconsideration, I agree with the majority opinion that
a candidate who received second place in the election cannot be declared elected to said
office simply because the one who received first place turned out to be ineligible.

90
VELILLA VS. POSADA Ida M. Palmer through her
Facts: attorney.
• That Arthur Graydon Moody died in Calcutta, India, on February 18, 1931. • The parties reserve their right to introduce additional evidence at the hearing of the present
• That Arthur Graydon Moody executed in the Philippine Islands a will, by virtue of which case.
will, he bequeathed all his • Manila, August 15, 1933.
property to his only sister, Ida M. Palmer, who then was and still is a citizen and resident of • In addition to the foregoing agreed statement of facts, both parties introduced oral and
the State of New York, documentary evidence from which
United States of America. it appears that Arthur G. Moody, an American citizen, came to the Philippine Islands in 1902 or
• That on February 24,1931, a petition for appointment of special administrator of the estate of 1903 and engaged
the deceased Arthur actively in business in these Islands up to the time of his death in Calcutta, India, on February
Graydon Moody was filed by W. Maxwell Thebaut with the Court of First Instance of Manila. 18, 1931. He had no
• That subsequently or on April 10, 1931, a petition to the will of the deceased Arthur Graydon business elsewhere and at the time of his death left an estate consisting principally of bonds and
Moody, and the same was, shares of stock of
after hearing, duly probated by the court in a decree dated May 5, 1931. corporations organized under the laws of the Philippine Islands, bank deposits and other
• That on July 14, 1931, Ida M. Palmer was declared to be the sole and only heiress of the intangibles and personal
deceased Arthur Graydon property valued by the commissioners of appraisal and claims at P609,767.58 and by the
Moody Collector of Internal Revenue
• That the property left by the late Arthur Graydon Moody consisted principally of bonds and for the purposes of inheritance tax at P653,657.47. All of said property at the time of his
shares of stock of death was located and had
corporations organized under the laws of the Philippine Islands, bank deposits and other its situs within the Philippine Islands. So far as this record shows, he left no property of
personal properties. any kind located
• That on July 22, 1931, the Bureau of Internal Revenue prepared for the estate of the late anywhere else. In his will,he made a statement that: Arthur G. Moody, a citizen of the United
Arthur Graydon Moody States of America,
an inheritance tax return. residing in the Philippine Islands, hereby publish and declare the following as my last Will and
• That on September 9, 1931, an income tax return for the fractional period from January 1, Testament . . ..
1931 to June 30, 1931, was also Ruling:
prepared by the Bureau of Internal Revenue for the estate of the said deceased Arthur Graydon To effect the abandonment of one's domicile, there must be a deliberate and provable
Moody. choice of a new domicile, coupled
• That on December 3, 1931, the committee on claims and appraisals filed with the court its with actual residence in the place chosen, with a declared or provable intent that it should
report. be one's fixed and permanent
• That on November 4, 1931, and in answer to the letter mentioned in the preceding paragraph, place of abode, one's home. There is a complete dearth of evidence in the record that Moody
the Bureau of Internal ever established a new domicile in
Revenue addressed to the attorney for Ida M. Palmer another letter, copy of which marked a foreign country.
Exhibit NN is hereto Finding no merit in any of the assignments of error of the appellant, the court affirm the
attached and made a part hereof. judgment of the trial court, first, because
• That the estate of the late Arthur Graydon Moody paid under protest the sum of P50,000 on the property in the estate of Arthur G. Moody at the time of his death was located and had its
July 22, 1931, and the other situs within the Philippine Islands
sum of P40,019.75 on January 19, 1932, making assessment for inheritance tax and the sum of and, second, because his legal domicile up to the time of his death was within the
P13,001.41 covers the Philippine Islands.
assessment for income tax against said estate.
• That on January 21, 1932, the Collector of Internal Revenue overruled the protest made by
91
[G.R. No. 43314. December 19, 1935.]
"I. That Arthur Graydon Moody died in Calcutta, India, on February 18, 1931.
A. L. VELILLA, administrator of the estate of Arthur Graydon Moody, Plaintiff-
Appellant, v. JUAN POSADAS, JR., Collector of Internal Revenue, Defendant- "II. That Arthur Graydon Moody executed in the Philippine Islands a will, certified
Appellee. copy of which marked Exhibit AA is hereto attached and made a part hereof, by virtue
of which will, he bequeathed all his property to his only sister, Ida M. Palmer, who then
Ohnick & Opisso for Appellant. was and still is a citizen and resident of the State of New York, United States of
America.
Solicitor-General Hilado for Appellee.
"III. That on February 24, 1931, a petition for appointment of special administrator of
SYLLABUS the estate of the deceased Arthur Graydon Moody was filed by W. Maxwell Thebaut
with the Court of First Instance of Manila, the same being designated as case No.
1. INHERITANCE TAX; DOMICILE OF TAXPAYER. — To effect the abandonment 39113 of said court. Copy of said petition marked Exhibit BB is hereto attached and
of one’s domicile, there must be a deliberate and provable choice of a new domicile, made a part hereof.
coupled with actual residence in the place chosen, with a declared or provable intent
that it should be one’s fixed and permanent place of abode, one’s home. There is a "IV. That subsequently or on April 10, 1931, a petition was filed by Ida M. Palmer,
complete dearth of evidence in the record that M ever established a new domicile in a asking for the probate of said will of the deceased Arthur Graydon Moody, and the
foreign country. same was, after hearing, duly probated by the court in a decree dated May 5, 1931.
Copies of the petition and of the decree marked Exhibits CC and DD, respectively, are
2. INHERITANCE AND INCOME TAXES. — As M’s legal domicile at the time of his hereto attached and made parts hereof.
death was the Philippine Islands and his estate had its situs here, the inheritance and
income taxes here involved were lawfully collected. "V. That on July 14, 1931, Ida M. Palmer was declared to be the sole and only heiress
of the deceased Arthur Graydon Moody by virtue of an order issued by the court in said
case No. 39113, copy of which marked Exhibit EE is hereto attached and made a part
DECISION hereof; and that during the hearing for the declaration of heirs, Ida M. Palmer presented
as evidence a letter dated February 28, 1925, and addressed to her by Arthur Graydon
Moody, copy of which marked Exhibit FF is hereto attached and made a part hereof.
BUTTE, J.:
"VI. That the property left by the late Arthur Graydon Moody consisted principally of
bonds and shares of stock of corporations organized under the laws of the Philippine
This is an appeal from a judgment of the Court of First Instance of Manila in an action Islands, bank deposits and other personal properties, as are more fully shown in the
to recover from the defendant-appellee as Collector of Internal Revenue the sum of inventory of April 17, 1931, filed by the special administrator with the court in said
P77,018,39 as inheritance taxes and P13,001.41 as income taxes assessed against the case No. 39113, certified copy of which inventory marked Exhibit GG is hereto
estate of Arthur G. Moody, deceased. attached and made a part hereof. This stipulation does not, however, cover the
respective values of said properties for the purpose of the inheritance tax.
The parties submitted to the court an agreed statement of facts as
follows:jgc:chanrobles.com.ph "VII. That on July 22, 1931, the Bureau of Internal Revenue prepared for the estate of
92
the late Arthur Graydon Moody an inheritance tax return, certified copy of which "XVI. The parties reserve their right to introduce additional evidence at the hearing of
marked Exhibit HH is hereto attached and made a part hereof. the present case.

"VIII. That on September 9, 1931, an income tax return for the fractional period from "Manila, August 15, 1933."cralaw virtua1aw library
January 1, 1931 to June 30, 1931, certified copy of which marked Exhibit II is hereto
attached and made a part hereof, was also prepared by the Bureau of Internal Revenue In addition to the foregoing agreed statement of facts, both parties introduced oral and
for the estate of the said deceased Arthur Graydon Moody. documentary evidence from which it appears that Arthur G. Moody, an American
citizen, came to the Philippine Islands in 1902 or 1903 and engaged actively in
"IX. That on December 3, 1931, the committee on claims and appraisals filed with the business in these Islands up to the time of his death in Calcutta, India, on February 18,
court its report, certified copy of which marked Exhibit KK is hereto attached and 1931. He had no business elsewhere and at the time of his death left an estate
made a part hereof. consisting principally of bonds and shares of stock of corporations organized under the
laws of the Philippine Islands, bank deposits and other intangibles and personal
"X. That on September 15, 1931, the Bureau of Internal Revenue addressed to the property valued by the commissioners of appraisal and claims at P609,767.58 and by
attorney for the administratrix Ida M. Palmer a letter, copy of which marked Exhibit LL the Collector of Internal Revenue for the purposes of inheritance tax at P653,657.47.
is hereto attached and made a part hereof. All of said property at the time of his death was located and had its situs within the
Philippine Islands. So far as this record shows, he left no property of any kind located
"XI. That on October 15, 1931, the attorney for Ida M. Palmer answered the letter of anywhere else. In his will, Exhibit AA, executed without date in Manila in accordance
the Collector of Internal Revenue referred to in the preceding paragraph. Said answer with the formalities of the Philippine law, in which he bequeathed all his property to his
marked Exhibit MM is hereto attached and made a part hereof. sister, Ida M. Palmer, he stated:jgc:chanrobles.com.ph

"XII. That on November 4, 1931, and in answer to the letter mentioned in the preceding "I, Arthur G. Moody, a citizen of the United States of America, residing in the
paragraph, the Bureau of Internal Revenue addressed to the attorney for Ida M. Palmer Philippine Islands, hereby publish and declare the following as my last Will and
another letter, copy of which marked Exhibit NN is hereto attached and made a part Testament . . . ."cralaw virtua1aw library
hereof.
The substance of the plaintiff’s cause of action is stated in paragraph 7 of his complaint
"XIII. That on December 7, 1931, the attorney for Ida M. Palmer again replied in a as follows:jgc:chanrobles.com.ph
letter, marked Exhibit OO, hereto attached and made a part hereof.
"That there is no valid law or regulation of the Government of the Philippine Islands
"XIV. That the estate of the late Arthur Graydon Moody paid under protest the sum of under or by virtue of which any inheritance tax may be levied, assessed or collected
P50,000 on July 22, 1931, and the other sum of P40,019,75 on January 19, 1932, upon transfer, by death and succession, of intangible personal properties of a person not
making a total of P90,019,75, of which P77,018.39 covers the assessment for domiciled in the Philippine Islands, and the levy and collection by defendant of
inheritance tax and the sum of P13,001.41 covers the assessment for income tax against inheritance tax computed upon the value of said stocks, bonds, credits and other
said estate. intangible properties as aforesaid constituted and constitutes the taking and deprivation
of property without due process of law contrary to the Bill of Rights and organic law of
"XV. That on January 21, 1932, the Collector of Internal Revenue overruled the protest the Philippine Islands."cralaw virtua1aw library
made by Ida M. Palmer through her attorney.
Section 1536 of the Revised Administrative Code (as amended) provides as
93
follows:jgc:chanrobles.com.ph Arthur G. Moody was legally domiciled in the Philippine Islands on the day of his
death. Moody was never married and there is no doubt that he had his legal domicile in
"SEC. 1536. Conditions and rate of taxation. — Every transmission by virtue of the Philippine Islands from 1902 or 1903 forward during which time he accumulated a
inheritance, devise, bequest, gift mortis causa or advance in anticipation of inheritance, fortune from his business in the Philippine Islands. He lived in the Elks’ Club in Manila
devise, or bequest of real property located in the Philippine Islands and real rights in for many years and was living there up to the date he left Manila the latter part of
such property; of any franchise which must be exercised in the Philippine Islands; of February, 1928, under the following circumstances: He was afflicted with leprosy in an
any shares, obligations, or bonds issued by any corporation or sociedad anonima advanced stage and had been informed by Dr. Wade that he would be reported to the
organized or constituted in the Philippine Islands in accordance with its laws; of any Philippine authorities for confinement in the Culion Leper Colony as required by the
shares or rights in any partnership, business or industry established in the Philippine law. Distressed at the thought of being thus segregated and in violation of his promise
Islands or of any personal property located in the Philippine Islands shall be subject to to Dr. Wade that he would voluntarily go to Culion, he surreptitiously left the Islands
the following tax:" the latter part of February, 1928, under cover of night, on a freighter, without ticket,
passport or tax clearance certificate. The record does not show where Moody was
x x x during the remainder of the year 1928. He lived with a friend in Paris, France, during
the months of March and April of the year 1929 where he was receiving treatment for
leprosy at the Pasteur Institute. The record does not show where Moody was in the
It is alleged in the complaint that at the time of his death, Arthur G. Moody was a "non- interval between April, 1929, and November 26, 1930, on which latter date he wrote a
resident of the Philippine Islands." The answer, besides the general denial, sets up as a letter, Exhibit B, to Harry Wendt of Manila, offering to sell him his interest in the
special defense that "Arthur G. Moody, now deceased, was and prior to the date of his Camera Supply Company, a Philippine corporation, in which Moody owned 599 out of
death, a resident in the City of Manila, Philippine Islands, where he was engaged 603 shares. In this letter, among other things, he states: "Certainly I’ll never return there
actively in business." Issue was thus joined on the question: Where was the legal to live or enter business again." In this same letter he says:jgc:chanrobles.com.ph
domicile of Arthur G. Moody at the time of his death?
"I wish to know as soon as possible now (as to the purchase) for I have very recently
The Solicitor-General raises a preliminary objection to the consideration of any decided either to sell or put in a line of school or office supplies . . . before I go to the
evidence that Moody’s domicile was elsewhere than in Manila at the time of his death necessary investments in placing any side lines. I concluded to get your definite reply
based on the proposition that as no such objection was made before the Collector of to this . . . I have given our New York buying agent a conditional order not to be
Internal Revenue as one of the grounds of the protest against the payment of the tax, executed until March and this will give you plenty of time . . . anything that kills a
this objection cannot be considered in a suit against the Collector to recover the taxes business is to have it peddled around as being for sale and this is what I wish to avoid."
paid under protest. He relies upon the decision in the case of W. C. Tucker v. A. C. He wrote letters dated December 12, 1930, and January 3, 1931, along the same line to
Alexander, Collector (15 Fed. [2], 356). We call attention, however, to the fact that this Wendt. As Moody died of leprosy less than two months after these letters were written,
decision was reversed in 275 U. S., 232; 72 Law. ed., 256, and the case remanded for there can be no doubt that he would have been immediately segregated in the Culion
trial on the merits on the ground that the requirement that the action shall be based Leper Colony had he returned to the Philippine Islands. He was, therefore, a fugitive,
upon the same grounds, and only such, as were presented in the protest had been not from justice, but from confinement in the Culion Leper Colony in accordance with
waived by the collector. In the case before us no copy of the taxpayer’s protest is the law of the Philippine Islands.
included in the record and we have no means of knowing its contents. We think,
therefore, the preliminary objection made on behalf of the appellee does not lie. There is no statement of Moody, oral or written, in the record that he had adopted a
new domicile while he was absent from Manila. Though he was physically present for
We proceed, therefore, to the consideration of the question on the merits as to whether some months in Calcutta prior to the date of his death there, the appellant does not
94
claim that Moody had a domicile there although it was precisely from Calcutta that he contends that this assessment involves triple taxation: First, because the corporation
wrote and cabled that he wished to sell his business in Manila and that he had no paid income tax on the same amount during the years it was accumulated as surplus;
intention to live there again. Much less plausible, it seems to us, is the claim that he second, that an inheritance tax on the same amount was assessed against the estate, and
established a legal domicile in Paris in February, 1929. The record contains no writing third, the same amount is assessed as income of the estate. As to the first, it appears
whatever of Moody from Paris. There is no evidence as to where in Paris he had any from the collector’s assessment, Exhibit II, that the collector allowed the estate a
fixed abode that he intended to be his permanent home. There is no evidence that he deduction of the normal income tax on said amount because it had already been paid at
acquired any property in Paris or engaged in any settled business on his own account the source by the Camera Supply Company. The only income tax assessed against the
there. There is no evidence of any affirmative factors that prove the establishment of a estate was the additional tax or surtax that had not been paid by the Camera Supply
legal domicile there. The negative evidence that he told Cooley that he did not intend to Company for which the estate, having actually received the income, is clearly liable. As
return to Manila does not prove that he had established a domicile in Paris. His short to the second alleged double taxation, it is clear that the inheritance tax and the
stay of three months in Paris is entirely consistent with the view that he was a transient additional income tax in question are entirely distinct. They are assessed under different
in Paris for the purpose of receiving treatments at the Pasteur Institute. The evidence in statutes and we are not convinced by the appellant’s argument that the estate which
the record indicates clearly that Moody’s continued absence from his legal domicile in received these dividends should not be held liable for the payment of the income tax
the Philippines was due to and reasonably accounted for by the same motive that thereon because the operation was simply the conversion of the surplus of the
caused his surreptitious departure, namely, to evade confinement in the Culion Leper corporation into the property of the individual stockholders. (Cf. U. S. v. Phellis, 257
Colony; for he doubtless knew that on his return he would be immediately confined, U. S., 171, and Taft v. Bowers, 278 U. S., 460.) Section 4 of Act No. 2833 as amended,
because his affliction became graver while he was absent than it was on the day of his which is relied on by the appellant, plainly provides that the income from exempt
precipitous departure and he could not conceal himself in the Philippines where he was property shall be included as income subject to tax.
well known, as he might do in foreign parts.
Finding no merit in any of the assignments of error of the appellant, we affirm the
Our Civil Code (art. 40) defines the domicile of natural persons as "the place of their judgment of the trial court, first, because the property in the estate of Arthur G. Moody
usual residence." The record before us leaves no doubt in our minds that the "usual at the time of his death was located and had its situs within the Philippine Islands and,
residence" of this unfortunate man, whom appellant describes as a "fugitive" and second, because his legal domicile up to the time of his death was within the Philippine
"outcast", was in Manila where he had lived and toiled for more than a quarter of a Islands. Costs against the Appellant.
century, rather than in any foreign country he visited during his wanderings up to the
date of his death in Calcutta. To effect the abandonment of one’s domicile, there must Malcolm, Villa-Real, and Imperial, JJ., concur.
be a deliberate and provable choice of a new domicile, coupled with actual residence in
the place chosen, with a declared or provable intent that it should be one’s fixed and
permanent place of abode, one’s home. There is a complete dearth of evidence in the
record that Moody ever established a new domicile in a foreign country.

The contention under the appellant’s third assignment of error that the defendant
collector illegally assessed an income tax of P13,001.41 against the Moody estate is, in
our opinion, untenable. The grounds for this assessment, stated by the Collector of
Internal Revenue in his letter, Exhibit NN, appear to us to be sound. That the amount of
P259,986.69 was received by the estate of Moody as dividends declared out of surplus
by the Camera Supply Company is clearly established by the evidence. The appellant
95
UJANO VS. REPUBLIC citizen of the United States of
Facts: America, was admitted into this country as a temporary visitor, a status he has
Petitioner seeks to reacquire his Philippine citizenship in a petition filed before the Court maintained at the time of the filing of
of First Instance of Ilocos the present petition for reacquisition of Philippine citizenship and which continues up to
Sur. Petitioner was born 66 years ago of Filipino parents in Magsingal Ilocos Sur. He is the present. Such being the
married to Maxima O. Ujano with case, he has not complied with the specific requirement of law regarding six months residence
whom he has one son, Prospero, who is now of legal age. He left the Philippines for the before filing his present
United States of America in
1927 where after a residence of more than 20 years he acquired American citizenship by petition."
naturalization. He returned
to the Philippines on November 10, 1960 to which he was admitted merely for a temporary
stay. He owns an agricultural
land and a residential house situated in Magsingal, Ilocos Sur. He receives a monthly
pension from the Social Security
Administration of the United States of America. He has no record of conviction and it is his
intention to renounce his
allegiance to the U.S.A. After hearing, the court a quo rendered decision denying the petition
on the ground that petitioner
did not have the residence required by law six months before he filed his petition for
reacquisition of Philippine
citizenship.
Ruling:
The court a quo, in denying the petition, made the following comment: "One of the
qualifications for reacquiring
Philippine citizenship is that the applicant 'shall have resided in the Philippines at least
six months before he applies
for naturalization' [Section 3(1), Commonwealth Act No. 63]. This 'residence' requirement
in cases of naturalization,
has already been interpreted to mean the actual or constructive permanent home
otherwise known as legal residence
or domicile (Wilfredo Uytengsu vs. Republic of the Philippines, 95 Phil. 890). A place in a
country or state where he lives
and stays permanently, and to which he intends to return after a temporary absence, no matter
how long, is his domicile. In
other words domicile is characterized by animus manendi. So an alien who has been
admitted into this country as a
temporary visitor, either for business or pleasure, or for reasons of health, though actually
present in this country cannot be
said to have established his domicile here because the period of his stay is only temporary in
nature and must leave when the
purpose of his coming is accomplished. In the present case, petitioner, who is presently a
96
vs. Republic of the Philippines, 95 Phil. 890). A place in a country or state where he
G.R. No. L-22041 May 19, 1966 lives and stays permanently, and to which he intends to return after a temporary
absence, no matter how long, is his domicile. In other words domicile is characterized
MELECIO CLARINIO UJANO, petitioner and appellant, by animus manendi. So an alien who has been admitted into this country as a temporary
vs. visitor, either for business or pleasure, or for reasons of health, though actually present
REPUBLIC OF THE PHILIPPINES, oppositor and appellee. in this country cannot be said to have established his domicile here because the period
of his stay is only temporary in nature and must leave when the purpose of his coming
Tagayuna, Arce and Tabaino for petitioner and appellant. is accomplished. In the present case, petitioner, who is presently a citizen of the United
Office of the Solicitor General Arturo A. Alafriz, Assistant Solicitor F. C. Zaballero and States of America, was admitted into this country as a temporary visitor, a status he has
Solicitor Camilo D. Quiason for oppositor and appellee. maintained at the time of the filing of the present petition for reacquisition of Philippine
citizenship and which continues up to the present. Such being the case, he has not
BAUTISTA ANGELO, J.: complied with the specific requirement of law regarding six months residence before
filing his present petition."
Petitioner seeks to reacquire his Philippine citizenship in a petition filed before the
Court of First Instance of Ilocos Sur. We can hardly add to the foregoing comment of the court a quo. We find it to be a
correct interpretation [Section 3 (1) of Commonwealth Act No. 63] which requires that
Petitioner was born 66 years ago of Filipino parents in Magsingal Ilocos Sur. He is before a person may reacquire his Philippine citizenship he "shall have resided in the
married to Maxima O. Ujano with whom he has one son, Prospero, who is now of legal Philippines at least six months before he applies for naturalization." The word
age. He left the Philippines for the United States of America in 1927 where after a "residence" used therein imports not only an intention to reside in a fixed place but also
residence of more than 20 years he acquired American citizenship by naturalization. He personal presence coupled with conduct indicative of such intention (Yen vs. Republic,
returned to the Philippines on November 10, 1960 to which he was admitted merely for L-18885, January 31,1964; Nuval vs. Guray, 52 Phil. 645). Indeed, that term cannot
a temporary stay. He owns an agricultural land and a residential house situated in refer to the presence in this country of a person who has been admitted only on the
Magsingal, Ilocos Sur worth not less than P5,000.00. He receives a monthly pension of strength of a permit for temporary residence. In other words, the term residence used in
$115.00 from the Social Security Administration of the United States of America. He said Act should have the same connotation as that used in Commonwealth Act No. 473,
has no record of conviction and it is his intention to renounce his allegiance to the the Revised Naturalization Law, even if in approving the law permitting the
U.S.A.1äwphï1.ñët reacquisition of Philippine citizenship our Congress has liberalized its requirement by
foregoing the qualifications and special disqualifications prescribed therein. The only
After hearing, the court a quo rendered decision denying the petition on the ground that way by which petitioner can reacquire his lost Philippine citizenship is by securing a
petitioner did not have the residence required by law six months before he filed his quota for permanent residence so that he may come within the purview of the residence
petition for reacquisition of Philippine citizenship. Hence the present appeal. requirement of Commonwealth Act No. 63.

The court a quo, in denying the petition, made the following comment: "One of the
qualifications for reacquiring Philippine citizenship is that the applicant 'shall have
resided in the Philippines at least six months before he applies for naturalization'
[Section 3(1), Commonwealth Act No. 63]. This 'residence' requirement in cases of
naturalization, has already been interpreted to mean the actual or constructive
permanent home otherwise known as legal residence or domicile (Wilfredo Uytengsu
97
CAASI VS. COURT OF APPEALS elections on January 18, 1988.
Facts: Ruling: Despite his vigorous disclaimer, Miguel's immigration to the United States in
• These two cases were consolidated because they have the same objective; the 1984 constituted an abandonment of his
disqualification under Section 68 of the domicile and residence in the Philippines. For he did not go to the United States merely
Omnibus Election Code of the private respondent, Merito Miguel for the position of to visit his children or his doctor there; he
municipal mayor of Bolinao, entered the limited States with the intention to have there permanently as evidenced by
Pangasinan, to which he was elected in the local elections of January 18, 1988, on the his application for an immigrant's (not a
ground that he is a green card visitor's or tourist's) visa. Based on that application of his, he was issued by the U.S.
holder, hence, a permanent resident of the United States of America, not of Bolinao. Government the requisite green card or
• In his answer to both petitions, Miguel admitted that he holds a green card issued to authority to reside there permanently.
him by the US Immigration Service, To be "qualified to run for elective office" in the Philippines, the law requires that the
but he denied that he is a permanent resident of the United States. He allegedly candidate who is a green card holder must
obtained the green card for convenience have "waived his status as a permanent resident or immigrant of a foreign country."
in order that he may freely enter the United States for his periodic medical examination Therefore, his act of filing a certificate of
and to visit his children there. candidacy for elective office in the Philippines, did not of itself constitute a waiver of
He alleged that he is a permanent resident of Bolinao, Pangasinan, that he voted in all his status as a permanent resident or
previous elections, including the immigrant of the United States. The waiver of his green card should be manifested by
plebiscite on February 2,1987 for the ratification of the 1987 Constitution, and the some act or acts independent of and done
congressional elections on May prior to filing his candidacy for elective office in this country. Without such prior
18,1987. waiver, he was "disqualified to run for any
• After hearing the consolidated petitions before it, the COMELEC dismissed the
petitions on the ground that: The elective office" (Sec. 68, Omnibus Election Code).
possession of a green card by the respondent (Miguel) does not sufficiently establish
that he has abandoned his
residence in the Philippines. On the contrary, inspite (sic) of his green card, Respondent
has sufficiently indicated his
intention to continuously reside in Bolinao as shown by his having voted in successive
elections in said municipality.
As the respondent meets the basic requirements of citizenship and residence for
candidates to elective local officials
(sic) as provided for in Section 42 of the Local Government Code, there is no legal
obstacle to his candidacy for mayor
of Bolinao, Pangasinan. (p. 12, Rollo, G.R. No. 84508).
Issues: (1) whether or not a green card is proof that the holder is a permanent resident
of the United States, and
(2) whether respondent Miguel had waived his status as a permanent resident of or
immigrant to the U.S.A. prior to the local
98
position of municipal mayor of Bolinao, Pangasinan, also to disqualify Merito Miguel
G.R. No. 88831 November 8, 1990 on account of his being a green card holder.

MATEO CAASI, petitioner, In his answer to both petitions, Miguel admitted that he holds a green card issued to
vs. him by the US Immigration Service, but he denied that he is a permanent resident of
THE HON. COURT OF APPEALS and MERITO C. MIGUEL, respondents. the United States. He allegedly obtained the green card for convenience in order that he
may freely enter the United States for his periodic medical examination and to visit his
G.R. No. 84508 November 13, 1990 children there. He alleged that he is a permanent resident of Bolinao, Pangasinan, that
he voted in all previous elections, including the plebiscite on February 2,1987 for the
ANECITO CASCANTE petitioner, ratification of the 1987 Constitution, and the congressional elections on May 18,1987.
vs.
THE COMMISSION ON ELECTIONS and MERITO C. MIGUEL, respondents. After hearing the consolidated petitions before it, the COMELEC with the exception of
Commissioner Anacleto Badoy, Jr., dismissed the petitions on the ground that:
Ireneo B. Orlino for petitioner in G.R. Nos. 88831 & 84508.
The possession of a green card by the respondent (Miguel) does not
Montemayor & Montemayor Law Office for private respondent. sufficiently establish that he has abandoned his residence in the
Philippines. On the contrary, inspite (sic) of his green card, Respondent
has sufficiently indicated his intention to continuously reside in Bolinao
as shown by his having voted in successive elections in said
GRIÑO-AQUINO, J.: municipality. As the respondent meets the basic requirements of
citizenship and residence for candidates to elective local officials (sic) as
These two cases were consolidated because they have the same objective; the provided for in Section 42 of the Local Government Code, there is no
disqualification under Section 68 of the Omnibus Election Code of the private legal obstacle to his candidacy for mayor of Bolinao, Pangasinan. (p. 12,
respondent, Merito Miguel for the position of municipal mayor of Bolinao, Pangasinan, Rollo, G.R. No. 84508).
to which he was elected in the local elections of January 18, 1988, on the ground that
he is a green card holder, hence, a permanent resident of the United States of America, In his dissenting opinion, Commissioner Badoy, Jr. opined that:
not of Bolinao.
A green card holder being a permanent resident of or an immigrant of a
G.R. No. 84508 is a petition for review on certiorari of the decision dated January 13, foreign country and respondent having admitted that he is a green card
1988 of the COMELEC First Division, dismissing the three (3) petitions of Anecito holder, it is incumbent upon him, under Section 68 of the Omnibus
Cascante (SPC No. 87-551), Cederico Catabay (SPC No. 87-595) and Josefino C. Election Code, to prove that he "has waived his status as a permanent
Celeste (SPC No. 87-604), for the disqualification of Merito C. Miguel filed prior to resident or immigrant" to be qualified to run for elected office. This
the local elections on January 18, 1988. respondent has not done. (p. 13, Rollo, G.R. No. 84508.)

G.R. No. 88831, Mateo Caasi vs. Court of Appeals, et al., is a petition for review of the In G.R. No. 88831, "Mateo Caasi, petitioner vs. Court of Appeals and Merito Miguel,
decision dated June 21, 1989, of the Court of Appeals in CA-G.R. SP No. 14531 respondents," the petitioner prays for a review of the decision dated June 21, 1989 of
dismissing the petition for quo warranto filed by Mateo Caasi, a rival candidate for the the Court of Appeals in CA-G.R. SP No. 14531 "Merito C. Miguel, petitioner vs. Hon.
99
Artemio R. Corpus, etc., respondents," reversing the decision of the Regional Trial In view of current rumor that a good number of elective and appointive public officials
Court which denied Miguel's motion to dismiss the petition for quo warranto filed by in the present administration of President Corazon C. Aquino are holders of green cards
Caasi. The Court of Appeals ordered the regional trial court to dismiss and desist from in foreign countries, their effect on the holders' right to hold elective public office in the
further proceeding in the quo warranto case. The Court of Appeals held: Philippines is a question that excites much interest in the outcome of this case.

... it is pointless for the Regional Trial Court to hear the case questioning In the case of Merito Miguel, the Court deems it significant that in the "Application
the qualification of the petitioner as resident of the Philippines, after the for Immigrant Visa and Alien Registration" (Optional Form No. 230, Department of
COMELEC has ruled that the petitioner meets the very basic State) which Miguel filled up in his own handwriting and submitted to the US Embassy
requirements of citizenship and residence for candidates to elective local in Manila before his departure for the United States in 1984, Miguel's answer to
officials (sic) and that there is no legal obstacles (sic) for the candidacy Question No. 21 therein regarding his "Length of intended stay (if permanently, so
of the petitioner, considering that decisions of the Regional Trial Courts state)," Miguel's answer was, "Permanently."
on quo warranto cases under the Election Code are appealable to the
COMELEC. (p. 22, Rollo, G.R. No. 88831.) On its face, the green card that was subsequently issued by the United States
Department of Justice and Immigration and Registration Service to the respondent
These two cases pose the twin issues of: (1) whether or not a green card is proof that Merito C. Miguel identifies him in clear bold letters as a RESIDENT ALIEN. On the
the holder is a permanent resident of the United States, and (2) whether respondent back of the card, the upper portion, the following information is printed:
Miguel had waived his status as a permanent resident of or immigrant to the U.S.A.
prior to the local elections on January 18, 1988. Alien Registration Receipt Card.

Section 18, Article XI of the 1987 Constitution provides: Person identified by this card is entitled to reside
permanently and work in the United States." (Annex A
Sec. 18. Public officers and employees owe the State and this pp. 189-190, Rollo of G.R. No. 84508.)
Constitution allegiance at all times, and any public officer or employee
who seeks to change his citizenship or acquire the status of an Despite his vigorous disclaimer, Miguel's immigration to the United States in 1984
immigrant of another country during his tenure shall be dealt with by constituted an abandonment of his domicile and residence in the Philippines. For he did
law. not go to the United States merely to visit his children or his doctor there; he entered
the limited States with the intention to have there permanently as evidenced by his
In the same vein, but not quite, Section 68 of the Omnibus Election Code of the application for an immigrant's (not a visitor's or tourist's) visa. Based on that
Philippines (B.P. Blg. 881) provides: application of his, he was issued by the U.S. Government the requisite green card or
authority to reside there permanently.
SEC. 68. Disqualifications ... Any person who is a permanent resident of
or an immigrant to a foreign country shall not be qualified to run for any Immigration is the removing into one place from another; the act of
elective office under this Code, unless said person has waived his status immigrating the entering into a country with the intention of residing in
as permanent resident or immigrant of a foreign country in accordance it.
with the residence requirement provided for in the election laws. (Sec.
25, 1971, EC). An immigrant is a person who removes into a country for the purpose
of permanent residence. As shown infra 84, however, statutes sometimes
100
give a broader meaning to the term "immigrant." (3 CJS 674.) Code, unless such person has waived his status as permanent resident or
immigrant of a foreign country in accordance with the residence
As a resident alien in the U.S., Miguel owes temporary and local allegiance to the U.S., requirement provided for in the election laws.'
the country in which he resides (3 CJS 527). This is in return for the protection given to
him during the period of his residence therein. Did Miguel, by returning to the Philippines in November 1987 and presenting himself
as a candidate for mayor of Bolinao in the January 18,1988 local elections, waive his
Aliens reading in the limited States, while they are permitted to remain, status as a permanent resident or immigrant of the United States?
are in general entitled to the protection of the laws with regard to their
rights of person and property and to their civil and criminal To be "qualified to run for elective office" in the Philippines, the law requires that the
responsibility. candidate who is a green card holder must have "waived his status as a permanent
resident or immigrant of a foreign country." Therefore, his act of filing a certificate of
In general, aliens residing in the United States, while they are permitted candidacy for elective office in the Philippines, did not of itself constitute a waiver of
to remain are entitled to the safeguards of the constitution with regard to his status as a permanent resident or immigrant of the United States. The waiver of his
their rights of person and property and to their civil and criminal green card should be manifested by some act or acts independent of and done prior to
responsibility. Thus resident alien friends are entitled to the benefit of filing his candidacy for elective office in this country. Without such prior waiver, he
the provision of the Fourteenth Amendment to the federal constitution was "disqualified to run for any elective office" (Sec. 68, Omnibus Election Code).
that no state shall deprive "any person" of life liberty, or property
without due process of law, or deny to any person the equal protection of Respondent Merito Miguel admits that he holds a green card, which proves that he is a
the law, and the protection of this amendment extends to the right to permanent resident or immigrant it of the United States, but the records of this case are
earn a livelihood by following the ordinary occupations of life. So an starkly bare of proof that he had waived his status as such before he ran for election as
alien is entitled to the protection of the provision of the Fifth municipal mayor of Bolinao on January 18, 1988. We, therefore, hold that he was
Amendment to the federal constitution that no person shall be deprived disqualified to become a candidate for that office.
of life, liberty, or property without due process of law. (3 CJS 529-530.)
The reason for Section 68 of the Omnibus Election Code is not hard to find. Residence
Section 18, Article XI of the 1987 Constitution which provides that "any public officer in the municipality where he intends to run for elective office for at least one (1) year at
or employee who seeks to change his citizenship or acquire the status of an immigrant the time of filing his certificate of candidacy, is one of the qualifications that a
of another country during his tenure shall be dealt with by law" is not applicable to candidate for elective public office must possess (Sec. 42, Chap. 1, Title 2, Local
Merito Miguel for he acquired the status of an immigrant of the United States before he Government Code). Miguel did not possess that qualification because he was a
was elected to public office, not "during his tenure" as mayor of Bolinao, Pangasinan. permanent resident of the United States and he resided in Bolinao for a period of only
three (3) months (not one year) after his return to the Philippines in November 1987
The law applicable to him is Section 68 of the Omnibus Election Code (B.P. Blg. 881), and before he ran for mayor of that municipality on January 18, 1988.
which provides:
In banning from elective public office Philippine citizens who are permanent residents
xxx xxx xxx or immigrants of a foreign country, the Omnibus Election Code has laid down a clear
policy of excluding from the right to hold elective public office those Philippine
Any person who is a permanent resident of or an immigrant to a foreign citizens who possess dual loyalties and allegiance. The law has reserved that privilege
country shall not be qualified to run for any elective office under this for its citizens who have cast their lot with our country "without mental reservations or
101
purpose of evasion." The assumption is that those who are resident aliens of a foreign
country are incapable of such entire devotion to the interest and welfare of their
homeland for with one eye on their public duties here, they must keep another eye on
their duties under the laws of the foreign country of their choice in order to preserve
their status as permanent residents thereof.

Miguel insists that even though he applied for immigration and permanent residence in
the United States, he never really intended to live there permanently, for all that he
wanted was a green card to enable him to come and go to the U.S. with ease. In other
words, he would have this Court believe that he applied for immigration to the U.S.
under false pretenses; that all this time he only had one foot in the United States but
kept his other foot in the Philippines. Even if that were true, this Court will not allow
itself to be a party to his duplicity by permitting him to benefit from it, and giving him
the best of both worlds so to speak.

Miguel's application for immigrant status and permanent residence in the U.S. and his
possession of a green card attesting to such status are conclusive proof that he is a
permanent resident of the U.S. despite his occasional visits to the Philippines. The
waiver of such immigrant status should be as indubitable as his application for it.
Absent clear evidence that he made an irrevocable waiver of that status or that he
surrendered his green card to the appropriate U.S. authorities before he ran for mayor
of Bolinao in the local elections on January 18, 1988, our conclusion is that he was
disqualified to run for said public office, hence, his election thereto was null and void.

WHEREFORE, the appealed orders of the COMELEC and the Court of Appeals in
SPC Nos. 87-551, 87-595 and 87-604, and CA-G.R. SP No. 14531 respectively, are
hereby set aside. The election of respondent Merito C. Miguel as municipal mayor of
Bolinao, Pangasinan is hereby annulled. Costs against the said respondent.

SO ORDERED.

102
declaring (petitioner) disqualified and canceling the certificate of candidacy." 7
G.R. No. 119976 September 18, 1995
On March 29, 1995, petitioner filed an Amended/Corrected Certificate of Candidacy,
IMELDA ROMUALDEZ-MARCOS, petitioner, changing the entry "seven" months to "since childhood" in item no. 8 of the amended
vs. certificate. 8 On the same day, the Provincial Election Supervisor of Leyte informed
COMMISSION ON ELECTIONS and CIRILO ROY MONTEJO, respondents. petitioner that:

[T]his office cannot receive or accept the aforementioned Certificate of


Candidacy on the ground that it is filed out of time, the deadline for the
KAPUNAN, J.: filing of the same having already lapsed on March 20, 1995. The
Corrected/Amended Certificate of Candidacy should have been filed on
A constitutional provision should be construed as to give it effective operation and or before the March 20, 1995 deadline. 9
suppress the mischief at which it is aimed. 1 The 1987 Constitution mandates that an
aspirant for election to the House of Representatives be "a registered voter in the Consequently, petitioner filed the Amended/Corrected Certificate of Candidacy with
district in which he shall be elected, and a resident thereof for a period of not less than the COMELEC's Head Office in Intramuros, Manila on
one year immediately preceding the election." 2 The mischief which this provision — March 31, 1995. Her Answer to private respondent's petition in SPA No. 95-009 was
reproduced verbatim from the 1973 Constitution — seeks to prevent is the possibility likewise filed with the head office on the same day. In said Answer, petitioner averred
of a "stranger or newcomer unacquainted with the conditions and needs of a that the entry of the word "seven" in her original Certificate of Candidacy was the
community and not identified with the latter, from an elective office to serve that result of an "honest misinterpretation" 10 which she sought to rectify by adding the
community." 3 words "since childhood" in her Amended/Corrected Certificate of Candidacy and that
"she has always maintained Tacloban City as her domicile or residence. 11 Impugning
Petitioner Imelda Romualdez-Marcos filed her Certificate of Candidacy for the position respondent's motive in filing the petition seeking her disqualification, she noted that:
of Representative of the First District of Leyte with the Provincial Election Supervisor
on March 8, 1995, providing the following information in item no. 8: 4 When respondent (petitioner herein) announced that she was intending
to register as a voter in Tacloban City and run for Congress in the First
RESIDENCE IN THE CONSTITUENCY WHERE I SEEK TO BE District of Leyte, petitioner immediately opposed her intended
ELECTED IMMEDIATELY PRECEDING THE ELECTION: registration by writing a letter stating that "she is not a resident of said
__________ Years and seven Months. city but of Barangay Olot, Tolosa, Leyte. After respondent had
registered as a voter in Tolosa following completion of her six month
On March 23, 1995, private respondent Cirilo Roy Montejo, the incumbent actual residence therein, petitioner filed a petition with the COMELEC
Representative of the First District of Leyte and a candidate for the same position, filed to transfer the town of Tolosa from the First District to the Second
a "Petition for Cancellation and Disqualification" 5 with the Commission on Elections District and pursued such a move up to the Supreme Court, his purpose
alleging that petitioner did not meet the constitutional requirement for residency. In his being to remove respondent as petitioner's opponent in the congressional
petition, private respondent contended that Mrs. Marcos lacked the Constitution's one election in the First District. He also filed a bill, along with other Leyte
year residency requirement for candidates for the House of Representatives on the Congressmen, seeking the creation of another legislative district to
evidence of declarations made by her in Voter Registration Record 94-No. remove the town of Tolosa out of the First District, to achieve his
3349772 6and in her Certificate of Candidacy. He prayed that "an order be issued purpose. However, such bill did not pass the Senate. Having failed on
103
such moves, petitioner now filed the instant petition for the same not Tacloban. She never disputed this claim and instead implicitly
objective, as it is obvious that he is afraid to submit along with acceded to it by registering in Tolosa.
respondent for the judgment and verdict of the electorate of the First
District of Leyte in an honest, orderly, peaceful, free and clean elections This incident belies respondent's claim of "honest misinterpretation or
on May 8, 1995. 12 honest mistake." Besides, the Certificate of Candidacy only asks for
RESIDENCE. Since on the basis of her Answer, she was quite aware of
On April 24, 1995, the Second Division of the Commission on Elections (COMELEC), "residence of origin" which she interprets to be Tacloban City, it is
by a vote of 2 to 1, 13 came up with a Resolution 1) finding private respondent's Petition curious why she did not cite Tacloban City in her Certificate of
for Disqualification in SPA 95-009 meritorious; 2) striking off petitioner's Candidacy. Her explanation that she thought what was asked was her
Corrected/Amended Certificate of Candidacy of March 31, 1995; and 3) canceling her actual and physical presence in Tolosa is not easy to believe because
original Certificate of Candidacy. 14 Dealing with two primary issues, namely, the there is none in the question that insinuates about Tolosa. In fact, item
validity of amending the original Certificate of Candidacy after the lapse of the no. 8 in the Certificate of Candidacy speaks clearly of "Residency in the
deadline for filing certificates of candidacy, and petitioner's compliance with the one CONSTITUENCY where I seek to be elected immediately preceding
year residency requirement, the Second Division held: the election." Thus, the explanation of respondent fails to be persuasive.

Respondent raised the affirmative defense in her Answer that the printed From the foregoing, respondent's defense of an honest mistake or
word "Seven" (months) was a result of an "honest misinterpretation or misinterpretation, therefore, is devoid of merit.
honest mistake" on her part and, therefore, an amendment should
subsequently be allowed. She averred that she thought that what was To further buttress respondent's contention that an amendment may be
asked was her "actual and physical" presence in Tolosa and not made, she cited the case of Alialy v. COMELEC (2 SCRA 957). The
residence of origin or domicile in the First Legislative District, to which reliance of respondent on the case of Alialy is misplaced. The case only
she could have responded "since childhood." In an accompanying applies to the "inconsequential deviations which cannot affect the result
affidavit, she stated that her domicile is Tacloban City, a component of of the election, or deviations from provisions intended primarily to
the First District, to which she always intended to return whenever secure timely and orderly conduct of elections." The Supreme Court in
absent and which she has never abandoned. Furthermore, in her that case considered the amendment only as a matter of form. But in the
memorandum, she tried to discredit petitioner's theory of instant case, the amendment cannot be considered as a matter of form or
disqualification by alleging that she has been a resident of the First an inconsequential deviation. The change in the number of years of
Legislative District of Leyte since childhood, although she only became residence in the place where respondent seeks to be elected is a
a resident of the Municipality of Tolosa for seven months. She asserts substantial matter which determines her qualification as a candidacy,
that she has always been a resident of Tacloban City, a component of the specially those intended to suppress, accurate material representation in
First District, before coming to the Municipality of Tolosa. the original certificate which adversely affects the filer. To admit the
amended certificate is to condone the evils brought by the shifting minds
Along this point, it is interesting to note that prior to her registration in of manipulating candidate, of the detriment of the integrity of the
Tolosa, respondent announced that she would be registering in Tacloban election.
City so that she can be a candidate for the District. However, this
intention was rebuffed when petitioner wrote the Election Officer of Moreover, to allow respondent to change the seven (7) month period of
Tacloban not to allow respondent since she is a resident of Tolosa and her residency in order to prolong it by claiming it was "since childhood"
104
is to allow an untruthfulness to be committed before this Commission. Thus, her animus revertendi is pointed to Metro Manila and not
The arithmetical accuracy of the 7 months residency the respondent Tacloban.
indicated in her certificate of candidacy can be gleaned from her entry in
her Voter's Registration Record accomplished on January 28, 1995 This Division is aware that her claim that she has been a resident of the
which reflects that she is a resident of Brgy. Olot, Tolosa, Leyte for 6 First District since childhood is nothing more than to give her a color of
months at the time of the said registration (Annex A, Petition). Said qualification where she is otherwise constitutionally disqualified. It
accuracy is further buttressed by her letter to the election officer of San cannot hold ground in the face of the facts admitted by the respondent in
Juan, Metro Manila, dated August 24, 1994, requesting for the her affidavit. Except for the time that she studied and worked for some
cancellation of her registration in the Permanent List of Voters thereat so years after graduation in Tacloban City, she continuously lived in
that she can be re-registered or transferred to Brgy. Olot, Tolosa, Leyte. Manila. In 1959, after her husband was elected Senator, she lived and
The dates of these three (3) different documents show the respondent's resided in San Juan, Metro Manila where she was a registered voter. In
consistent conviction that she has transferred her residence to Olot, 1965, she lived in San Miguel, Manila where she was again a registered
Tolosa, Leyte from Metro Manila only for such limited period of time, voter. In 1978, she served as member of the Batasang Pambansa as the
starting in the last week of August 1994 which on March 8, 1995 will representative of the City of Manila and later on served as the Governor
only sum up to 7 months. The Commission, therefore, cannot be of Metro Manila. She could not have served these positions if she had
persuaded to believe in the respondent's contention that it was an error. not been a resident of the City of Manila. Furthermore, when she filed
her certificate of candidacy for the office of the President in 1992, she
xxx xxx xxx claimed to be a resident of San Juan, Metro Manila. As a matter of fact
on August 24, 1994, respondent wrote a letter with the election officer of
Based on these reasons the Amended/Corrected Certificate of Candidacy San Juan, Metro Manila requesting for the cancellation of her
cannot be admitted by this Commission. registration in the permanent list of voters that she may be re-registered
or transferred to Barangay Olot, Tolosa, Leyte. These facts manifest that
xxx xxx xxx she could not have been a resident of Tacloban City since childhood up
to the time she filed her certificate of candidacy because she became a
Anent the second issue, and based on the foregoing discussion, it is clear resident of many places, including Metro Manila. This debunks her
that respondent has not complied with the one year residency claim that prior to her residence in Tolosa, Leyte, she was a resident of
requirement of the Constitution. the First Legislative District of Leyte since childhood.

In election cases, the term "residence" has always been considered as In this case, respondent's conduct reveals her lack of intention to make
synonymous with "domicile" which imports not only the intention to Tacloban her domicile. She registered as a voter in different places and
reside in a fixed place but also personal presence in-that place, coupled on several occasions declared that she was a resident of Manila.
with conduct indicative of such intention. Domicile denotes a fixed Although she spent her school days in Tacloban, she is considered to
permanent residence to which when absent for business or pleasure, or have abandoned such place when she chose to stay and reside in other
for like reasons, one intends to return. (Perfecto Faypon vs. Eliseo different places. In the case of Romualdez vs. RTC (226 SCRA 408) the
Quirino, 96 Phil 294; Romualdez vs. RTC-Tacloban, 226 SCRA 408). In Court explained how one acquires a new domicile by choice. There must
respondent's case, when she returned to the Philippines in 1991, the concur: (1) residence or bodily presence in the new locality; (2)
residence she chose was not Tacloban but San Juan, Metro Manila. intention to remain there; and (3) intention to abandon the old domicile.
105
In other words there must basically be animus manendi with animus non RESOLVED to DENY it, no new substantial matters having been raised
revertendi. When respondent chose to stay in Ilocos and later on in therein to warrant re-examination of the resolution granting the petition
Manila, coupled with her intention to stay there by registering as a voter for disqualification. 18
there and expressly declaring that she is a resident of that place, she is
deemed to have abandoned Tacloban City, where she spent her On May 11, 1995, the COMELEC issued a Resolution allowing petitioner's
childhood and school days, as her place of domicile. proclamation should the results of the canvass show that she obtained the highest
number of votes in the congressional elections in the First District of Leyte. On the
Pure intention to reside in that place is not sufficient, there must same day, however, the COMELEC reversed itself and issued a second Resolution
likewise be conduct indicative of such intention. Respondent's directing that the proclamation of petitioner be suspended in the event that she obtains
statements to the effect that she has always intended to return to the highest number of votes. 19
Tacloban, without the accompanying conduct to prove that intention, is
not conclusive of her choice of residence. Respondent has not presented In a Supplemental Petition dated 25 May 1995, petitioner averred that she was the
any evidence to show that her conduct, one year prior the election, overwhelming winner of the elections for the congressional seat in the First District of
showed intention to reside in Tacloban. Worse, what was evident was Leyte held May 8, 1995 based on the canvass completed by the Provincial Board of
that prior to her residence in Tolosa, she had been a resident of Manila. Canvassers on May 14, 1995. Petitioner alleged that the canvass showed that she
obtained a total of 70,471 votes compared to the 36,833 votes received by Respondent
It is evident from these circumstances that she was not a resident of the Montejo. A copy of said Certificate of Canvass was annexed to the Supplemental
First District of Leyte "since childhood." Petition.

To further support the assertion that she could have not been a resident On account of the Resolutions disqualifying petitioner from running for the
of the First District of Leyte for more than one year, petitioner correctly congressional seat of the First District of Leyte and the public respondent's Resolution
pointed out that on January 28, 1995 respondent registered as a voter at suspending her proclamation, petitioner comes to this court for relief.
precinct No. 18-A of Olot, Tolosa, Leyte. In doing so, she placed in her
Voter Registration Record that she resided in the municipality of Tolosa Petitioner raises several issues in her Original and Supplemental Petitions. The
for a period of six months. This may be inconsequential as argued by the principal issues may be classified into two general areas:
respondent since it refers only to her residence in Tolosa, Leyte. But her
failure to prove that she was a resident of the First District of Leyte prior I. The issue of Petitioner's qualifications
to her residence in Tolosa leaves nothing but a convincing proof that she
had been a resident of the district for six months only. 15 Whether or not petitioner was a resident, for election purposes, of the
First District of Leyte for a period of one year at the time of the May 9,
In a Resolution promulgated a day before the May 8, 1995 elections, the COMELEC en 1995 elections.
banc denied petitioner's Motion for Reconsideration 16 of the April 24, 1995 Resolution
declaring her not qualified to run for the position of Member of the House of II. The Jurisdictional Issue
Representatives for the First Legislative District of Leyte. 17 The Resolution tersely
stated: a) Prior to the elections

After deliberating on the Motion for Reconsideration, the Commission Whether or not the COMELEC properly exercised its jurisdiction in
106
disqualifying petitioner outside the period mandated by the Omnibus as soon as his purpose is established it is residence. 22 It is thus, quite perfectly normal
Election Code for disqualification cases under Article 78 of the said for an individual to have different residences in various places. However, a person can
Code. only have a single domicile, unless, for various reasons, he successfully abandons his
domicile in favor of another domicile of choice. In Uytengsu vs. Republic, 23 we laid
b) After the Elections this distinction quite clearly:

Whether or not the House of Representatives Electoral Tribunal There is a difference between domicile and residence. "Residence" is
assumed exclusive jurisdiction over the question of petitioner's used to indicate a place of abode, whether permanent or temporary;
qualifications after the May 8, 1995 elections. "domicile" denotes a fixed permanent residence to which, when absent,
one has the intention of returning. A man may have a residence in one
I. Petitioner's qualification place and a domicile in another. Residence is not domicile, but domicile
is residence coupled with the intention to remain for an unlimited time.
A perusal of the Resolution of the COMELEC's Second Division reveals a startling A man can have but one domicile for the same purpose at any time, but
confusion in the application of settled concepts of "Domicile" and "Residence" in he may have numerous places of residence. His place of residence is
election law. While the COMELEC seems to be in agreement with the general generally his place of domicile, but it is not by any means necessarily so
proposition that for the purposes of election law, residence is synonymous with since no length of residence without intention of remaining will
domicile, the Resolution reveals a tendency to substitute or mistake the concept of constitute domicile.
domicile for actual residence, a conception not intended for the purpose of determining
a candidate's qualifications for election to the House of Representatives as required by For political purposes the concepts of residence and domicile are dictated by the
the 1987 Constitution. As it were, residence, for the purpose of meeting the peculiar criteria of political laws. As these concepts have evolved in our election law,
qualification for an elective position, has a settled meaning in our jurisdiction. what has clearly and unequivocally emerged is the fact that residence for election
purposes is used synonymously with domicile.
Article 50 of the Civil Code decrees that "[f]or the exercise of civil rights and the
fulfillment of civil obligations, the domicile of natural persons is their place of habitual In Nuval vs. Guray, 24 the Court held that "the term residence. . . is synonymous with
residence." In Ong vs. Republic 20 this court took the concept of domicile to mean an domicile which imports not only intention to reside in a fixed place, but also personal
individual's "permanent home", "a place to which, whenever absent for business or for presence in that place, coupled with conduct indicative of such intention." 25 Larena
pleasure, one intends to return, and depends on facts and circumstances in the sense vs. Teves 26 reiterated the same doctrine in a case involving the qualifications of the
that they disclose intent." 21 Based on the foregoing, domicile includes the twin respondent therein to the post of Municipal President of Dumaguete, Negros
elements of "the fact of residing or physical presence in a fixed place" and animus Oriental. Faypon vs. Quirino, 27 held that the absence from residence to pursue studies
manendi, or the intention of returning there permanently. or practice a profession or registration as a voter other than in the place where one is
elected does not constitute loss of residence. 28 So settled is the concept (of domicile) in
Residence, in its ordinary conception, implies the factual relationship of an individual our election law that in these and other election law cases, this Court has stated that the
to a certain place. It is the physical presence of a person in a given area, community or mere absence of an individual from his permanent residence without the intention to
country. The essential distinction between residence and domicile in law is that abandon it does not result in a loss or change of domicile.
residence involves the intent to leave when the purpose for which the resident has taken
up his abode ends. One may seek a place for purposes such as pleasure, business, or The deliberations of the 1987 Constitution on the residence qualification for certain
health. If a person's intent be to remain, it becomes his domicile; if his intent is to leave elective positions have placed beyond doubt the principle that when the Constitution
107
speaks of "residence" in election law, it actually means only "domicile" to wit: domicile. 32

Mr. Nolledo: With respect to Section 5, I remember that in the 1971 In the light of the principles just discussed, has petitioner Imelda Romualdez Marcos
Constitutional Convention, there was an attempt to require residence in satisfied the residency requirement mandated by Article VI, Sec. 6 of the 1987
the place not less than one year immediately preceding the day of the Constitution? Of what significance is the questioned entry in petitioner's Certificate of
elections. So my question is: What is the Committee's concept of Candidacy stating her residence in the First Legislative District of Leyte as seven (7)
residence of a candidate for the legislature? Is it actual residence or is it months?
the concept of domicile or constructive residence?
It is the fact of residence, not a statement in a certificate of candidacy which ought to
Mr. Davide: Madame President, insofar as the regular members of the be decisive in determining whether or not and individual has satisfied the constitution's
National Assembly are concerned, the proposed section merely provides, residency qualification requirement. The said statement becomes material only when
among others, "and a resident thereof", that is, in the district for a period there is or appears to be a deliberate attempt to mislead, misinform, or hide a fact which
of not less than one year preceding the day of the election. This was in would otherwise render a candidate ineligible. It would be plainly ridiculous for a
effect lifted from the 1973 Constitution, the interpretation given to it candidate to deliberately and knowingly make a statement in a certificate of candidacy
was domicile. 29 which would lead to his or her disqualification.

xxx xxx xxx It stands to reason therefore, that petitioner merely committed an honest mistake in
jotting the word "seven" in the space provided for the residency qualification
Mrs. Rosario Braid: The next question is on Section 7, page 2. I think requirement. The circumstances leading to her filing the questioned entry obviously
Commissioner Nolledo has raised the same point that "resident" has resulted in the subsequent confusion which prompted petitioner to write down the
been interpreted at times as a matter of intention rather than actual period of her actual stay in Tolosa, Leyte instead of her period of residence in the First
residence. district, which was "since childhood" in the space provided. These circumstances and
events are amply detailed in the COMELEC's Second Division's questioned resolution,
Mr. De los Reyes: Domicile. albeit with a different interpretation. For instance, when herein petitioner announced
that she would be registering in Tacloban City to make her eligible to run in the First
Ms. Rosario Braid: Yes, So, would the gentleman consider at the proper District, private respondent Montejo opposed the same, claiming that petitioner was a
time to go back to actual residence rather than mere intention to reside? resident of Tolosa, not Tacloban City. Petitioner then registered in her place of actual
residence in the First District, which is Tolosa, Leyte, a fact which she subsequently
Mr. De los Reyes: But we might encounter some difficulty especially noted down in her Certificate of Candidacy. A close look at said certificate would
considering that a provision in the Constitution in the Article on reveal the possible source of the confusion: the entry for residence (Item No. 7) is
Suffrage says that Filipinos living abroad may vote as enacted by law. followed immediately by the entry for residence in the constituency where a candidate
So, we have to stick to the original concept that it should be by domicile seeks election thus:
and not physical residence. 30
7. RESIDENCE (complete Address): Brgy. Olot, Tolosa, Leyte
31
In Co vs. Electoral Tribunal of the House of Representatives, this Court concluded
that the framers of the 1987 Constitution obviously adhered to the definition given to POST OFFICE ADDRESS FOR ELECTION PURPOSES: Brgy. Olot,
the term residence in election law, regarding it as having the same meaning as Tolosa, Leyte
108
8. RESIDENCE IN THE CONSTITUENCY WHERE I SEEK TO because she became a resident of many places" flies in the face of settled jurisprudence
BE ELECTED IMMEDIATELY PRECEDING THE in which this Court carefully made distinctions between (actual) residence and domicile
ELECTION:_________ Years and Seven Months. for election law purposes. In Larena vs. Teves, 33 supra, we stressed:

Having been forced by private respondent to register in her place of actual residence in [T]his court is of the opinion and so holds that a person who has his own
Leyte instead of petitioner's claimed domicile, it appears that petitioner had jotted down house wherein he lives with his family in a municipality without having
her period of stay in her legal residence or domicile. The juxtaposition of entries in ever had the intention of abandoning it, and without having lived either
Item 7 and Item 8 — the first requiring actual residence and the second requiring alone or with his family in another municipality, has his residence in the
domicile — coupled with the circumstances surrounding petitioner's registration as a former municipality, notwithstanding his having registered as an elector
voter in Tolosa obviously led to her writing down an unintended entry for which she in the other municipality in question and having been a candidate for
could be disqualified. This honest mistake should not, however, be allowed to negate various insular and provincial positions, stating every time that he is a
the fact of residence in the First District if such fact were established by means more resident of the latter municipality.
convincing than a mere entry on a piece of paper.
More significantly, in Faypon vs. Quirino, 34 We explained that:
We now proceed to the matter of petitioner's domicile.
A citizen may leave the place of his birth to look for "greener pastures,"
In support of its asseveration that petitioner's domicile could not possibly be in the First as the saying goes, to improve his lot, and that, of course includes study
District of Leyte, the Second Division of the COMELEC, in its assailed Resolution of in other places, practice of his avocation, or engaging in business. When
April 24,1995 maintains that "except for the time when (petitioner) studied and worked an election is to be held, the citizen who left his birthplace to improve
for some years after graduation in Tacloban City, she continuously lived in Manila." his lot may desire to return to his native town to cast his ballot but for
The Resolution additionally cites certain facts as indicative of the fact that petitioner's professional or business reasons, or for any other reason, he may not
domicile ought to be any place where she lived in the last few decades except Tacloban, absent himself from his professional or business activities; so there he
Leyte. First, according to the Resolution, petitioner, in 1959, resided in San Juan, Metro registers himself as voter as he has the qualifications to be one and is not
Manila where she was also registered voter. Then, in 1965, following the election of willing to give up or lose the opportunity to choose the officials who are
her husband to the Philippine presidency, she lived in San Miguel, Manila where she as to run the government especially in national elections. Despite such
a voter. In 1978 and thereafter, she served as a member of the Batasang Pambansa and registration, the animus revertendi to his home, to his domicile or
Governor of Metro Manila. "She could not, have served these positions if she had not residence of origin has not forsaken him. This may be the explanation
been a resident of Metro Manila," the COMELEC stressed. Here is where the confusion why the registration of a voter in a place other than his residence of
lies. origin has not been deemed sufficient to constitute abandonment or loss
of such residence. It finds justification in the natural desire and longing
We have stated, many times in the past, that an individual does not lose his domicile of every person to return to his place of birth. This strong feeling of
even if he has lived and maintained residences in different places. Residence, it bears attachment to the place of one's birth must be overcome by positive
repeating, implies a factual relationship to a given place for various purposes. The proof of abandonment for another.
absence from legal residence or domicile to pursue a profession, to study or to do other
things of a temporary or semi-permanent nature does not constitute loss of residence. From the foregoing, it can be concluded that in its above-cited statements supporting its
Thus, the assertion by the COMELEC that "she could not have been a resident of proposition that petitioner was ineligible to run for the position of Representative of the
Tacloban City since childhood up to the time she filed her certificate of candidacy First District of Leyte, the COMELEC was obviously referring to petitioner's various
109
places of (actual) residence, not her domicile. In doing so, it not only ignored settled presidency, at the height of the Marcos Regime's powers, petitioner kept her close ties
jurisprudence on residence in election law and the deliberations of the constitutional to her domicile of origin by establishing residences in Tacloban, celebrating her
commission but also the provisions of the Omnibus Election Code (B.P. 881). 35 birthdays and other important personal milestones in her home province, instituting
well-publicized projects for the benefit of her province and hometown, and establishing
What is undeniable, however, are the following set of facts which establish the fact of a political power base where her siblings and close relatives held positions of power
petitioner's domicile, which we lift verbatim from the COMELEC's Second Division's either through the ballot or by appointment, always with either her influence or
assailed Resolution: 36 consent. These well-publicized ties to her domicile of origin are part of the history and
lore of the quarter century of Marcos power in our country. Either they were entirely
In or about 1938 when respondent was a little over 8 years old, she ignored in the COMELEC'S Resolutions, or the majority of the COMELEC did not
established her domicile in Tacloban, Leyte (Tacloban City). She studied know what the rest of the country always knew: the fact of petitioner's domicile in
in the Holy Infant Academy in Tacloban from 1938 to 1949 when she Tacloban, Leyte.
graduated from high school. She pursued her college studies in St. Paul's
College, now Divine Word University in Tacloban, where she earned her Private respondent in his Comment, contends that Tacloban was not petitioner's
degree in Education. Thereafter, she taught in the Leyte Chinese School, domicile of origin because she did not live there until she was eight years old. He avers
still in Tacloban City. In 1952 she went to Manila to work with her that after leaving the place in 1952, she "abandoned her residency (sic) therein for
cousin, the late speaker Daniel Z. Romualdez in his office in the House many years and . . . (could not) re-establish her domicile in said place by merely
of Representatives. In 1954, she married ex-President Ferdinand E. expressing her intention to live there again." We do not agree.
Marcos when he was still a congressman of Ilocos Norte and registered
there as a voter. When her husband was elected Senator of the Republic First, minor follows the domicile of his parents. As domicile, once acquired is retained
in 1959, she and her husband lived together in San Juan, Rizal where until a new one is gained, it follows that in spite of the fact of petitioner's being born in
she registered as a voter. In 1965, when her husband was elected Manila, Tacloban, Leyte was her domicile of origin by operation of law. This domicile
President of the Republic of the Philippines, she lived with him in was not established only when her father brought his family back to Leyte contrary to
Malacanang Palace and registered as a voter in San Miguel, Manila. private respondent's averments.

[I]n February 1986 (she claimed that) she and her family were abducted Second, domicile of origin is not easily lost. To successfully effect a change of
and kidnapped to Honolulu, Hawaii. In November 1991, she came home domicile, one must demonstrate: 37
to Manila. In 1992, respondent ran for election as President of the
Philippines and filed her Certificate of Candidacy wherein she indicated 1. An actual removal or an actual change of domicile;
that she is a resident and registered voter of San Juan, Metro Manila.
2. A bona fide intention of abandoning the former place of residence and
Applying the principles discussed to the facts found by COMELEC, what is establishing a new one; and
inescapable is that petitioner held various residences for different purposes during the
last four decades. None of these purposes unequivocally point to an intention to 3. Acts which correspond with the purpose.
abandon her domicile of origin in Tacloban, Leyte. Moreover, while petitioner was
born in Manila, as a minor she naturally followed the domicile of her parents. She grew In the absence of clear and positive proof based on these criteria, the residence of origin
up in Tacloban, reached her adulthood there and eventually established residence in should be deemed to continue. Only with evidence showing concurrence of all three
different parts of the country for various reasons. Even during her husband's requirements can the presumption of continuity or residence be rebutted, for a change
110
of residence requires an actual and deliberate abandonment, and one cannot have two residencia. Los Tribunales, sin embargo, podran con justa causa eximirla
legal residences at the same time. 38 In the case at bench, the evidence adduced by de esta obligacion cuando el marido transende su residencia a ultramar
private respondent plainly lacks the degree of persuasiveness required to convince this o' a pais extranjero.
court that an abandonment of domicile of origin in favor of a domicile of choice indeed
occurred. To effect an abandonment requires the voluntary act of relinquishing Note the use of the phrase "donde quiera su fije de residencia" in the aforequoted
petitioner's former domicile with an intent to supplant the former domicile with one of article, which means wherever (the husband) wishes to establish residence. This part of
her own choosing (domicilium voluntarium). the article clearly contemplates only actual residence because it refers to a positive act
of fixing a family home or residence. Moreover, this interpretation is further
In this connection, it cannot be correctly argued that petitioner lost her domicile of strengthened by the phrase "cuando el marido translade su residencia" in the same
origin by operation of law as a result of her marriage to the late President Ferdinand E. provision which means, "when the husband shall transfer his residence," referring to
Marcos in 1952. For there is a clearly established distinction between the Civil Code another positive act of relocating the family to another home or place of actual
concepts of "domicile" and "residence." 39 The presumption that the wife automatically residence. The article obviously cannot be understood to refer to domicile which is a
gains the husband's domicile by operation of law upon marriage cannot be inferred fixed,
from the use of the term "residence" in Article 110 of the Civil Code because the Civil fairly-permanent concept when it plainly connotes the possibility of transferring from
Code is one area where the two concepts are well delineated. Dr. Arturo Tolentino, one place to another not only once, but as often as the husband may deem fit to move
writing on this specific area explains: his family, a circumstance more consistent with the concept of actual residence.

In the Civil Code, there is an obvious difference between domicile and The right of the husband to fix the actual residence is in harmony with the intention of
residence. Both terms imply relations between a person and a place; but the law to strengthen and unify the family, recognizing the fact that the husband and the
in residence, the relation is one of fact while in domicile it is legal or wife bring into the marriage different domiciles (of origin). This difference could, for
juridical, independent of the necessity of physical presence. 40 the sake of family unity, be reconciled only by allowing the husband to fix a single
place of actual residence.
Article 110 of the Civil Code provides:
Very significantly, Article 110 of the Civil Code is found under Title V under the
Art. 110. — The husband shall fix the residence of the family. But the heading: RIGHTS AND OBLIGATIONS BETWEEN HUSBAND AND WIFE.
court may exempt the wife from living with the husband if he should Immediately preceding Article 110 is Article 109 which obliges the husband and wife
live abroad unless in the service of the Republic. to live together, thus:

A survey of jurisprudence relating to Article 110 or to the concepts of domicile or Art. 109. — The husband and wife are obligated to live together,
residence as they affect the female spouse upon marriage yields nothing which would observe mutual respect and fidelity and render mutual help and support.
suggest that the female spouse automatically loses her domicile of origin in favor of the
husband's choice of residence upon marriage. The duty to live together can only be fulfilled if the husband and wife are physically
together. This takes into account the situations where the couple has many residences
Article 110 is a virtual restatement of Article 58 of the Spanish Civil Code of 1889 (as in the case of the petitioner). If the husband has to stay in or transfer to any one of
which states: their residences, the wife should necessarily be with him in order that they may "live
together." Hence, it is illogical to conclude that Art. 110 refers to "domicile" and not to
La mujer esta obligada a seguir a su marido donde quiera que fije su "residence." Otherwise, we shall be faced with a situation where the wife is left in the
111
domicile while the husband, for professional or other reasons, stays in one of their Upon examination of the authorities, we are convinced that it is not
(various) residences. As Dr. Tolentino further explains: within the province of the courts of this country to attempt to compel
one of the spouses to cohabit with, and render conjugal rights to, the
Residence and Domicile — Whether the word "residence" as used with other. Of course where the property rights of one of the pair are invaded,
reference to particular matters is synonymous with "domicile" is a an action for restitution of such rights can be maintained. But we are
question of some difficulty, and the ultimate decision must be made disinclined to sanction the doctrine that an order, enforcible (sic) by
from a consideration of the purpose and intent with which the word is process of contempt, may be entered to compel the restitution of the
used. Sometimes they are used synonymously, at other times they are purely personal right of consortium. At best such an order can be
distinguished from one another. effective for no other purpose than to compel the spouses to live under
the same roof; and he experience of those countries where the courts of
xxx xxx xxx justice have assumed to compel the cohabitation of married people
shows that the policy of the practice is extremely questionable. Thus in
Residence in the civil law is a material fact, referring to the physical England, formerly the Ecclesiastical Court entertained suits for the
presence of a person in a place. A person can have two or more restitution of conjugal rights at the instance of either husband or wife;
residences, such as a country residence and a city residence. Residence and if the facts were found to warrant it, that court would make a
is acquired by living in place; on the other hand, domicile can exist mandatory decree, enforceable by process of contempt in case of
without actually living in the place. The important thing for domicile is disobedience, requiring the delinquent party to live with the other and
that, once residence has been established in one place, there be an render conjugal rights. Yet this practice was sometimes criticized even
intention to stay there permanently, even if residence is also established by the judges who felt bound to enforce such orders, and in Weldon
in some other v. Weldon (9 P.D. 52), decided in 1883, Sir James Hannen, President in
place. 41 the Probate, Divorce and Admiralty Division of the High Court of
Justice, expressed his regret that the English law on the subject was not
In fact, even the matter of a common residence between the husband and the wife the same as that which prevailed in Scotland, where a decree of
during the marriage is not an iron-clad principle; In cases applying the Civil Code on adherence, equivalent to the decree for the restitution of conjugal rights
the question of a common matrimonial residence, our jurisprudence has recognized in England, could be obtained by the injured spouse, but could not be
certain situations 42 where the spouses could not be compelled to live with each other enforced by imprisonment. Accordingly, in obedience to the growing
such that the wife is either allowed to maintain a residence different from that of her sentiment against the practice, the Matrimonial Causes Act (1884)
husband or, for obviously practical reasons, revert to her original domicile (apart from abolished the remedy of imprisonment; though a decree for the
being allowed to opt for a new one). In De la Vina vs. Villareal 43 this Court held that restitution of conjugal rights can still be procured, and in case of
"[a] married woman may acquire a residence or domicile separate from that of her disobedience may serve in appropriate cases as the basis of an order for
husband during the existence of the marriage where the husband has given cause for the periodical payment of a stipend in the character of alimony.
divorce." 44 Note that the Court allowed the wife either to obtain new residence or to
choose a new domicile in such an event. In instances where the wife actually opts, In the voluminous jurisprudence of the United States, only one court, so
.under the Civil Code, to live separately from her husband either by taking new far as we can discover, has ever attempted to make a preemptory order
residence or reverting to her domicile of origin, the Court has held that the wife could requiring one of the spouses to live with the other; and that was in a case
not be compelled to live with her husband on pain of contempt. In Arroyo vs. Vasques where a wife was ordered to follow and live with her husband, who had
de Arroyo 45 the Court held that: changed his domicile to the City of New Orleans. The decision referred
112
to (Bahn v. Darby, 36 La. Ann., 70) was based on a provision of the making the choice of domicile a product of mutual agreement between the spouses. 46
Civil Code of Louisiana similar to article 56 of the Spanish Civil Code.
It was decided many years ago, and the doctrine evidently has not been Without as much belaboring the point, the term residence may mean one thing in civil
fruitful even in the State of Louisiana. In other states of the American law (or under the Civil Code) and quite another thing in political law. What stands clear
Union the idea of enforcing cohabitation by process of contempt is is that insofar as the Civil Code is concerned-affecting the rights and obligations of
rejected. (21 Cyc., 1148). husband and wife — the term residence should only be interpreted to mean "actual
residence." The inescapable conclusion derived from this unambiguous civil law
In a decision of January 2, 1909, the Supreme Court of Spain appears to delineation therefore, is that when petitioner married the former President in 1954, she
have affirmed an order of the Audiencia Territorial de Valladolid kept her domicile of origin and merely gained a new home, not a domicilium
requiring a wife to return to the marital domicile, and in the alternative, necessarium.
upon her failure to do so, to make a particular disposition of certain
money and effects then in her possession and to deliver to her husband, Even assuming for the sake of argument that petitioner gained a new "domicile" after
as administrator of the ganancial property, all income, rents, and interest her marriage and only acquired a right to choose a new one after her husband died,
which might accrue to her from the property which she had brought to petitioner's acts following her return to the country clearly indicate that she not only
the marriage. (113 Jur. Civ., pp. 1, 11) But it does not appear that this impliedly but expressly chose her domicile of origin (assuming this was lost by
order for the return of the wife to the marital domicile was sanctioned by operation of law) as her domicile. This "choice" was unequivocally expressed in her
any other penalty than the consequences that would be visited upon her letters to the Chairman of the PCGG when petitioner sought the PCGG's permission to
in respect to the use and control of her property; and it does not appear "rehabilitate (our) ancestral house in Tacloban and Farm in Olot, Leyte. . . to make
that her disobedience to that order would necessarily have been followed them livable for the Marcos family to have a home in our homeland." 47 Furthermore,
by imprisonment for contempt. petitioner obtained her residence certificate in 1992 in Tacloban, Leyte, while living in
her brother's house, an act which supports the domiciliary intention clearly manifested
Parenthetically when Petitioner was married to then Congressman Marcos, in 1954, in her letters to the PCGG Chairman. She could not have gone straight to her home in
petitioner was obliged — by virtue of Article 110 of the Civil Code — to follow her San Juan, as it was in a state of disrepair, having been previously looted by vandals.
husband's actual place of residence fixed by him. The problem here is that at that time, Her "homes" and "residences" following her arrival in various parts of Metro Manila
Mr. Marcos had several places of residence, among which were San Juan, Rizal and merely qualified as temporary or "actual residences," not domicile. Moreover, and
Batac, Ilocos Norte. There is no showing which of these places Mr. Marcos did fix as proceeding from our discussion pointing out specific situations where the female
his family's residence. But assuming that Mr. Marcos had fixed any of these places as spouse either reverts to her domicile of origin or chooses a new one during the
the conjugal residence, what petitioner gained upon marriage was actual residence. She subsistence of the marriage, it would be highly illogical for us to assume that she
did not lose her domicile of origin. cannot regain her original domicile upon the death of her husband absent a positive act
of selecting a new one where situations exist within the subsistence of the marriage
On the other hand, the common law concept of "matrimonial domicile" appears to have itself where the wife gains a domicile different from her husband.
been incorporated, as a result of our jurisprudential experiences after the drafting of the
Civil Code of 1950, into the New Family Code. To underscore the difference between In the light of all the principles relating to residence and domicile enunciated by this
the intentions of the Civil Code and the Family Code drafters, the term residence has court up to this point, we are persuaded that the facts established by the parties weigh
been supplanted by the term domicile in an entirely new provision (Art. 69) distinctly heavily in favor of a conclusion supporting petitioner's claim of legal residence or
different in meaning and spirit from that found in Article 110. The provision recognizes domicile in the First District of Leyte.
revolutionary changes in the concept of women's rights in the intervening years by
113
II. The jurisdictional issue In any event, with the enactment of Sections 6 and 7 of R.A. 6646 in relation to Section
78 of B.P. 881, 52 it is evident that the respondent Commission does not lose jurisdiction
Petitioner alleges that the jurisdiction of the COMELEC had already lapsed considering to hear and decide a pending disqualification case under Section 78 of B.P. 881 even
that the assailed resolutions were rendered on April 24, 1995, fourteen (14) days before after the elections.
the election in violation of Section 78 of the Omnibus Election Code. 48 Moreover,
petitioner contends that it is the House of Representatives Electoral Tribunal and not As to the House of Representatives Electoral Tribunal's supposed assumption of
the COMELEC which has jurisdiction over the election of members of the House of jurisdiction over the issue of petitioner's qualifications after the May 8, 1995 elections,
Representatives in accordance with Article VI Sec. 17 of the Constitution. This is suffice it to say that HRET's jurisdiction as the sole judge of all contests relating to the
untenable. elections, returns and qualifications of members of Congress begins only after a
candidate has become a member of the House of Representatives. 53 Petitioner not
It is a settled doctrine that a statute requiring rendition of judgment within a specified being a member of the House of Representatives, it is obvious that the HRET at this
time is generally construed to be merely directory, 49 "so that non-compliance with point has no jurisdiction over the question.
them does not invalidate the judgment on the theory that if the statute had intended
such result it would have clearly indicated it." 50 The difference between a mandatory It would be an abdication of many of the ideals enshrined in the 1987 Constitution for
and a directory provision is often made on grounds of necessity. Adopting the same us to either to ignore or deliberately make distinctions in law solely on the basis of the
view held by several American authorities, this court in Marcelino vs. Cruz held that: 51 personality of a petitioner in a case. Obviously a distinction was made on such a ground
here. Surely, many established principles of law, even of election laws were flouted for
The difference between a mandatory and directory provision is often the sake perpetuating power during the pre-EDSA regime. We renege on these sacred
determined on grounds of expediency, the reason being that less injury ideals, including the meaning and spirit of EDSA ourselves bending established
results to the general public by disregarding than enforcing the letter of principles of principles of law to deny an individual what he or she justly deserves in
the law. law. Moreover, in doing so, we condemn ourselves to repeat the mistakes of the past.

In Trapp v. Mc Cormick, a case calling for the interpretation of a statute WHEREFORE, having determined that petitioner possesses the necessary residence
containing a limitation of thirty (30) days within which a decree may be qualifications to run for a seat in the House of Representatives in the First District of
entered without the consent of counsel, it was held that "the statutory Leyte, the COMELEC's questioned Resolutions dated April 24, May 7, May 11, and
provisions which may be thus departed from with impunity, without May 25, 1995 are hereby SET ASIDE. Respondent COMELEC is hereby directed to
affecting the validity of statutory proceedings, are usually those which order the Provincial Board of Canvassers to proclaim petitioner as the duly elected
relate to the mode or time of doing that which is essential to effect the Representative of the First District of Leyte.
aim and purpose of the Legislature or some incident of the essential act."
Thus, in said case, the statute under examination was construed merely MARCOS vs COMELEC
to be directory.
(residence and domicile, for purposes of election laws are synonymous…)
The mischief in petitioner's contending that the COMELEC should have abstained from The mischief which this provision — reproduced verbatim from the 1973 Constitution — seeks
rendering a decision after the period stated in the Omnibus Election Code because it to prevent is the possibility of a
"stranger or newcomer unacquainted with the conditions and needs of a community and not
lacked jurisdiction, lies in the fact that our courts and other quasi-judicial bodies would
identified with the latter, from an
then refuse to render judgments merely on the ground of having failed to reach a elective office to serve that community."
decision within a given or prescribed period. Petitioner Imelda Romualdez-Marcos filed her Certificate of Candidacy for the position of
114
Representative of the First District of studies or practice a profession or registration as a voter other than in the place where one is
Leyte with the Provincial Election Supervisor on March 8, 1995. elected does not constitute loss of
Private respondent Cirilo Roy Montejo, the incumbent Representative of the First District of residence. So settled is the concept (of domicile) in our election law that in these and other
Leyte and a candidate for the same election law cases, this Court has
position, filed a "Petition for Cancellation and Disqualification" with the Commission on stated that the mere absence of an individual from his permanent residence without the
Elections alleging that petitioner did not intention to abandon it does not result in a
meet the constitutional requirement for residency. In his petition, private respondent contended loss or change of domicile.
that Mrs. Marcos lacked the In Co vs. Electoral Tribunal of the House of Representatives, this Court concluded that the
Constitution's one year residency requirement for candidates for the House of Representatives. framers of the 1987 Constitution
Held: obviously adhered to the definition given to the term residence in election law, regarding it as
So settled is the concept (of domicile) in our election law that in these and other election law having the same meaning as
cases, this Court has stated that the domicile.
mere absence of an individual from his permanent residence without the intention to A Person cannot have 2 domiciles. As long as the Domicile was not lost, it continues to be
abandon it does not result in a loss the same until replaced by a new
or change of domicile. one. Marcos did not overtedly abandon her domicile since even if living in Malakanyang,
It stands to reason therefore, that petitioner merely committed an honest mistake in jotting the she constantly goes home to her
word "seven" in the space provided domicile.
for the residency qualification requirement. It would be plainly ridiculous for a candidate to Prior to this:
deliberately and knowingly make a The civil code provides that the wife follows the domicile of her husband.
statement in a certificate of candidacy which would lead to his or her disqualification.
Residence in the civil law is a material fact, referring to the physical presence of a person in a
place. A person can have two or
more residences, such as a country residence and a city residence. Residence is acquired by
living in place; on the other hand,
domicile can exist without actually living in the place. The important thing for domicile is that,
once residence has been
established in one place, there be an intention to stay there permanently, even if residence is
also established in some other place.
For political purposes the concepts of residence and domicile are dictated by the peculiar
criteria of political laws. As these
concepts have evolved in our election law, what has clearly and unequivocally emerged is
the fact that residence for
election purposes is used synonymously with domicile.
In Nuval vs. Guray, the Court held that "the term residence. . . is synonymous with
domicile which imports not only
intention to reside in a fixed place, but also personal presence in that place, coupled with
conduct indicative of such
intention." Larena vs. Teves reiterated the same doctrine in a case involving the qualifications
of the respondent therein to the
post of Municipal President of Dumaguete, Negros Oriental. Faypon vs. Quirino, held that the
absence from residence to pursue
115

Anda mungkin juga menyukai